Sol MBA-2020 QA-Class Handouts 01-33

You might also like

Download as pdf or txt
Download as pdf or txt
You are on page 1of 121

QA - 01 : Percentage - 1 CEX-Q-0202/20

Answers and Explanations

21 3 22 1 23 3 24 4 25 2 26 2 27 3 28 2 29 4 30 4
31 1 32 1 33 2 34 3 35 1

1. 3363. 15. Here, base = 1000


Therefore, (1012)2 = 1000 + 2 × 12 | (12)2
2. 4536. = 1024 | 144 = 1024144.

3. 7224.
(30 + 1)3 = (30 ) + (1) + 3.30.1(30 + 1)
3 3
16.
4. 8184. = 27000 + 1 + 2790 = 29791.

5. 10528.
17. Base 100. Answer is (100 – 27) | 3 × ( −9 )2 | ( −9 )3
6. 12312. = 73 | 243 | – 729
= 73 | 235 | 800 – 729 {Taking borrow 8 from the other
7. 187335 side}
= 73 | 235 | 71 = 73 + 2( = 75) | 35 | 71 = 753571.
8. 192832.
18. Base 100. Answer is (100 + 36) | 3 × 144 | 1728
9. 44100. = 1404928.

10. 537462. 19. Base 1000. Answer is (1000 + 15) | 3 × 25 | 125


= 1015075125.
11. Here, base = 100

Therefore, (92 )2 = 100 + 2 × ( −8) | ( −8 )2 20. Base 1000. Answer is (1000 – 9) | 3 × ( −3 ) | ( −3 )


2 3

= 84 | 64 = 991 | 27 | – 27
= 8464. = 991 | 026 | 1000 – 27 = 991026973.

12. Here, base = 100


Therefore, (108)2 = (100 + 2 × 8) | 82 338 2
21. 3 Since ≈
= 116 | 64 = 11664. 473 3

13. Here, base = 1000 2


338 + 27 ×
338 3 = 356 = 0.712 i.e. 71.2%.
∴ =
Therefore, (993 )2
= 1000 − 2 × 7 | ( −7 ) = 986 | 049
2
473 473 + 27 500
= 986049.

1
14. Here, base = 1000 22. 1 Since = 14.28%
Therefore, 7

(1008 )2 = 1000 + 2 × 8 | (8 )2 = 1016 | 064 = 1016064. ∴


3
= 14.28 × 3 = 42.84% .
7

QA - 01 Page 1
443 1 30 40 80
23. 3 Since ≈ 29. 4 ×A+ ×B = ×B
898 2 100 100 100
⇒ 30A + 40B = 80B
1
443 + 102 × ⇒ 30A = 40B
Therefore, 443 = 2 = 494
898 898 + 102 1000 30
Percentage of A is B = × 100 = 75%.
= 0.494 i.e, 49.4%. 40

90 30
1 30. 4 ×A = ×B
24. 4 Since ≈ 5.26% 100 100
19
B 90
8 ⇒ = =3
∴ = 8 × 5.26 = 42.08%. A 30
19
B 2x
=
A 100
547 3
25. 2 Since ≈
1973 11 2x
∴3 =
100
3
547 + × 27 ⇒ 2x = 300 ⇒ x = 150.
547 11 547 + 7.36
Therefore, = =
1973 1973 + 27 2000
3 1
554.36 31. 1 X = 37.5% of 20% of 48 = × × 48 = 3.6
= = 0.27718 ≈ 27.72% . 8 5
2000
1 3
Y = 14.28% of 27.27% of 77 = × × 77 = 3
26. 2 1.6 × 1.12 = 1.792 7 11
1.792 0.13 13 13 ∴ X > Y.
= 25% + = 25% + , ; 2%.
6.63 6.63 663 663
Thus, the answer = 27.02% 42
32. 1 Passing marks = × 250 = 105
100
1 He is failed by 105 – 95 = 10 marks.
27. 3 Since = 5.26%
19
33. 2 Let x be the number. Then,
5
⇒ = 5 × 5.26 = 26.3%. 74
19 × x = 555 ⇒ x = 750
100

28. 2 (i) 20% of x = 10 44


∴ 44% of 750 = × 750 = 330.
100
20
⇒ × x = 10 ⇒ x = 50 .
100
(ii) 8% of x = 90 34. 3 40.96 + 0.4096 + 0.004096 + 0.00004096
8 = 6.4 + 0.64 + 0.064 + 0.0064
⇒ × x = 90 ⇒ x = 1125 = 7.1104.
100
(iii) 15% of x = 15
1 a+b
15 35. 1 =
⇒ × x = 15 ⇒ x = 100 (a + b) ab
100
(iv) 17.5% of x = 35 ⇒ (a + b)2 − ab = 0
17.5
⇒ × x = 35 ⇒ x = 200. ⇒ a2 + b2 + ab = 0
100

Page 2 QA - 01
QA - 02 : Percentage - 2 CEX-Q-0203/20
Answers and Explanations

1 4 2 1 3 3 4 4 5 2 6 2 7 2 8 3 9 4 10 1
11 1 12 1 13 3 14 4 15 – 16 – 17 2 18 1 19 4 20 3
21 3 22 2 23 3 24 – 25 3 26 4 27 3 28 3 29 – 30 3

1. 4 Let initially the sides be x, y and z. 7. 2 Let time taken to answer a question = t
Initial volume = xyz Ist IInd IIIrd IVth
After the change sides will be 1.2x, 1.2y and 1.2z. Read 12t 12t 12t 12t → 48t
New volume = 1.728(xyz) Q’s 5t 8t 8t 6t → 27t
∴ Increase in volume is 72.8%. Total 17t 20t 20t 18t → 75t
Total time = 75t
2. 1 The percentage of charcoal 10% less = 67.5t
= [100 – (75 + 10)] % = 15 % ⇒ Time spent on only reading all 4 passages
Quantity of charcoal = 15 % of 24 kg = 67.5t – 27t = 40.5t
15 40.5
= ´ 24 = 3.6 kg. ∴ Each passage requires = = 10.125t
100 4

12t
3. 3 Let total number of votes be x. He has to increase his speed by = 1.185
Then, 30% of x + 15000 = 70% of x 10.125t
⇒ 15000 = 0.4x Increase in speed = 1.185 – 1 = 0.185 i.e. 18.5%
⇒ x = 37500
The required number = 70% of 37500 8. 3 Expenditure in this case will be 0.96 × 0.92 times the
70 earlier expenditure. Using the base of 100, the product
= ´ 37500 = 26250 will be (96 – 8) × 100 + (–4) × (–8) = 0.8832
100
Thus, the expenditure will decrease by
4. 4 Net percentage change (1 – 0.8832) × 100 = 11.68%.

ab 30 ´ 30 9. 4 Percentage of boys who play cricket in the class


=a+b+ = 30 + 30 + = 69%
100 100
70 10
= ´ ´100 = 7%
100 100
5. 2 Suppose, he has 100 fruits. He sells 30 fruits and
throws away 40% of 70 i.e. 28 fruits. Now he was 3 14% of students are cricket players. (Given)
left with 42 fruits. Next day, he sells 50% of 42 = 21 ∴ Percentage of cricket players who are boys
fruits. So, he throws 21 + 28 = 49 fruits.
7
Hence, the required percentage = 49.
= 100 ´100 = 50%.
14
6. 2 Let the original price of the bicycle be Rs. x. Then,
100
 9.09   8.33   7.7 
x × 1 + × 1+ × 1+ = 1274
 100   100   100 
10. 1 Let there be 100 products in the stockpile.
Hence, products from M1 = 40, from M2 = 30 and from
 1  1  1
⇒ x × 1 +  × 1 +  × 1 +  = 1274 M3 = 30. Number of defective products from
 11   12   13  M1 = 0.03 × 40 = 1.2, from M2 = 0.01 × 30 = 0.3 and
from M3 = 0.05 × 30 = 1.5
12 13 14
⇒ x× × × = 1274 Therefore, total number of defective products = 3,
11 12 13
and percentage of defective products = 3%.
⇒ x = 91 × 11 = Rs. 1,001.

QA - 02 Page 1
Alternative method: 18. 1 Let weight of silver and copper be x and y respectively,
Total percentage defectives = 3% of 40% + 1% of in alloy A
30% + 5% of 30% = 3% of total. Alloy B Alloy C
Silver : x + 15 x+9
11. 1 We have 7(x + y) = 4x + 8y where x and y are the Copper : y y
production for first and second quarter. The equation x + 15 90
=
gives x : y = 1 : 3 From B : x + y + 15 100 ⇒ x – 9y = –15

12. 1 Given that, initially A : B = 4 : 5 x+9 84


=
After one year, we have A = 5x, B = 5x + 50,000 From C : x + y + 10 100 ⇒ 4x – 21y = –15
As per the question, we have ∴ x = 12, y = 3
5x 9 12
= ∴ Percentage of silver in A = ´100 = 80%
5x + 50,000 10 15

⇒ x = 90,000 19. 4 For a difference of 1 year, CI can be computed as SI.


which gives A = Rs. 3,60,000 Hence, from the 2nd year to the 3rd year, interest
earned = (675 – 625) = Rs. 50 on Rs. 625.
13. 3 Let total score = x Hence, the rate of interst
Passing marks = 0.39x + 58 = 0.55x – 22 50
⇒ 80 = 0.16x = × 100 = 8% per annum
625
80
⇒x= ´100 Alternative method :
16 Since the values are for consecutive years,
Hence, maximum marks = 500 3rd year value 675 2
∴ = = 1 ,
2nd year value 625 25
14. 4 Weight of first lump = x kg
which is the interst component.
Weight of second lump = (20 – x) kg
31.25x + 30(20 – x) = 617.5 ⇒ x = 14 kg 1
= 4%
Total gold = 75% of 14 + 85% of 6 kg = 15.6 kg 25
2
15.6 ∴ = 8%
Now percentage of gold = ´100 = 78% 25
20

20. 3 Let p be the principal value, r% be the rate of interest


15. We have (A – 20 ) = 0.4(B + 20), i.e. A – 0.4B = 28 and n = 2 = duration.
And (B – 40) = 0.4(A + 40), i.e. B – 0.4A = 56 Thus, according to the question
Solving, we get A = Rs. 60
 2  2r 
  r 
P   1 +  − 1 −  = 160
16. Let x be the total number of people, college will ask for  100  
100 
  
donations.
∴ People already solicited = 0.6x
 r2 
Amount raised from the people solicited  
⇒ P = 160
2
...(i)
= 600 × 0.6x = 360x  (100 ) 
Now 360x constitutes 75% of the amount.
Hence, remaining 25% constitutes 120x P×r ×2
∴ Average donation from remaining people Also 2880 =
100
120x
= = 300 2880 × 100
0.4x ⇒ Pr = ...(ii)
2
∴ Using (ii) in (i), we get
17. 2 Assume x be population of the town.
2880 × 100
Number of people who work r = 160 × 100 × 100
2
= 0.37x × 0.76 + 0.63x × 0.54 = 0.6214x
⇒ 0.6214x = 124280 16 × 100 × 2 100 1
r= = = 11 %
⇒ x = 2,00,000 288 9 9

Page 2 QA - 02
Alternative method : 24. Let x be the sum that he lent to his friend.
Let r be the rate of interest.
x ´ 8 ´ 4 (10000 - x) ´12 ´ 4
Simple Interest and Compound Interest for first year ∴ + = 3800
100 100
2880
will be same = = Rs.1440 x = Rs. 6250.
2
Difference between CI and SI = SI on SI of first year, 25. 3 SI for 2 years = 354.
1440 × 1× r 254
160 = Hence, for one year = = 177.
100 2
(Let’s assume the sums of Narayan and Murthy are x
160 × 100 100 1
∴r= = % = 11 % and y respectively.)
144 9 9
6x 7y
∴ 177 = + ⇒ 6x + 7y = 17700 ...(i)
21. 3 Suppose principal is Rs. x and it doubles in time t. 100 100
We have
x y 4
∴ = ⇒x= y
2t 4 5 5
 20 / 2 
2x = x  1 +
 100  Putting the value of x in (i), we get
2t
or 2 = (1.1) 4
6 ´ y + 7y = 17700
⇒ 2t = 8 5
or t = 4 years. (approximately)
24y
+ 7y = 17700
22. 2 Suppose the principal is P, r = 5% (given) 5
2 ∴ y = 1500 and x = 1200
 r 
CI – SI (for 2 years) = P   Hence, total sum (x + y) = 1500 + 1200 = Rs. 2,700.
 100 
Alternative method:
2
 5 
or 100 = P   1 1
 100  Narayan’s investment × = Murthy’s investment ×
4 5
or P = Rs. 40,000
∴ Narayan : Murthy = 4 : 5
Let Narayan be 4x ⇒ murthy = 5x.
23. 3 Compound interest (CI) = Amount – Principle (P)
Now Narayan + Murthy = 9x has to be divisible by 9.
æ r ö÷
n ∴ (a), (b) and (e) are eliminated.
= P ççç1 + ÷÷ - P Try (c) and (d) and find the interest earned in each
è 100 ø
case.

éæ r ö÷
n ù
êç ú 26. 4 Difference of sum after 2 and 3 years = 1,728 – 1,440
= P êççè1 + 100 ÷÷ø - 1ú , where = Rs. 288
êë úû
Rs. 288 is the simple interest on Rs. 1,440 for one
r is the rate of interest, n is the duration year.

éæ 2 ù 1440 × r × 1
ê 10 ö÷ 288 = ⇒ r = 20% .
Now, CI = P êççç1 + ÷÷ - 1úú 100
êëè 100 ø úû
27. 3 Let three parts be a, b and c where a > b > c
éæ 11ö2 ù é121- 100 ù 21P Therefore, by the simple interest formula
420 = P êêçç ÷÷÷ - 1úú = P ê ú =
ç
êëè10 ø úû ê
ë 100 û ú 100 1.04a = 1.08b = 1.12c and a + b + c = 2189
Solving above equations, we get c = 703
⇒ P = Rs. 2,000
28. 3 Let, the amount that was invested by Rohit in bank be
P ´r ´ t P ´10 ´ 2 'C'.
∴ SI = =
100 100 1.2(1.2C – 200) = 400
⇒ 1.44C = 240 + 400
2000 ´10 ´ 2
= = Rs. 400 640 4000
100 ∴C= = Rs.
1.44 9

QA - 02 Page 3
8 æ 5 ÷ö
4
æ 10 ÷ö
2
29. Interest earned on first part = 3500 ´ 3 ´ = Rs. Difference = 5000 ççç1 + ÷ - 5000 çç1 + ÷
100 30. 3
è ÷
100 ø çè ÷
100 ø
840.00
= 5000(1.2155 – 1.21) = Rs. 27.50.
Interest earned on second part
2
æ 10 ÷ö
= 3500 ççç1 + ÷÷ - 3500
è 100 ø

11 11
= 3500 ´ ´ - 3500
10 10
= 4235 – 3500 = Rs. 735
Difference = Rs. (840 – 735) = Rs. 105

Page 4 QA - 02
QA - 03 : Percentage - 3 CEX-Q-0204/20
Answers and Explanations

1 3 2 4 3 4 4 2 5 4 6 2 7 3 8 4 9 1 10 2
11 2 12 – 13 1 14 – 15 – 16 – 17 – 18 1 19 1 20 4
21 4 22 3 23 4 24 4 25 4 26 1 27 2 28 2 29 4 30 3

1. 3 Amount at the end of the 1st year after repayment 4. 2 If CP = Rs. 100, MP = Rs. 120.
= 6000 × 1.05 – 1200 = Rs. 5,100.
æ1 1 1 ö
Therefore, amount at the beginning of the 2nd year Average SP = Rs. ççç ´108 + ´120 + ´ 90÷÷÷
will be Rs. 5,100. è4 2 4 ø

Amount at the end of the 2nd year after repayment = Rs. (27 + 60 + 22.5) = 109.5
= 5,100 × 1.05 – 1200 = Rs. 4,155. Hence, profit percentage

109.5 − 100
Therefore, amount at the beginning of the 3rd year will = × 100 = Rs. 9.5.
be Rs. 4,155. 100

Alternate method: 5. 4 Let the cost of 1 apple be ‘a’, cost of 1 mango be ‘m’
Amount at and the cost of 1 orange be ‘o’
Amount after
Year the beginning Interest Repayment Therefore, (2a + 3m + 4o) – (a + 2m + 2o) = 6 ...(i)
repayment
of the year And (3a + 3m + 5o) – (a + 2m + o) = 8 ...(ii)
600 × 5 ×1 6000 + 300 – 1200
1st R
Rs.6,000 = Rs.300 Rs. 1,200 2 × (ii) – (i) : 3a + m + 6o = 10.
100 = Rs. 5,100
5100 × 5 × 1 5100 + 255 – 1200
To make a profit of 5% one should sell 3 apples,
2nd Rs.5,100 = Rs. 255 Rs.1, 200
100 = Rs. 4,155 1 mango and 6 oranges at Rs. 1.05 × 10 = Rs. 10.5.
3rd Rs.4,155
(12 - 10.5)
Percent discount = × 100 = 12.5%.
12
2. 4 Let the value of each instalment be x.
x x x x 6. 2 Suppose he invests Rs. 150 in each of the lots.
∴ 1.02 + + + = 33000
(1.02)2 (1.02)3 (1.02)4 So total investment = Rs. 300.
⇒ x = 8666.6. He will get 5 dozens of first lot and 3 dozens of second
lot. In all he will have 8 dozens, selling price of which
3. 4 Suppose Rehman buys (LCM of 15, 20 and 35) will be 40 × 8 = Rs. 320
= 420 apples.
Total cost of apples bought at 15 for a rupee 320 - 300 20
∴ Profit percentage = ´100 = » 6.67%.
300 3
420
= = Rs. 28
15
Total cost of apples bought at 20 for a rupee 7. 3 Let the list price of the book be Rs. 100. The booksellers
cost price (CP) is Rs. 58. He earns 20% profit on his
420 selling price (SP)
= = Rs. 21
20 0.2 SP = (SP – CP)
∴ Total C.P = Rs. (28 + 21) = Rs. 49
CP 58
840 ´ 2 SP = = = 72.5
S.P for (420 + 420) 840 apples = Rs. = Rs. 48 0.8 0.8
35
Hence, maximum discount that can be offered
49 - 48 = (100 – 72.5) = 27.5%
∴ Loss % = × 100 = 2.04%.
49

QA - 03 Page 1
8. 4 Let the cost price of the washing machine be Rs. 100 15. Let the man buy Rs. z shares.
and list price be Rs. x. z(100 – x) = 4500 and (z – 10) (100 + x) = 6250.
Solving, we get z = 60.
20
Then, x – x × = 112
100 16. The selling price of the item = 850 × 0.8 = Rs. 680.00
⇒ 0.8x = 112 ⇒ x = Rs. 140
680 - 500 180
When the shopkeeper gives a discount of 25%, then ∴ Profit percentage = ´100 = = 36%.
selling price 500 5

25
= 140 – 140 × = 140 – 35 = Rs. 105 17. Let the CP of each T-shirt be Rs. x
100 CP of the (3 + 1) T-shirts to the shop = 4x
Hence, there will be a profit of 5%. SP of the 3 T-shirts = 1.6 × 3x = 4.8x
0.8x
9. 1 Loss = CP – SP. Profit percentage = ´100 = 20%.
1 dozen SP = 6 dozen CP – 6 dozen SP 4x
7 dozen SP = 6 dozen CP
1 18. 1 Let the weight of 'p' apples be equal to 1 kg and the
Loss% = × 100 ≈ 14.28. weight of 'q' oranges be equal to 1 kg.
7
Let the cost price and selling price of 1 apple be 'C1'
10. 2 SP of television = 3200 × (1 – 0.25) × (1 – 0.15) and 'S1' respectively.
= Rs. 2040. Let the cost price and selling price of 1 orange be 'C2'
and 'S2' respectively.
11. 2 Total selling price = Rs. 24,000 and in this type of Therefore,
transaction there is always loss. 10 × C2 = p × C1 ....(i)
q × C2 = 12 × C1 ....(ii)
2
 20  15 × S2 = p × S1 ....(iii)
Therefore, loss percentage =   = 4%
 100  Let the selling price of 1 kg of oranges be equal to that
of 'x' apples.
24000 × 100 Therefore, q × S2 = x × S1 ....(iv)
Now, total cost price = = Rs.25,000
96 10 p
Loss = 25000 – 24000 = Rs. 1,000. Dividing the equation (i) by (ii), we get that =
q 12

12. Let the CP be Rs. x ⇒ p × q = 120


∴ SP = Rs. 1.2 x 15 p
New CP = Rs. 0.8x Dividing the equation (iii) by (iv), we get that =
q x
New SP = Rs. (1.2x – 5)
1.25 (0.8x) = 1.2x – 5 Therefore, x = 8.
x = 1.2x – 5
∴ x = 25. 19. 1 Let 'x' and 'y' be the cost price of 1 pen and 1 pencil
respectively.
13. 1 Let CP of one article = Rs. 1 Let the selling price of 1 pen be 'z'.
CP of 5 articles = Rs. 5. Cost price of 3 pencils and 5 pens = 5x + 3y
SP of 5 articles = CP of 4 articles = Rs. 4. Therefore, 5z = 1.2(5x + 3y) or z = 0.24(5x + 3y).
Hence SP < CP Also, 2z = 0.75(2x + 6y)
Loss = 5 – 4 = 1 or, 2(0.24(5x + 3y)) = 0.75(2x + 6y)
or, 5x = 17y.
1 Selling price of 6 pens = 6z = 1.44 (5x + 3y) = 1.44
Loss % = × 100 = 20%.
5 (20y) = 28.8y
Cost price of 6 pens and 4 pencils = 6x + 4y = 24.4y
14. Suppose the initial cost price be Rs. x and the selling
 (28.8y − 24.4y) 
price be Rs. y. Initial profit = Rs. y – x Profit percent =   × 100 = 18.03%.
After the increase in manufacturing, cost price  24.4y 
= Rs. 1.2x
New profit = Rs. (y – 1.2x) 20. 4 Let x kg be the quantity sold at 17% profit, and the
∴ (y – x) – (y – 1.2x) = 20 price of sugar be Rs. 1/kg
⇒ y – x – y + 1.2x = 20 ∴ 1.17x + (100 – x)1.07 = 110
⇒ 0.2x = 20 ⇒ 0.10x = 3
⇒ x = 100. ⇒ x = 30.

Page 2 QA - 03
Let the S.P. of both the items be Rs. 100x each.
 20 
21. 4 Piano is sold for Rs. 325 − 325 ×  = Rs.260 af-
 100  Item SP % Pr CP
ter giving 20% discount. 1 100x –10 111.11x
After additional discount, it was sold for Rs. 234.
2 100x 20 83.33x
260 − 234
So, second discount percentage = × 100
260 Total 200x – 194.44x

26 ∴ Overall percentage profit


× 100 = 10% .
260 (200 − 194.44 ) x × 100
= = 2.86%
194.44x
22. 3 Let the CP of 1000 g be Rs 100.
⇒ CP of 850 g = Rs.85 26. 1 Let C.P. of the book be Rs. x
SP of 850 gm when he sells at 5% loss Since loss = 20%
= 100 – 5 = Rs. 95 S.P. = Rs. 0.8x
If S.P. = Rs. (0.8x + 3), profit = 30%.
100 200
Percentage profit (m) = (95 – 85) × =
100
85 17 Hence, = Rs. (0.8x + 3)
SP of 850 g when he sells at 5% gain 30
=100 + 5 =Rs. 105
100
100 400 ⇒x= (0.8x + 3) ⇒ x = 6
Percentage profit, n = (105 – 85) × = 130
85 17
6.60 − 6
600 ∴ Required profit percentage = × 100 = 10%.
⇒m +n= . 6
17

23. 4 Let us look at the two equations. Let (5 pens + 7 27. 2 Total cost = 9x + 5x + 11x = 25x
pencils + 4 erasers) cost Rs. x, and (6 pens + 14
1
pencils + 8 erasers) will cost Rs. 1.5x. 20% of total cost = 20% of 25x = × 25x = 5x
In the second case, had Rajan decided to buy 10 pens 5
instead of 6, it would have cost him Rs. 2x. Manpower cost : Profit = 5x : 5x = 1 : 1
∴ (10 pens + 14 pencils + 8 erasers) = Rs. 2x.
Now, subtracting the second equation from the third,
200
we get 4 pens cost Rs. 0.5x. 28. 2 Trader’s net profit = × 100 = 25% .
So 5 pens will cost Rs. 0.625 x. 800
This is the amount that I have spent on pens.
Hence, fraction of the total amount paid 29. 4 Let’s say the shopkeeper buys at Re. 1 per gram.
= 0.625 = 62.5%. He buys 1000 gm, but pays for 1,100 g.
He now mark-up his CP by 10%, hence he charges at
the rate of Rs. 1.1 per gram.
(100 − 20 − 1.5x )
24. 4 Revenue in 1 hr = 100 (50 + x ) Hence, he makes a revenue of Rs. 1,100.
100 So, he neither makes a profit nor a loss.

(100 + 2x )
Cost = 40 (50 + x ) 30. 3 Let the actual value be 100x.
100 After he used the counterfeit weight, the value
Profit = 2000 – 2.3x2 – 75x. becomes 80% of the actual value or 80x.
If profit > 0, x < 17.38 Also, after adding 20% impurities to the value now, it
⇒ x = 17. becomes 120% of 80x = 96x.
And because of faulty balance it becomes 1.25 × 96x
25. 4 He must have suffered only 10% loss in one item and = 120x
earned 20% profit on the other item. 120x − 80x
Otherwise he cannot make an overall profit. So, the profit = × 100 = 50%.
80x

QA - 03 Page 3
QA - 04 : Ratio - 1 CEX-Q-0205/20
Answers and Explanations

1 1 2 2 3 3 4 1 5 1 6 – 7 3 8 2 9 3 10 1
11 1 12 3 13 – 14 2 15 – 16 – 17 – 18 1 19 3 20 2
21 1 22 4 23 2 24 3 25 3 26 1 27 2 28 1 29 3 30 1

a 1 b 2
= = a b c 2a − 3b + 5c
1. 1 4. 1 Let = = = =λ
b 3 c 1 2 3 4 k
⇒ a:b:c=2:6:3
Similarly a : b : c : d : e : f = 2 : 6 : 3 : 6 : 2 : 8 ∴ a = 2λ, b = 3λ, c = 4λ

abc 2×6×3 3 2a − 3b + 5c
∴ = = 3 =λ
def 6×2×8 8 k
Hence option (1). 2 × 2λ − 3 × 3λ + 5 × 4λ
⇒ =λ
k
Alternate Solution:
15λ
a c b abc ⇒ = λ ⇒ k = 15.
We have to find the value of × × = k
f d e def

c 1 a 5. 1 Let income of X be Rs. p and income of Y be Rs. 3p


Now, = and we will get by multiplying all the
d 2 f Let expenses of X be Rs. 19q and expenses of Y
be Rs. 40q
a 1 Hence, savings of X = Rs. (p – 19q) and savings of Y
ratios. So, = .
f 4 = Rs. (3p – 40q)
Total savings = (4p – 59q) = 36020
b c d b
Similarly, × × = = 3. Difference of savings = (2p – 21q) = 18860
f d e e Solving the above equations, we get p = 10480, Thus,
income of X and Y is Rs. 10,480 and Rs. 31, 440
abc 1 1 3
So, = × ×3= . respectively.
def 4 2 8
6. Let the number of one rupee coins, 50 paise coins and
2. 2 Let a – b = x, b – c = 2x and c – d = 3x 25 paise coins be 3x, 4x and 12x respectively.
∴ c = 3x + d
4x
b = 2x + c = 5x + d Value of one rupee coins = 3x, 50 paise coins =
2
a = x + b = 6x + d
a + d 6x + d + d 2 12x
∴ = = . and 25 paise coins =
c 3x + d 1 4
Thus, 3x + 2x + 3x = 600 ⇒ x = 75

3. 3
a b b c c d
= , = , = 12 × 75
2 3 3 5 5 6 Therefore, value of 25 paise coins =
4
a b c d = Rs. 225
∴ = = = = k (say)
2 3 5 6
7. 3 Ratio of amounts collected from first and second
3a + 2b + 4c + d 3(2k) + 2(3k) + 4(5k) + (6k) classes = (3 × 1) : (1 × 27) = 1 : 9
∴ = So, amount collected from first class passenger is
3d + 4c 3(6k) + 4(5k)
1
38 = Rs. × 2700 = Rs. 270.
= = 1. 10
38

QA - 04 Page 1
8. 2 Let present ages (in years) of mother and daughter Hence, the respective share of A and B from the amount
be 9x and 4x respectively. of C is Rs. 70 and 10 respectively.
9x − 10 7
∴ = 15. Let the sum of ages of the remaining students be S38.
4x − 10 2
⇒x=5 S38 + 14 + a
∴ = 18 … (i)
Hence, present ages of mother and daughter are 40
45 years and 20 years respectively.
S38
and = 18 … (ii)
Alternate Solution: 38
Pick the options and check. So, answer is option (2). Solving (i) and (ii), a = 22 years. (Where ‘a’ is the age
of 2nd student).
9. 3 On the basis of the given information, we can conclude
that ratio of present ages of Chinta, Mani and Suknaya 16. Let his marks be 5x, 4x, 6x, 8x and 7x respectively.
is 14 : 19 : 10. Aggregate marks obtained = 30x = 50% of aggregate
Let the ages (in years) of Chinta, Mani and Sukanya maximum marks. Thus, aggregate maximum marks
be ‘14x’, ‘19x’ and ‘10x’ respectively. = 60x.
⇒ Maximum marks per subject = 12x
∴ 14x + 19x + 10x = 43 ⇒ x = 3 55% of maximum marks per subject = 6.6x. Hence in
3 two subjects, marks are more than 6.6x
∴ Age of Mani = 19x = 19 × 3 = 57 years.
17. 1st 2nd 3rd 4th sum
10. 1 Let the numbr of chocolate with Sheela, Munni and
Chameli be ‘s’, ‘m’ and ‘c’ respectively. 2 3
s = c + 5 and s = m + 3 5 6
⇒m–c=2
8 9
3c 80 120 144 162 506
⇒ −c = 2 [ 3 m : c = 3 : 2]
2
The above total = 506. For the sum of 253 the numbers
⇒c=4 are 40, 60, 72 and 81. Average of the 2nd and 3rd is
∴ s = 9 and m = 6 66.
∴ m + c + s = 6 + 4 + 9 = 19.
Alternate solution:
11. 1 The ratio = (4 × 1.1) : (3 × 1.2) = 11 : 9. Ratio of four numbers = 1st : 2nd : 3rd : 4th = 40 : 60 :
72 : 81
12. 3 Let the quantity of pure copper which is melted along Therefore, sum = 40 + 60 + 72 + 81 = 253.
with the two alloys be x kg. Hence, average of 2nd and 3rd number is 66.
8+4+x 3
∴ = 18. 1 Let a, b and c be the cost-prices of three brands of
2 + 12 2
juices per litre
⇒x=9
Hence, the weight of new alloy = 10 + 16 + 9 = 35 kg. 3a + 4b + 5c
Therefore, = 20
12
13. Let the present ages (in years) of the father and his ⇒ 3a + 4b + 5c = 240
son be 9x and 4x respectively.
Age of the father after 8 years = (9x + 8) 4a + 5b + 6c
and = 25
Age of the son 2 years ago = 4x – 2 15
9x + 8 8 ⇒ 4a + 5b + 6c = 375
∴ =
4x − 2 3
⇒ 27x + 24 = 32x – 16 (6a + 7b + 8c)

⇒x=8 21
∴ Present age of the son = 4x = 32 years.
3(4a + 5b + 6c) – 2(3a + 4b + 5c)
=
21
8
14. 2 Each person receives = = 2.67 apples 3(375) – 2(240)
3 = = Rs. 30.7.
The ratio of the amount received by A and B from C
21

5 − 2.67
= = 7 : 1.
3 − 2.67

Page 2 QA - 04
19. 3 Let initial value be v1. 23. 2 The shortest method of arrive at the answer is to
Here v1 = k(w)2, where k is a proportionality constant, verify the answer choices. We find tht choice (2) fits
and w is the weight. in with the data.
According to the question, w = 4 ⇒ v1 = 16 k However, a theoretical solution is given below.
Suppose the diamond breaks into two pieces of Let the number of boxes seller had be x.
weights x kg and (4 – x) kg, and let their values be v2
x 1
and v3 respectively. First man buys + boxes.
Hence, v2 = kx2 and v3 = k(4 – x)2 = k(x2 + 16 – 8x) 2 2
Also, (v2 + v3) = (62.5)% of v1
1   x 1  1
Second man buys = x− + +
5 2   2 2   2
⇒ 2k(x – 4x + 8) = × 16k
2
8
⇒ x = 1 or 3 1  x 1 1 x 1
= − + = +
Therefore, weight of the parts are 1 kg and 3 kg. 2  2 2  2 4 4

x 1 x 1
20. 2 Let the weights (in kg) of the father, the son and the
Total number of boxes sold = + + + = 3x + 3
mother be x, y and z respectively. 2 2 4 4 4 4
According to the question,
x : y = 3 : 2 and y : z = 5 : 6 3x 3
= +
Therefore, x : y : z = 15 : 10 : 12 4 4
Weight of the father = 2 × 35 = 70 kg
3x 3 x 3
12 ⇒ x= + ⇒ = ∴ x = 3.
Weight of the mother = × 70 = 56 kg. 4 4 4 4
15
24. 3 Let the number of students in Batch A in January be
21. 1 Let the incomes (in Rs.) of Aishwarya, Babita and 2x.
Charu be 7x, 9x and 12x and their expenditures Let number of students in Batch A in February be 5y.
(in Rs.) be 8y, 9y and 15y respectively. Then, Total numbers of students in Batch A and Batch B
7x combined in January and February are 5x and 13y
7x – 8y = respectively.
4 Therefore 13y : 5x = 26 : 5 or y : x = 2 : 1 or y = 2x.
21 From here, we can compile the following table:
⇒ x = 8y
4
Batch A Batch B
32 Number of students in 2x 3x
⇒x= y
21 January
Number of students in 10x 16x
32 96
So their respective incomes (in Rs.) are y, y February
3 7
Rate of increase from 400% (1300/3)%
128 Jan to Feb
and y and respective savings (in Rs.) will be
7 Rate of increase from 800% 1300%
Feb to March
8 33 23
y, y and y. Number of students in 90x 224x
3 7 7
March
Hence, the required ratio is 56 : 99 : 69.
Required Ratio = 45 : 112.
22. 4 Assume the last year Harsha’s salary and Mandar’s
salary were 3x and 5x respectively. 25. 3 Suppose the person buys A apples and M mangoes
Then their salaries in this year are 4.5 x and 6.25 x. and cost price of an apple is Rs. x.
So, the ratio of their salaries in this year is Therefore, cost price of a mango will be 2x.
4.5 x : 6.25 x, i.e. 18 : 25 respectively. Total cost price = Ax + 2Mx.
∴ Present salary of Harsha Now selling price of an apple is 2x.
∴ SP of a mango will be 6x.
18 Total SP = 2Ax + 6Mx.
= 86000 × = Rs. 36,000.
Now we have
43
5 M 1
2Ax + 6Mx = (Ax + 2Mx) or = .
2 A 2

QA - 04 Page 3
26. 1 Let x be the fixed cost and y the variable cost. Then,
1
17500 = x + 25y … (i) 29. 3 P ∝ T and P ∝ , combining
30000 = x + 50y … (ii) V
Solving the equation (i) and (ii), we get
T T
x = 5000, y = 500 P∝ or P = K
Now if the average expense of 100 boarders be ‘A’. V V
Then PV
100 × A = 5000 + 500 × 100 Hence, =K
V
⇒ A = Rs. 550.
P1V1 P2 V2
or = =K
1 T1 T2
27. 2 A ∝ B; B ∝ ; C ∝ D2; D ∝ E1/3
C
150 × 20 150 × 8
or =
k k k 100 T2
∴ A = k1B = 2 = 32 = 24/ 3
C D E ⇒ T2 = 40° kelvin
So as A increases, E decreases.
30. 1 Let the initial capacity of the can be x.
28. 1 Height (H) is proportional to the square root of age (A) After the process,
i.e. H = K A , where K = Proportionality constatn H 7
(x − 9)
= 4ft at A = 9 years 12 7
ratio of A to total mixture = =
x 16
4
∴ K=
3 7  9 7
⇒ 1−  =
12  x  16
4 1
So when A = 16 years, H = 16 = 5 ft or 5 ft 4
3 3 9 3
⇒ 1−=
inches. x 4
⇒ x = 36 L
7
∴ A= × 36 = 21L
12

Page 4 QA - 04
QA - 05 : Ratio - 2 CEX-Q-0206/20
Answers and Explanations

1 2 2 – 3 4 4 4 5 3 6 2 7 2 8 – 9 2 10 –
11 2 12 – 13 3 14 – 15 2 16 1 17 3 18 1 19 3 20 2
21 2 22 3 23 2 24 1 25 1 26 4 27 3 28 4 29 3 30 2

1. 2 Let x be the initial average age of persons and y be 6. 2 Suppose the number of boys be S excluding a 100 kg
the age of new persons. Then, boy.

23x − 20 + y 4 S + 100 1 S + New


=x+ Then + =
23 12 40 4 40
⇒ New boy’s weight = 110 kg.
23
⇒ 23x – 20 + y = 23x +
3 7. 2 Let the number of students in classes X, Y and Z be a,
b and c respectively. Then
23 83 Total of X = 83a
⇒y= + 20 = = 27 years 8 months
3 3 Total of Y = 76b
Total of Z = 85c

2. The change in average age of the group And 83a + 76b = 79, i.e. 4a = 3b
= 58 – 56 = 2 years. a+b
Actual change in years = 30 years.
Also 76b + 85c = 81, i.e. 4c = 5b
30 b+c
Therefore, Total number of members = = 15
2
4 5 5 4 5
Hence, b = a,c= b = × a = a
3 4 4 3 3
3. 4 Average got reduced by 2 cm. So, overall decrease
of 26 cm for 13 people.
83a + 76b + 85c
Height of new person Average of X, Y and Z =
a+b+c
= 184 – 13 × 2 = 184 – 26 = 158 cm.
4 5
4. 4 Sum of marks for 24 students = 24 × 89 = 2136 83a + 76 × a + 85 × a
= 3 3 = 978 = 81.5
Average marks of a student cannot increase beyond 4 5 12
a+ a+ a
100. So, total marks for 21 students cannot exceed 3 3
2100.
So, maximum increase in average = 100 – 89 = 11.
8. Let s = sum of the ages of all students in the class.
x = sum of the ages of 2 students who left the class.
5. 3 60 is wrong because then to arrive at a total of 121,
y = sum of the ages of 4 students who joined the
the other box will have to weight 61 kg which will be class.
obviously inot the highest. 64 is wrong too, because We have three equations.
then to add up to 121, the other weight will have to be
57 and to make up to a total of 120, the next box shall s s−x+y
(i) =A (ii) =A
have a weight 63 which obviously makes the maximum n n+2
possible total as 64 + 63 = 127. 62 is the correct (iii) x + y = 72
answer because the other boxes shall be 59, 54, 58, From (ii) & (iii) we get x = y = 36 years but we cannot
56. These will give all the totals given above. calculate the value of A, from the given information.

QA - 05 Page 1
9. 2 Total rainfall for the week = 3.7 × 7 = 25.9 cm ...(i) 14. Let x be the next test score. Then,
Rainfall for Monday, Tuesday and Wednesday = 4.3 ×
78 + 92 + 83 + 99 + x
3 = 12.9 cm ...(ii) The average = = 90
5
Rainfall for Friday, Saturday and Sunday = 3.9 × 3
= 11.7 cm ...(iii)
352 + x
Rainfall for Thursday = Equation (i) – Equation (ii) – ⇒ = 90 ⇒ x = 98.
5
Equation (iii) = 25.9– 12.9 – 11.7 = 1.3 cm.

10. Let us assume that the average age of 5 members 3 15. 2 Let the number of boys and girls in the class be ‘b’ and
years ago be M years. ‘g’ respectively and the average age (in years) of
Total age of 5 members 3 years ago = 5M boys and girls be ‘x’ and ‘y’ respectively.
Let the age of the old member and new member be O
b 7g
years and N years respectively. Given, b + g = 60 and + = 60
2 4
Present age of these 5 members = 5M + 15
⇒ b = 36 and g = 24
(5M + 15 − O + N) Now, 36x + 24y = 720 and 18x + 42y = 810
⇒ New Average age (M) =
5 ⇒ x = 10 and y = 15
⇒ N – O = 5M – 5M –15 Hence, the average age of the boys is 10 years.
⇒ O – N = 15 years
16. 1 Let the initial strength of the students in the institute be
11. 2 Average speed of the train x. Then,
Total age of x students = 15x
63 × 5 + 58 × 3 + 72 × 2 633 According to the question,
= = = 63.3 km / hr.
10 10 15x + (10 × 12) = 14.8(x + 10)
⇒ 15x + 120 = 14.8x + 148
12. Ten years ago, the total age of all the eight people in ⇒ 0.2x = 28
the family = 231
28 × 10
As per the information given in the question, the total ⇒ x= = 140.
2
age of all the people in the family
= 231 + 3 × 8 – 60 + 0 = 195 17. 3 Let 3x and 5x be the salaries of A and B respectively.
Similarly, the total age of the people 7 years ago in the ⇒ 9x + 750 = 10x + 500 ⇒ x = 250
family four years ago = 195 + 3 × 8 – 60 + 0 = 159. Salary of A = 250 × 3 = Rs. 750 and B = 250 × 5
Therefore, the current average age of all the people in = Rs. 1, 250.

159 + 32 Alternate Method:


the family = = 24 years.
8 Pick the options and check the conditions.

13. 3 Let, the number of candidates who passed in the 18. 1 Let the salaries (in Rs. lakh) of 5 VPs (Vice presidents)
examination be ‘p’ and number of candidates who be a, b, c, d and e respectively in ascending order.
failed in the examination be ‘f’. Median salary is Rs. 5 lakh i.e. c = 5

60p + 25f
∴ 43 < ≤ 49 As, mode of the salaries is Rs. 8 lakh, it implies that at
p+f least two VPs are getting salaries of Rs. 8 lakh. Also,
salaries of only two VPs are greater than Rs. 5 lakh.
60p + 25f
⇒ 43 − 25 < − 25 ≤ 49 − 25 Hence, d = e = 8
p+f Given mean of the salaries is Rs. 5 lakh.

18 p 24 a+b+c +d+e
⇒ < ≤ ∴ =5
35 f + p 35 5

Note: For positive numbers a and b if a > b then


a+b+5+8+8
⇒ =5 ⇒a+b= 4
1 1 5
< .
a b

Page 2 QA - 05
⇒ a = 2 and b = 2 or a = 1 and b = 3 (As salaries are 22. 3 A : B : C = 2000 : 5000 : 4000 = 2 : 5 : 4
in integer lakh). 2
Profit share of A = Rs. × 1210 = Rs. 220
But if a = 2 and b = 2, then Rs. 8 lakh will not be the only 11
mode.
Therefore, a = 1 lakh and b = 3 lakh 5
Profit share of B = Rs. × 1210 = Rs. 550
Hence, required sum = 8 + 1 = Rs. 9lakh. 11

19. 3 Marks obtained should be less than 50 in each paper. 4


Profit share of C = Rs. × 1210 = Rs. 440
There are three cases possible. 11
(a) Marks are 8, 9, 10, 13, 14, 15.
(b) Marks are 16, 18, 20, 26, 28, 30. 23. 2 Profit received by Z = Rs. {1500 – (450 + 550)}
(c) Marks are 24, 27, 30, 39, 42, 45. = Rs. 500
Ratio in capital = X : Y : Z = 450 : 550 : 500 = 9 : 11 : 10
If we take case (a), then he required 81 marks more to
get 50% of the total. 10
∴ Z’s capital = Rs. × 3300 = Rs. 1,100
If we take case (c), then he got more than 50% marks. 30
IN case (b) he required 12 marks more to get 50%
marks, which is less than the case (a). So this is the 24. 1 Let n be 4. Then consecutive odd numbers beginning
right answer. with x will be:
x, x + 2, x + 4, x + 6
20. 2 Let first number be x. n consecutive odd number beginning with x + 4 will
Then, 7th number will be x + 6, and 14th number will be:
be x + 13. x + 4, x + 6, x + 8, x + 10
Average of first seven numbers Average of first group of numbers, A = (x + 3)
Average of second group of numbers, B = (x + 7)
x + x + 1+ x + 2 + x + 3 + x + 4 + x + 5 + x + 6 Therefore (B – A) = 4
= K=
7 Similarly, if n is odd, we can deduce that difference
still remains 4. The difference will not depend upon n
7x + 21 or x.
⇒K = = x+3
7
25. 1 Ratio of investments is
Average of next seven numbers
1 1 1
x + 7 + x + 8 + x + 9 + x + 10 + x + 11 + x + 12 + x + 13 ⇒A:B:C= : : =6:4:3
= 2 3 4
7
Ratio of time period is
7x + 70 1 1
= = x + 10 = K + 7. ⇒ tA : tB : tC = 1: : =4:2:1
7 2 4
∴ Ratio of profit shared
⇒ AtA : BtB : CtC = 24 : 8 : 3
40
21. 2 Profit = Rs. 8,0000 × = Rs. 32,000 1
100 3 B’s share is of A’s share
3
Ratio of shares of A, B and C in the profit = Ratio of
amount invested 1
∴ B’s share = × 30,000 = Rs. 10,000
3
6
B’s share in the profit = Rs. 32000 × = Rs. 12,000
16
26. 4 Ratio of profit sharing = 4500 × 12 : 5400 × n
where n is the period for which B invested.
7
C’s share in the profit = Rs. 32000 × = Rs. 14,000 3 Ratio = 4500 × 12 : 5400 × n = 10 : n
16
Now, this ratio is equal to 2 : 1.
Sum of the shares of B and C in the profit ∴ 10 : n = 2 : 1
= Rs. 12,000 + Rs. 14,000 = Rs. 26,000 ⇒ n = 5 months
Thus, B joined A after 12 – 5 = 7 months

QA - 05 Page 3
27. 3 Let D’s share be 27x. 30. 2 In terms of Rupee-Month for the entire year,
Then, C’s share = 9x, B’s share (i) Abhishek’s Investment
= 3x and A’s share = x = (100 × 3) + (75 × 3) + (50 × 6) Lakhs R-month
Then, x + 3x + 9x + 27x = 4,00,000
= 825 lakhs R-month
⇒ x = 10,000
(ii) Rani’s Investment
Hence, C’s share = 9x = Rs. 90,000.
= (200 × 3) + (150 × 3) + (100 × 6)
= 1650 Lakhs R month
28. 4 The ratio in which profit will be shared among Ajay,
Boman and Chirag (iii) Susmita’s Investment
= 28000 × 12 : 40000 × 7 : 560000 × 5 = 6 : 5 : 5 = 50 × 6 = 300 Lakhs R-month
5 (iv) Randeep’s Investment = 100 × 3 = 300 Lakhs
Hence, share of Boman = × 32000 = Rs. 10,000. R-month
16
∴ Their ratio of Investment = 11 : 22 : 4 : 4
40 ∴ Abhishek and Rani’s combined share of profit
29. 3 Profit = Rs. 80000 × = Rs. 32,000
100
Ratio of shares of A, B and C in the profit = Ratio of  11 + 22 
=  × 41 = 33 lakhs.
amount invested  41 
6
B’s share in the profit = Rs. 32000 × = Rs. 12,000
16
7
C’s share in the profit = Rs. 32000 × = Rs. 14,000
16
∴ Sum of the shares of B and C in the profit
= Rs. 12,000 + Rs. 14,000 = Rs. 26,000.

Page 4 QA - 05
QA - 06 : Ratio - 3 CEX-Q-0207/20
Answers and Explanations

1 – 2 2 3 1 4 1 5 3 6 1 7 – 8 3 9 – 10 1
11 3 12 1 13 4 14 2 15 4 16 2 17 3 18 4 19 2 20 1
21 4 22 – 23 1 24 – 25 4 26 4 27 1 28 2 29 1 30 3

Individual content in the mixture:


80
1. In 20 L solution, wine = 20 ×
100 8
Milk = × 75 = 40 L
= 16 L and water = 4 L 15
Let x L water be added. Then, Water = 75 – 40 = 35 L
5L of water was added afterwards
4 + x 60
⇒ = Now, the new ratio of milk and water is:
16 40 40 : 35 + 5 = 40 : 40 = 1 : 1
⇒ x = 20 L
Alternate Method:
2. 2 30 L of mixture is drawn out. Total volume = 105 L
8
2 Total amount of Milk = × 105 = 56 L
∴ In 90 L of mixture, milk = 90 × = 36 L and water 15
5
7
3 and total volume of water = × 105 = 49 L
= 90 × = 54 L 15
5 Now, when 14 L of water is removed, i.e., 16 L of milk
Now, milk = 36 + 18 = 54 L and water = 54 + 12 = 66 L is removed.
∴ New ratio of milk and water = 54 : 66 = 9 : 11 So, new ratio is 40 : 49 – 14 + 5 = 40 : 40 = 1 : 1.

3. 1 By alligation, 6. 1
30 % 60 % 10 0 I 75 I 75 I + 25 IW
50 %
M W M W M W
10 20 I II
∴ Required ratio = 10 : 20 i.e. 1 : 2. 80 20 60 15 60 15 +25

4. 1 CP = 10(at 10% profit)


Using alligation, The diagram is self explanatory. Removal of 25 litres
at stage I will result in volume of milk being reduced by
3 10 − (x − 2)
⇒ = 80% of 25 lit i.e. 20 lit and volume of water being
2 x − 10 reduced by the remaining 5 lit. So M = 60 lit and W = 15
⇒ x = 10.8 lit. Addition of 25 lit water will finally given M = 60 lit and
W = 40 M. Hence the ratio of W and M = 40 : 60 = 2 : 3.
Q1 Cm − C 2
=
Q2 C1 − Cm
7. Let initial capacity of the can be x.
7
5. 3 Total volume = 105 L (x − 9)
12 7
Milk : Water = 8 : 7 After the process, proportion of A = =
14 L of water was removed x 16

15 7  9 7 9 3
∴ Amount of mixture removed = ⇒ 1−  = ⇒1– ⇒ x = 36 L
12 
× 14 = 30 L =
7 x 16 x 4
Volume of mixture left = 105 – 30 = 75 L
7
∴A= × 36 = 21 L
12

QA - 06 Page 1
8. 3 Use Alligation Method:
x + 15 90
From B : = ⇒ x – 9y = – 15
G old C opp er x + y + 15 100
18 8
x+9 84
15 From C : = ⇒ 4x – 21y = – 15
x + y + 10 100
7 3 ∴ x = 12, y = 3
Thus the required ratio = 3 : 7. ∴ Percentage of sliver In A
x 12
9. Let I be the initial quantity of diluted milk in the can and = × 100 = × 100 = 80% .
x+y 15
the initial concentration of milk be m%.
ml
∴ = 30 13. 4 Let the quantity of each alloy be 1.
(l + 35)
7
Copper in first alloy =
ml 9
Also = 20
(l + 75)
7
⇒ 2(I + 75) = 3(I + 35) ⇒ I = 45 and mI = 2400. Copper in second alloy =
18
If w litres of water is required to be added to the can
to make the concentration of milk in the can 8% then 7 7 14 + 7 21
Copper in third alloy + = =
ml 2400 9 18 18 18
= 8 ⇒ 120 + w = = 300 ⇒ w = 180.
(l + 75 + w) 8 Total quantity of third alloy = 2.
21 15
Aluminium = 2 – =
10. 1 18 18
Sample I Sample II
Copper and aluminium in the third alloy = 21 : 15 = 7 : 5.
Wt 6 kg 12 kg
% of Iron x% y%
14. 2 Let the total quantity of alloy be x.
Pieces w and (6 − w) w and (12 − w)
82% of x 7 246x 126x
After Mixing (6 − w) + w (12 − w) + w ⇒
1424 3 { 1424 3 { 18% of x + 18
=
3 100
=
100
+ 126
x% y% y% x%

The percentage of iron in both the samples is the 120


⇒ x = 126,
same i.e. 100
(6 − w ) x + wy  (12 − w ) y + wx  10
= ⇒ x = 126 × = 105
6 12 12
⇒ x(12 – 3w) = y(12 – 3w) Quantity of new alloy = 105 + 18 = 123
as x ≠ y; (12 – 3w) must be zero
70
∴w=4 Copper = × 123 = 86.1
Hence, each cut off piece weighs 4 kg. 100
Zinc = 123 – 86.1 = 36.9
11. 3 Ratio of quantity of alcohol left to total quantity
15. 4 Let ‘x’ be the quantity of solution taken from beaker A
3 1 1
3 and ‘y’ be the quantity of solution taken from beaker B.
1 4 2 4
= × × × =
Quantity of methanol in beaker C =   +  
1 1 1 1 32 x 2y
6  8 
3 3
Alcohol percentage = × 100 = 9 %
32 8 (4x + 6y)
=
24
12. 1 Alloy A Alloy B Alloy C
 2x   5y 
10 × 90 Quantity of phenyl in beaker C =   +  
Silver : x x + 15 x+  6   8 
100
(8x + 15y)
10 × 10 =
Copper : y y y+ 24
100

Page 2 QA - 06
When a = 8, w1 = 40 and w2 + a = 60.
 3x   y 
Quantity of ethanol in beaker C =   +   Hence, ratio of acid to water coming out of
 6  8
a 8
pipe X = = = 2 : 13
(12x + 3y) w2 52
=
24
Required Ratio = (4x + 6y) : (8x + 15y) : (12x + 3y)
If x = y = 1, then required ratio = 10 : 23 : 15. so, option 1
19. 2 In first operation only L of alcohol will be poured
(1) is possible 2
 1  1 into the container A1.
If x =   and y =   then required tatio
4  15  1
In the second operation half of L will be poured in
= 7 : 15 : 16. 4
So, option (2) is possible. each of container A1 and A2.
 1  1 If the operations continue till infinite, the amount of
If x =   and y =   then required tatio alcohol in the container A1 will be
2 6
= 6 : 13 : 13. 1 1 1 1
= + + + + ...∞
So, option (3) is possible. 2 8 32 128

 1  1 1
If x =   and y =   then required tatio 2
8  15  2 =
1 3
= 9 : 20 : 17. 1−
So only option (4) is not possible. 4
In infinite operations, contents in A1 would be
16. 2 Statement I follows from the fact if volume ratio is 1 : 1
1 1 1
: 1 and CA, CB, CC are in AP then volume of milk in the + + + ...∞ = 1
container will also be in AP. 2 22 23
Statement II need not be true. For example, if the volumes
2
are such that the amount of milk is the same (i.e. in AP
with common difference = 0), then the volumes are ⇒ Concentration of alcohol = 3 × 100% ; 67%.
not equal. 1
Statement III would always be true if the volumes of VA
and VB are equal. 20. 1 For every litre of mixture sold Ramu received Rs. 2.5.
Therefore, the amount of Ramu’s share of milk in the
17. 3 Let a and b be the quantity and concentration of the
sugar solution in the vessel having the least quantity. 2.5 1
final mixture is =
20 8
ab (a + 5)b (a + 10)b
+ + = 5.4 i.e. Out of 8 litres of milk 1 litre was contributed by
100 100 100 Ramu and rest 7 litres was contributed by Kamu and
Also, b – a = 8 Namu.
Hence, the required quantities are 7 kg, 12 kg, 17 kg. Now, amount of milk contributed by
2
18. 4 In one hour, Kamu =   × 10 = 4 litres and
5
Let quantity of water from pipe Y = w1
Let quantity of water from pipe X = w2 and acid from  1
pipe X = a Namu =   × 15 = 3 litres
5
a 8 Between Kamu and Namu, they contributed 7 litres of
⇒ = ...(i)
w1 + w 2 + a 100 Milk and 18 litres of Water
But total mixture is of 5 + 10 + 15 = 30 litres
In second case, the bucket is not full.
Hence, Ramu had 1 litre of Milk and 4 litre of water
a 10 Required ratio = 1 : 4.
= … (ii)
w1 100
+ w2 + a
2
21. 4 In both jars concentration of milk is more than 50%.
Comparing (i) and (ii), Therefore, in jar three, concentration of milk can never
be 50%.

QA - 06 Page 3
22. Amount of alcohol that goes out in the 10% liquid taken
28. 2 A , 1/4
out is 20 ml. To this mixture, 100 ml of pure alcohol is
added. So total volume of alcohol in the new mixture X , 5/9
= 200 – 20 + 100 = 280 ml. B , 3/4
M
Therefore, percentage of alcohol in the new mixture B , 2/3
 280  Y, 4 /9
=   × 100 = 28%.
 1000  C , 1/3
Then, fraction of B in Product M
2
23. 1 Each day of wine is remaining.  5   3   4   2  77
3 =  ×  +  ×  =
3  9   4   9   3  108
2 8
So, at the end of third day   × 180L = × 180 In 864 units of Product M, amount of B
3 27
160  77 
= L is left = 864 ×   = 616
3  108 
In the final mixture 50% is B. Then amount of final
z z
 x  24  x  mixture = 616 × 2 = 1232
24. 54  1 −  = 24 or = 1 −  Water added = 1232 – 864 = 368.
 54  54  54 
Therefore, x = 18 L
n
 y
25. 4 Three successive decreases of 20% in the solution 29. 1 Use x  1 −  , where x is the original quantity, y is
 x
would mean that 51.2% of original amount of the
contents is left. the quantity taken out n times.
The ratio is 512 : 488. 2
 8 
∴ Milk left after two operations = 80  1 − 
2
 80 
26. 4  x − a  = 25 or a = x
 x 2a  = 64.8 L
 −  16 6
where x is taken as the capacity of the vessels.
30. 3 Let v be the initial volume of the cylindrical container.
27. 1 Let x be the initial amount of the mixture 4 1 9v
v – x = v ⇒ 9v = 28x or x =
5 7 4 28
∴ Amount of milk in litre = x and
8 Since by adding 35 L the level rises from a quarter to
3 a half, the volume of the vessel = 35 × 4 = 140 L.
water in litre = x
8
In 6 litre of mixture, 9 × 140
Therefore, x = = 45L
5 15 3 9 28
Milk = 6 × = litre and water = 6 × = litre
8 4 8 4
Alternate Method:
5 15 3 9
∴ x− = x− +6 We can also do this question by picking the options.
8 4 8 4
⇒ 2x = 60 ⇒ x = 30 litre
5
∴ Initial amount of milk = 30 × = 18.75 litre
8

Page 4 QA - 06
QA - 07 : TSD - 1 CEX-Q-0208/20
Answers and Explanations

1 3 2 2 3 1 4 3 5 3 6 2 7 2 8 3 9 1 10 3
11 2 12 2 13 3 14 2 15 1 16 2 17 3 18 1 19 1 20 1
21 – 22 3 23 – 24 – 25 3 26 3 27 – 28 2 29 2 30 –

1. 3 Let speed (in m/s) of a scooter, a car and a train be y, 5. 3 Let x be the distance between house and office and t
4y and 16y respectively. Let they all cover equal be the normal time taken.
distance of ‘d’ meters.
x 1
d d d =t+ … (i)
Time taken by each of the three is , and 5 12
y 4y 16y
respectively. x 1
∴ Required ratio of time taken is given as:
=t− … (ii)
6 12
d d d Subtracting (ii) from (i), we get
Scooter : Car : Train = : :
y 4y 16y  1 1 1
1 1 x −  =
= 1: : = 16 : 4 : 1. 5 6 6
4 16
 1  1
⇒ x  = ⇒ x = 5km.
2. 2 Let the usual speed of the man be ‘s’ m/min and usual  30  6
time be ‘t’ minutes.
4 5
When speed decreases to s, time taken will be t. 6. 2 Let the distance be x km.
5 4
(Since speed is inversely proportional to time taken.) x x
− = 8 ⇒ x = 300 km.
5 15 25
t − t = 10 ⇒ t = 40
4
Usual time taken = 40 minutes. 7. 2 Let the distance covered by the man each time be ‘d’
km and time taken by him to cover ‘d’ km at a speed of
3. 1 Let the normal speed be x km/hr and normal time taken 10 km/hr be ‘t’ hours.
be t hours.
d d
Total distance = xt km Then, = 10 and = 15
t t−2
xt 45 1 Equating the value of d : 10 × t = 15 × (t – 2)
⇒ =t+ ⇒ t = 2 hr
3 60 4 ⇒ t = 6 hours
x
4 d 10t
∴ Required speed = =
Short cut: t −1 t −1
4  45 1 10 × 6
 − 1 usual time = 60 ⇒ Usual time = 2 4 hr = = 12 km/hr.
3  5

4. 3 Let the actual time taken by the train to travel 100 km


1 928
100 8. 3 of the journey = = 464 km
be t hours. Then, speed = km/hr. 2 2
t
3
100 100 − x th of the time (10 hrs) = 6 hrs
Now, = t + 1⇒ t = 5
x x
Remaining distance = 464 km
100 100x Remaining time = 10 – 6 = 4 hrs
= km / hr
∴ Speed = 100 − x 100 − x
464
Speed for the remaining journey = = 116 km / hr
x 4

QA - 07 Page 1
9. 1 Since distance and time taken by the two men are As the cat and the train would reach B simultaneously,
same, their speed will be same. At 8 a.m., when the 2d d
the cat would cover the rest = km distance in
1 8 4
second man starts, the first man covers of the
2 the same time that the train takes to cover the whole
1 tunnel i.e. d km.
distance in 2 hr, i.e. he can cover distance in 1 hr. Therefore, the speed of the train = 4 × the speed of
4
the cat.
1
In the next hour, both cover distance and hence Hence, ratio of the speeds of the train and cat is 4 : 1.
4
meet at 9 a.m.
13. 3 The vehicle travels 19.5 km/L at the rate of 50 km/hr.

10. 3 Let the total distance be x. So the man travels a 19.5


So it should travel km/L at the rate of 70 km/hr
3x 1.3
distance at a speed 3a. Therefore, total time taken = 15 km/L.
5
to travel this distance The distance covered at 70 km/hr with 10 L = 10 × 15
= 150 km.
3x x  dis tance 
= = Time = speed 
(15a) (5a)  
2x 14. 2 18 ∝ 9
He then travels a distance at a speed 2b. Hence,
5 42 ∝ x; Here x = number of compartments
2x x
time taken to travel this distance = = .
(10b) (5b) 18 9
=
x x 42 x
So total time taken in going from A to B = + .
(5a) (5b) Simplifying, x = 49, but this is with reference to maximum
Now he travels from B to A and comes back. So total speed. Hence number of compartments would be one
distance travelled = 2x at a constant speed 5c. less in order to run i.e. 48.
2x
Hence, time taken to return = .
(5c )
60 6
Since the time taken in both the cases remains the 15. 1 Total time taken by B to cover 60 km = hr = hr
50 5
x x 2x .
same, we can write + =
5a 5b 5c 1
It stops at station C for hr.
1 1 2 4
Therefore, + = .
a b c 6 1 29
Now in  +  hr train X travels 70 × = 101.5 km
5 4 20
11. 2 Distance covered in 1 min = 10 × 20 = 200 ft
This means they do not cross each other by the time
970 train Y finishes its stop at station C.
Time taken for 970 ft = = 4.85 min.
200 Let they meet after t hr.
1 192.5
12. 2 AB is the tunnel and ‘d’ km be its length. Then, 70t + 50(t – ) = 180 ⇒ t = hr
4 120
A X B
192.5
3d Distance from A will be (70 × ) km
120
8
d = 112 km approximately
Let the current position of the cat be X. If it runs to-
wards A, it would reach A at the same time as the train 16. 2 Let length of the journey = x km and initial speed of the
reaches A. train = v km/hr.
However, if it runs towards the other end B, it would Companing the usual time taken and time taken on the
reach point Y at the same time when the train reaches day of accident, we get,
3d
A. Hence, point Y would be at a distance of km  v
8
1 3 x − 2  x 1
from X  + +  − =1
A X Y B  2 4 2v  v 2
 3 
3d 3d
8 8
d

Page 2 QA - 07
19. 1 Let the speed of the cyclist be x km/min, and that of
v v the car y km/min. The car had travelled for 10 min and
+ 60 x + − 60
2 3 2 x
Also + + − =1 the cyclist for 10 + 15 = 25 min when he was caught
v 4 2v v up by the car.
3 By this moment, they had covered the same distance.
Solving for x, we get x = 120 km and v = 60 km/hr. Consequently, 25x = 10y. By the time the car on its
return trip encountered the cyclist. The car had
Altervative method: covered 50y km and the cyclist 65x km. The sum of
Overall, the train was late by 1 hr 30 min. these distances is equal to twice the distance between
Out of this the train was standing for 45 min. So due to A and B.
Therefore, 65x + 50y = 38. Solving the system of
2 equations, we find x = 0.2 km/min and y = 0.5 km/min.
the reduction of the speed to rd of the original
3
speed, the train delayed by 45 min only. Speed of the car = 0.5 × 60 km/hr = 30 km/hr.

3  t 20. 1 For minimum petrol consumption, Zoheb should drive


∴  t − t  = = 45 min
2  2 160
or t = 90 min. where ‘t’ is the normal time taken by the at 40 kmph, petrol consumption = = 6.67 l.
24
train to cover the stretch from the point of accident to
the destination.
21. A B C D
Had the accident occurred 60 km down the line, the
train would have been delayed by 1 hr only. x y z
Due to the reduction in speed, the delay would have Let the starting point for all the three, the point at Hari
been 15 min only (1 hr – 45 min stoppage) gets off from the car, the point at which Ram picks up
∴ Normal time to cover the remaining distance Shyam and the final point of the journey be A, C, B and
⇒ t = 2 × 15 = 30 min. D respectively.
It means the train usually covers 60 km in 60 min (90 – Let AB = x, BC = y and CD = z.
30 min).
Speed of the train = 60 km/hr. x x + 2y
∴ = ⇒ y = 2x ...(i)
Total length of the stretch of the journey 5 25
= 30 km + 90 km = 120 km.
and z = z + 2y ⇒ y = 2z ...(ii)
17. 3 Let the distance to be covered be ‘d’ km. 5 25
From (i) and (ii), we get,
d x:y:z=1:2:1
Speed of A = km/min Therefore, x = z = 25 km and y = 50 km
30
x + y + 2y + z
d Hence, the required time =
Speed of B = km/min 25
70
25 + 50 + 100 + 25
= = 8 hours.
d d d 25
Speed of C = + = km/min
30 70 21
∴ Time taken by C = 21 minutes. 1 5 km
22. 3 2 .5 km 2 .5 km
Short cut: A B
G 1 G 2 G 3
30 × 70
Time taken by C = = 21 minutes.
30 + 70 30 × 5
AG1 = = 2.5 km
60
18. 1 Let the usual speed of Ankit be ‘x’ km/hr. G1G3 = 15 km
30 30 60 75 Time for AG1 = 5 min
+ = +
2x x x 60 Time for G1G3 + G3 A + G3B = 32.5 min
2
Time for AB = 5 + 32.5 = 37.5 min.
15 60 60 75 1 min is taken for transferring the patient into and out
⇒ + = + ⇒ x = 12 km/hr.
x x x 60 of the ambulance.
Hence, (40 – 37.5 – 1) = 1.5 min is remaining.

QA - 07 Page 3
23. We have to consider both, travelling to and from the
1
office. = × 54 × 15 × 48 × 15 = 2,91,600 m2 ≈ 0.29 km2
Suppose the speed of Honda is V. 2
Speed of McLaren = 2v
Speed of Ferrari = 4V 26. 3 Distance covered = 52.8 km
1
McLaren takes hr. Travel time = 1 hr 46
2 Time taken = 46 min = hr
Honda takes 1 hr. Travel time = 2 hr. 60
1 1 52.8
Ferrari takes hr . Travel time = hr × 60 = 68.86 km/hr
4 2 Average speed =
46
Note that after the first two days, 1 hour of the 5 hour
is already spent. In the next 3 days, he has to spend 4
hours. If he chooses Ferrari, in any of the subsequent Total dis tance travelled
days, he will hot be able to spent the 4 hours. So, in 27. Average speed = Total time taken
the next three days, he has to choose either Mclaren
or Honda.
45 + 50 + 25 120 × 3 360
He can choose to take McLaren 2 days and Honda = = = = 18 km / hr.
one day. This can be done in 3 ways. 45 50 25 20 20
+ +
MMH 15 25 15
 
MHM
HMM
  180
28. 2 Time taken to cover 180 km by car = = 3 hrs
60
For questions 24 and 25:
Since speed of Raju remains constant, distance covered by 240
him must be proportional to the time taken. Time taken to cover 240 km by train = = 3 hrs
80
Here, (48k)2+(90k)2=(102k)2 ⇒ BAC is right angled triangular
garden (k, being Raju’s speed)
300
B Time taken to cover 300 km by bus = = 6 hrs
50
Total time taken to complete the whole journey
= 3 + 3 + 6 = 12 hrs

Total dis tance


Average speed =
1 02 k 48 k Total time taken

180 + 240 + 300 720


= = = 60 km / hr.
12 12

29. 2 Let distance (in km) covered at a speed 4 km/hr be x.


C A
D x 50 − x 50
54 k ∴ + =
4 6 5
90 k
60% of 90k = 54k 3x + 100 − 2x
⇒ AD = (90 – 54)k = 36k
⇒ = 10
12
⇒ BD = k 482 + 362 = 60k ⇒ x + 100 = 120
⇒ x = 20
24. AD + DB + BA = (36 + 60 + 48)k = 144 k Hence, the distance covered at a speed of 4 km/hr is
⇒ Raju will take 144 minutes. 20 km.

25. 3 Raju’s speed = 15 m/min = k 30. Due to stoppages, the bus covers 9 km less in 1 hr.
⇒ CD = 54 × 15 meters and AB = 48 × 15 meters 9
Area of parking lot Time taken to cover 9 km = × 60 = 10 min .
54
1
= Area of ∆BDC = × CD × AB Hence, on average the bus stops for 10 min every
2
hour.

Page 4 QA - 07
QA - 08 : TSD - 2 CEX-Q-0209/20
Answers and Explanations

1 2 2 3 3 1 4 2 5 4 6 4 7 1 8 – 9 3 10 2
11 3 12 3 13 3 14 2 15 2 16 4 17 2 18 1 19 2 20 3
21 4 22 – 23 – 24 – 25 –

1. 2 Bridge Length = 10 km 3. 1 Let s be the speed and x be the length of the train.
Then
10
Train Length = = 2.5 km x
4 = 10 ⇒ x = 10s … (i)
Total distance travelled by train to clear bridge s
= 10 + 2.5 = 12.5 km
150 + x
Time taken by train = 2.5 min and = 22 ⇒ 150 + x = 22s … (ii)
s
Dis tance 12.5 From (i) and (ii), we get
Speed = = = 5 km/min
time 2.5 s = 12.5 m/sec and x = 125 m.
= 5 × 60 km/hr = 300 km/hr
4. 2 Distance travelled by 1st train in 30 minutes
2. 3 The boats will be colliding after a time which is given
1
by = 80 × = 40 km.
2
20 4
t= = hours = 80 minutes.
5 + 10 3 220 − 40 180
So, they will meet after = = = 1 hours.
Aftger this time of 80 minutes, boat (1) has covered 100 + 80 180
Hence, the distance = 40 + 80 × 1 = 120 km.
5 20
80 × kms = kms, whereas boat (2) has covered
60 3 5. 4 We see that the trains take 18 hr and 8 hr respectively
to complete their journeys.
10 40
80 × kms = kms. Let the distance between the stations be LCM(18, 8),
60 3 i.e. 72 km.
After 79 minutes, distance covered by the first boat ∴ Speed of the first train = 4 km/hr.
Speed of the second train = 9 km/hr.
 20 10 
= d1 =  −  kms By the time the train from A starts, train from B already
 3 60  would have travelled 9 km in 1 hr.
After 79 minutes, distance covered by the second Now the distance between the trains = 72 – 9 = 63 km.
Relative speed of the trains = 9 + 4 = 13 km/hr.
 40 10 
boat = d2 =  3 − 60  kms 63
  ∴ They will take hr after 11 p.m.
13
So the separeation between the two boats
i.e. approximately at 3.51 a.m.
1
= 20 − (d1 + d2 ) = kms.
4 6. 4 At 4.30 p.m. August Kranti Rajdhani Express crosses
Baroda. On the particular day, Rajdhani Express, which
is late by 10 min, would be 20 km away from August
Alternative method:
Kranti Rajdhani Express at 4.30 pm. Hence, they cross
Relative speed of two boats = 5 + 10 = 15 km/hr
when they together travel 20 km.
i.e. in 60 min they cover (together) = 15 km
Relative speed = 120 + 80 = 200 km/hr.
15 1 They cover 20 km in 6 min. Hence, they meet at
∴ in 1 min they will cover (together) = km
60 4 4.36 p.m.

QA - 08 Page 1
X Time taken for Mamta and Biman to meet from the
7. 1 X
instant Mamta start moving
A R B
(7000 − 600 × 3)
t y = 16 secon ds t x = 36 secon ds = 400 s. (when the condition is as
13
Let speed of X be a km/hr and that of Y be b km/hr. X stated in Case II).
and Y start simultaneously from A and B respectively The required time interval between the time when
to reach R.
2
a 16 2 AR 2 Mamta met Buddha and Mamta met Biman is 13 s,
= = ⇒ = 21
b 36 3 RB 3 therefore the condition is as stated in Case I.
Let Ar = 2K BR = 3k Total time elapsed from the instant Biman started
When Y starts late; X had already covered a distance moving till the instant Mamta meets Biman
of 20 km, to reach C. This time both of them meet at a
8800 13000
point S. = 600 + = s.
Let the distance RS be z km. 7 7
2 0 km Distance between Buddha and Biman when Mamta
z
13000
meets Biman = × (4 + 3) = 13000 m = 13 km.
A C R S B 7
After X reaches C, Y is still at B. We can assume that
X and Y start their respective journeys to meet at point 9. 3 We see that the car reduces its travel time by 30 min.
S. 30
So it reduces its one-way travelling time by
CS 2 2
We have =
BS 3 = 15 min. Thus, it would have met the children 15 min
earlier, i.e. 4.45 p.m. Now this means that the children
(AR − AC) + RS 2 (2k − 20) + z 2 walked from 4 p.m. to 4.45 p.m., i.e. 45 min.
or, = or, =
BR − RS 3 3k − z 3
⇒ 6k – 60 + 3z = 6k – 2z 10. 2 Spaceships A and B meet 10 hr after B leaves Jupiter.
⇒ 5z = 60 Distance travelled by A in 10 + 6 = 16 hr is 16 × 30,000
⇒ z = 12 km. = 4,80,000 km. Time taken by C is 16 – 12 = 4 hr.
480000
Hence, speed of C is = 1,20,000 km/hr.
S (S in gu r) 4
8.
N N 11. 3 After they meet, A has to travel a further
(N a nd ig ram ) (K o lka ta )
9,80,000 – 4,80,000 = 5,00,000 km.
NS = 7000 m and SK = 10,000 m
Speed of the car of Mamta = 36 kmph = 10 m/s. 500000
Hence, time is = 16.66 hr = 16 hr 40 min.
Case I: Buddha moves towards Nandigram and Biman 30000
moves towards Kolkata. B has to travel 4,80,000 km further.
Case II: Biman moves towards Nandigram and Buddha
480000
moves towards Kolkata. Hence B’s time is = 9.6 hr = 9 hr 36 min.
Time taken for Mamta and Buddha to meet from the 50000
instant Mamta starts moving Hence, the difference in the time is 7 hr and 4 min.
(7000 − 600 × 4) 2300
= = s. (when the condition is
14 7 12. 3 R S
P Q
as stated in Case I). 30o
Time taken for Mamta and Buddha to meet from the 60
instant Mamta starts moving 1 20
(7000 − 600 × 4) 4700
= = s. (when the condition is Before turning After turning
6 3
A (10 – t) hr t hr
as stated in Case II). B (8 – t) hr (t + 2) hr
Time taken for Mamta and Biman to meet from the
instant Mamta start moving Let VA and VB be their velocities. Then
(7000 − 600 × 3) 8800 VAt = 60 and VB(t + 2) = 120
= s. (when the condition is as Also VA (10 – t) + VB (8 – t) = 170
7 7
⇒ 10VA – 60 + 8VB – (120 – 2VB) = 170
stated in Case I).

Page 2 QA - 08
⇒ VA + VB = 35 Now average speed = 2.4
Now speed of N = 8
60 120 Now speed of S = y
∴ + = 35 = VA + VB
t (t + 2) 2×8× y
= 2.4 ⇒ y = 1.3
Solving, we get t = 4. 8+y
Required ratio = 1.3 : 8 ≈ 1:6
13. 3 2x x
17. 2 At 7.30 a.m., Navjivan Express is at 50 km from A and
6 :30 a .m . 8 :00 a .m . at the same time, Howrah-Ahmedabad Express is at
20 km from B.
Bus
Car 9 :30 a .m Hence, distance between the trains at 7.30 a.m. is
30 km.
Let the distance of Chennai from the point the car had
Relative speed = 50 + 40 = 90 kmph
over taken the bus, be 3x km. So, the car travels
3x km in 90 minutes. The car starts from Chennai at 30 1
Hence, time left = = hr = 20 min.
9:00 a.m. and meets the bus once again in another 90 3
30 minutes. The car travelled only x km in 30 minutes.
Obviously, the bus travelled only 2x km in 3hours 18. 1 Let tunnel = 8 km and speed of cat = 1 km/hr
while the bus met the car again. So the bus will travel
Time taken to reach entrance of tunnel by cat = 3 hr
1 Time taken to reach exit of tunnel by cat = 5 hr
3x km in 4 hours. It will reach Chennai at 11:00 a.m.
2 Train will cover the sum (length of tunnel) = 2 hr
Therefore, ratio of speeds of train and cat = 4 : 1
⇒ Speed of the train is greater by 3 : 1 than that of the
500
14. 2 The two cars meet = 5 hours later. During cat.
60 + 40
these 5 hr, the bird travels at 80 km/hr. So the total
distance travelled by the bird = 80 × 5 = 400 km. 19. 2 A P B
A ye sh a B h um ika
15. 2 For B, let the time taken to cover PQ, QT and TR be t1, A t 05 00 h rs A t 06 00 h rs
t2 and t3 respectively.
Let the speeds of Ayesha and Bhumika be ‘a’ and ‘b’
t1 : (t2 + t3) = 1 : 2.
units respectively.
Also since PQ = TR
Since Bhumika started running at 0600 hrs and they
4 met at 0700 hrs, the ratio of the distance AP : PB
∴ (t1)20 = t3(15). So t3 = t1 .
3 = 2a : b
We know that QT = y = 10 km It is also given that they met at the same point P while
10 2 coming back.
∴ t2 = = hrs. Thus, 2AP : 2PB = b : a
15 3
2
t1 t1 2a b a 1 a 1
∴ = =
1 ∴ = ⇒   = or = .
(t 2 + t 3 ) 4 t 2 2 b a  
b 2 b 2
+
3 1 3
20. 3 Let his average speed over the last stretch be x.
4 2 2 2 Hence, his average speed for first two stretches
Hence 2t1 = t + (or) t = or t1 = 1hr.
3 1 3 3 1 3 = 4x.
So the total time taken to cover the three stretches
2 4
So t1 = 1hr; t 2 = hr; t3 = hr. 4 2
3 3 = +
4x x
⇒ ( t 2 + t3 ) = 2 hr
His average speed over the race is 20 km/hr.
Total distance (PR) = t1 × 20 + (t2 + t3) × 15
= 1 × 20 + 2 × 15 = 50 km 6
Hence, the time taken to complete the race =
20
16. 4 If speed of N = 4, speed of S = 1,
4 2 6
2× 4 ×1 Hence, we have the equation + =
⇒ Average speed = = 1.6 4x x 20
4 +1
Solving this equation, we get x = 10 km/hr.
2 3
Since time available is , speed =
3 2

QA - 08 Page 3
21. 4 An escalator can be considered similar to a stream. 23. Let the speed of the escalator be x steps/sec.
The only difference is that a stream usually flows It is given that Rajat covers 11 steps in 35sec.
down, and this escalator is moving up. So in these 35 sec escalator will cover 35x steps.
So in this case, we can apply the same formula as Therefore, the total number of steps on the escalator
that of boats and streams, except for the fact that the must be 11 + 35x. …(i)
downstream condition here would correspond to a Similarly, Sharad takes 21 sec and covers 39 steps.
man moving up (since he is moving up with the So in these 21sec escalator will cover 21x steps.
escalator) and the upstream condition would Therefore, total steps on the escalator must be
correspond to a man moving down (as he is moving 39 + 21x. ...(ii)
down against the direction of the escalator). Solving (i) and (ii) we get;
Now, let the distance to be covered on the escalator 11 + 35x = 39 + 21x
x ⇒ x=2
be ‘x’. Hence, his downstream speed = and his Hence, total number of steps = 81.
30

x 24. In the time Samar takes 45 steps, Vivek takes 15 steps.


upstream speed = . Therefore, for every 3 steps of Samar, Vivek takes 1
90
step.
Therefore, his speed when the escalator is not moving If Samar takes 1 min for every 3 steps, then he takes
(corresponds to speed of boat in still water)
1
(Upstream speed + Downstream speed) 3
min for every step.
=
2
45
 x x  For 45 steps, he takes = 15 min.
 +  3
 30 90  = 2x So, Vivek takes 1 minute for every step.
=
2 90 For 40 steps, he takes 40 min.
Difference between their times = 25 min
x Difference between the number of steps covered
Hence, time taken by him at this speed = = 45s
 2x  = 45 – 40 = 5
 90 
  i.e. the escalator covers 5 steps in 25 min
1
which means, speed of the escalator is steps/min.
5
22. Due to movement of the escalator, the speed has
If the escalator is moving, Samar climbs 45 steps and
5
become of original speed. Hence, time should 1
2 the escalator covers 15 × = 3 steps.
5
2 3 Hence, total number of steps = 45 + 3 = 48.
become of the original time, thereby saving of
5 5
25. If Shyam takes 1 min for every 3 steps, then he takes
3 1/3 min for every step.
original time. Therefore, of original time = 3 sec For 25 steps, he takes 25/3 min, i.e. 8.33 min.
5
So Vyom takes 1/2 min for every step.
⇒ Original time = 5 sec.
For 20 steps, he takes 20/2 min, i.e. 10 min.
So, in 5 sec, Harish can cover 5 × 2 = 10 steps.
Difference between their time = 1.66 min.
∴ There are 10 steps in the escalator. Escalator takes 5 steps in 1.66 min and difference in
number of steps covered = 5
Speed of escalator is 1 step for 0.33 min,
i.e. 3 steps per minute.
If escalator is moving, then Shyam takes 25 steps and
escalator also takes 25 steps.
Hence, total number of steps = 50.

Page 4 QA - 08
QA - 09 : TSD - 3 CEX-Q-0210/20
Answers and Explanations
1 1 2 3 3 2 4 4 5 3 6 4 7 4 8 2 9 2 10 –
11 2 12 – 13 1 14 1 15 1 16 2 17 4 18 – 19 2 20 3
21 4 22 1 23 1 24 1 25 1 26 – 27 – 28 2 29 2 30 4

1. 1 Distance to be covered by A = (500 – 170) = 330 m. On solving, we get x = 50 m


In the time A covers 2 m, B covers 3 m. The distance between their apartment and the school
is given by (200 + x) = 200 + 50 = 250 m
3 
So, in the time A covers 330 m, B covers  × 330  m
2  5. 3 Suppose they normally meet at 9 a.m. at X.
= 495 m XY = 5 km. So A takes 15 min to cover it and B takes 20
∴ When A reaches the winning post, B covers 495 m min to cover it. So, when A, B are at Y the time is
and therefore remains 5 m behind. 9.15 a.m. when B reaches X it would be 9.35 a.m.
Hence, A wins by 5 m. Hence, B is 35 min late on that day.

2. 3 Let the speeds of A, B and C be SA, SB and SC. 6. 4 Situation (I):


In whatever time Karan covers a distance of 100 m,
1000 25
SA : SB = = Arjun covers 90 m in the same time.
960 24
Situation (II):
1000 40
SB : SC = = Now Karan is 10 m behind the starting point. Once
975 39
again to cover 100 m from this new point Karan will be
taking the same time as before. In this time Arjun will
1000
SA : SC = be covering 90 meters only. This means that now both
936
of them will be at the same point, which will be 10
Hence, A beats C by 64 m. meters away from the finish point. Since both of them
are required to cover the same distance of 10 m now
3. 2 Let the speeds of P, Q and R be S P, S Q and S R and Karan has a higher speed, he will beat Arjun by 1
respectively. metre.
∴ SP : SQ = 120 : 100 = 6 : 5
and SQ : SR = 120 : 96 = 5 : 4
⇒ SP : SQ : SR = 6 : 5 : 4 Divya 100 Preeta 85
7. 4 = , = .
Neeta 90 Divya 100
150 × 4
Hence, the distance = 150 − = 50m.
6 Preeta 100 85 17
Hence, = × = .
Neeta 90 100 18
4. 4 Let the distance where they met be 'x' metres from the
Hence, when Neeta runs 400 m, Preeta runs
school.
So the distance travelled by A = 200 m and that travelled 17
( 400 ) ≈ 377.77 m. Hence, Neeta beats Preeta by
by B = (2x + 200) m 18
Since the ratio of their speeds is 2 : 3, the ratio of the 400 – 377.77 m, i.e. 22.23 m (Approximately)
distances travelled should also be 2 : 3.

200 2
Hence, we have =
(200 + 2x ) 3

QA - 09 Page 1
8. 2 C  L − 18   L 
   t = L − 8
A B  t   L − 12 
2 .5 km ⇒ L = 48 m .
5 km

C (9 :3 0) 10. Let the speed of A and B be 50 m/sec and 40 m/sec,


R a m : A (9 :0 0 A M ) B (10 :0 0 A M ) respectively.
5000
Time taken by A to complete the race =
50
A C B (10 :0 0 A M ) = 100 sec.
(11 :0 0 A M ) (1 0:3 0 ) Time interval after which A crosses B every time

R a m : @ 5 kph 400
= = 40 sec.
50 − 40
S h ya m : A (9 :4 5 A M ) C (1 0 :00 A M ) B (10 :1 5 A M ) Hence, in 100 seconds A will cross B two times.

11. 2 Let length of the track = L


L
A C B (10 :1 5 A M ) ∴ Re quired time =
(1 0:4 5 A M ) (1 0:3 0 ) Relative Speed

S h ya m : @ 10 kph L 28 × 16 28 × 16 112
= = = = sec.
It is clear from the diagram that at 10:30; Shyam L

L 28 − 16 12 3
overtakes Ram. 16 28

Alternate: At 10:15 the situation is as show:


12. Sta rting p oint
2 0 m /se c
1 .25 km B A 15 m /sec
C
A B
D 1 40 00 –x x
S h ya m a t B
m ovin g @ 10 kph 1 st m ee tin g po in t
R a m at D
m ovin g @ 5 kh /h Let x be the distance covered by A.
Dis tance
Time taken for Shyam to overtake Ram Time =
Speed
1.25
= × 60 = 15 min. x 14000 − x
(10 − 5) ∴ =
15 20
⇒ Shyam overtakes Ram at 10:30 AM.
⇒ 35x = 14000 × 15
⇒ x = 6000 m = 6 km .
9. 2 Let L be length in metres of the race, which A finishes
in t seconds.
13. 1 A and B will meet at 5 – 3 = 2 points, irrespective of the
L radius of the circular track.
Speed of A = m/s
t
14. 1 Required ratio = 5 : 15 = 1 : 3
L – 12 (Since the time taken by both of them is same, distance
Speed of B = m/s
t covered will be directly proportional to their respective
speeds.)
L – 18
Speed of C = m/s
t 15. 1 If they travel in same direction they will meet at 10 – 3
Time taken by B to finish the race = 7 distinct points.
If they travel in opposite direction they will meet at
L  L  10 + 3 = 13
= s=  t s
(L − 12) / t  L −12  distinct points.
Hence, the ratio = 7 : 13.
In this time, C covers (L – 8) m

Page 2 QA - 09
For questions 16 and 17: When Charu and Chiku are running 21. 4 Circumference of the circular track = 105 km.
in the opposite direction. Let the speeds of A, B and C be 3x km/hr, 5x km/hr and
7x km/hr respectively.
A and B will meet for the first time after
16. 2 Charu and Chiku will meet for the first time after
105 105
= hrs
3300 5x − 3x 2x
= 82.5 seconds
(25 + 15 ) B and C will meet for the first time after

So they will meet for the third time after 105 105
= hrs
82.5 × 3 sec = 247.5 seconds 7x − 5x 2x
By this time total distance covered by Charu is So, A, B and C will meet for the first time after
25 × 247.5 m = 6187.5 m 105  105 105 
Hence, they are a distance of 2887.5 m i.e. hrs  LCM of and 
2x  2x 2x 
(6187.5 – 3300) m from point A.
But this is not the shortest distance. 105 105
Distance travelled by A in hrs = × 3x
The shortest distance between the point A and the 2x 2x
point at which they meet for the third time is 105 × 3
= km
(3300 – 2887.5) i.e., 412.5 m. 2
105 105
17. 4 Chaman will be at point B for the fifth time after Distance travelled by C in hrs = × 7x
2x 2x
(30 + 4 × 60) = 270 seconds.
105 × 7
At this point Charu has covered 270 × 25 m = 6750 m = km
2
i.e. he is at a distance of (6750 – 6600) m = 150 m from
point A. 105 × 4
Hence, the required difference = = 210 km.
At this point Chiku has covered 270 × 15 m = 4050 m 2
i.e. he is at a distance of (4050 – 3300) m = 750 m
22. 1 Since Kanishk is standing on meteor, there would be
from point A.
no effect on speed of Vaibhav due to rotation of the
Hence, the shortest distance between Charu and Chiku meteor.
is 750 + 150 = 900 m.
1
Speed of Vaibhav = 20% of 2.5 km/sec = km / sec
18. Ratio of speeds of the fastest and slowest runners is 2
2 : 1. Hence they should meet at only one point on Distance = π d = π × 7 = 22 kms
circumference i.e. the starting point (As difference in
ratio in reduced form is 1). For the two of them to meet 22 km
Therefore time recorded by Kanishk =
for first time, faster should have completed one 1
km / sec
complete round over slower one. Since two of them 2
meet for the first time after 5 min, faster one should
= 44 sec.
have completed 2 rounds (i.e. 2000 m) and slower
one should have completed 1 round (i.e. 1000 m) in
23. 1 This question is more of a visualisation than a
this time. Thus, faster one would complete the race
calculation. The correct time in the clock is 2:29:50.
(i.e. 4000 m) in 10 min.
24. 1 Let the position of minute hour be m.
0.736 × 60 0.736 × 60
19. 2 Required time = =
2.25 + 3.5 5.75 11
m − 30h
θ=
= 7.68 minutes 2

20. 3 As options are independent of n


11
Let n = 2 ⇒ 84 = × m − 30 × 7
2
1
Time taken for first round = + 1 + 2 + 4 = 7.5 min
2 (30 × 7 − 84) × 2 (30 × 7 + 84) × 2
Time taken for second round ⇒m= or
11 11
= 8 + 16 + 32 + 64 = 120 min
⇒ m ≈ 23 or 53.
120
Ratio = = 16.
7.5

QA - 09 Page 3
25. 1 In a correct clock the hour and minute hands coinside 28. 2 The clock is losing 12 minutes every 24 hours.
5 Hence, by 1:45 p.m. the following day i.e. after 18
after every 65 minutes. hours the clock would have lost 9 minutes. And in the
11
next 20 minutes. it would have lost another 10 seconds.
5 Hence, when the clock showed 1:45 p.m. actual time
∴ 64 minutes of correct clock = 65 of the faulty
11 would have been 1:35:50 p.m.
clock
4 29. 2 In a watch that is running correct, the minute hand
∴ 60 minutes of correct clock = 61 of the faulty
11 5
should cross the hour hand once in every 65 +
clock 11
4 min. So they should ideally cross three times once in
Hence, the incorrect clock gains 1 minute/hr
11  720  2060
3 ×   = 11 min = 196.36 min. But in the watch
8  11 
∴ Total gain in 24 hr = 32 minutes.
11 under consideration they meet after every 3 hr, 18 min
15 793
26. Number of hours between 4 pm and 7 am = 15 hrs and 15 s, i.e. (3 × 60 + 18 + )= min = 198.25
60 4
15 min. In other words, our watch is actually losing time
∴ Actual time elasped = = 12 hrs. (as it is slower than the normal watch). Hence, when
1.25
∴ The correct clock will show 4 am. our watch elapsed 198.25 min, it actually should have
elapsed 196.36 min. So in a day, when our watch will
27. It is clear from the question that in 60 hrs (LCM of 15 elapse (60 × 24) = 1440, it should actually elapse
and 20) one clock will gain 12 minutes and the other  196.36 
1440 ×  = 1426.27. Hence, the amount of time
198.25 
will lose 6 minutes
The time difference between the two clocks in 60 hrs 
= 18 m lost by our watch in one day = (1440 – 1426.27)
Both the clock will show the same time shen time = 13.73, i.e. 13 min and 50 s (approximately).
different = 12 hrs (i.e. 720 m)
30. 4 If a clock has to strike 4 or 4 times, there are 3 time
60
∴ Required time × 720 hrs intervals between the 4 strikes (Since the first strike
18
happens at the zeroth second).
= 2400 hrs = 100 days
Both the clocks will show the same time again after So in 7 seconds the pendulum elapses 3 time intervals.
100 day. Hence, the required day is Wednesday and To strike 11, there has to be 10 time intervals, which
time is 1 pm.
10 × 7
will take = 23.33 seconds.
3

Page 4 QA - 09
QA - 10 : TSD - 4 CEX-Q-0211/20
Answers and Explanations

1 1 2 1 3 2 4 2 5 4 6 4 7 4 8 4 9 – 10 4
11 – 12 3 13 4 14 – 15 3 16 – 17 4 18 1 19 2 20 2
21 3 22 3 23 4 24 4 25 3 26 1 27 3 28 1 29 1 30 2

1. 1 Let the speed of boat x km/hr. Then 1 2


r = x – s ⇒ x = r + s. 4. 2
2 00 m
m m
Time taken for return trip = = hr. W
s + x r + 2s 3 00 m
2 1
2. 1 Let ‘b’ and ‘c’ be the speed of boat in still water and
current respectively. Let W be the width of the lake in meter and S1, S2 are
Time taken in upstream = 2 × Time taken in downstream speeds of the boats
When, they meet for the first time, we get
AB AB
⇒ = 2× 300 (W − 300)
b−c b+c =
S2 S1 … (i)
⇒ 2b – 2c = b + c
⇒ b = 3c
(W − 300 + 200) (W − 200 + 300)
The required ratio is 3 : 1. Also, =
S2 S1
3. 2 Let the distance between point Q and point P be ‘x’ m. ⇒ (W – 100)S1 = (W + 100)S2
Let the speed of A, B and the river be a, b and r ⇒ 100 (S1 + S2) = W(S1 – S2) …(ii)
respectively. From (i), 300 (S1 + S2) = W S2
x 20 W S2
Therefore, = … (i) or, S1 + S2 =
(a + r) (b − r) 300

(x + 60) 40 100S2 W
Also, = … (ii) In (ii) W(S1 – S2) =
(a + r) (b + r) 300
Dividing equation (i) by (ii) we get: ⇒ 300 S1 – 300 S2 = 100 S2
⇒ 300 S1 = 400 S2
2x (b + r)
= … (iii)
(x + 60) (b − r) S1 400 4
⇒ S = 300 = 3 .
Given that b = 5r 2

2x
Therefore, = 1.5 ⇒ x = 180 m 5. 4 It is obvious that the cork is carried away by the flow
(x + 60)
of the water (i.e. it has the same speed as the water).
Distance between points Q and R = 180 + 20 = 200 m. So if the swimmer is swimming away from the cork
for half an hour (upstream), it will take him another
(a + r)
Putting value of x = 180 in (i) we get: = 9. half hour to swim back to the cork again (downstream,
(b − r)
where he reaches the cork at the moment it is passing
below the bridge). So in that 1 hr time, the cork has
a+r
⇒ = 9 ⇒ a = 35r = 7b moved 1 km towards to the bridge.
4r
The water in the canal flows at a speed of 1 km/hr.
Ratio of speed of A and B = 7 : 1.

QA - 10 Page 1
6. 4 Let x be rate of rowing by Rahul, and y be the rate of
PQ PQ
current in mph. = ...(ii)
y + x 3y − x
12 12 y 1
– =6⇒ 2 = [If we assume flow of the river is from Q to P, then
x–y x+y x – y2 4 3y + x = y – x, which is impossible.]
From (ii)
x2 – y2 3y – x = y + x ⇒ y = x
⇒y= ... (i)
4 Substituting in equation (i), we get
When Rahul doubles his rowing rate, then we have
(PQ − 30) 30
= ⇒ PQ = 60 meters.
12 12 1 2y 2x 2x
– =1 ⇒ =
2x – y 2x + y 4 x 2 – y 2 12
11. 8 children and 12 men = 4 + 12 i.e. 16 men
4x 2 – y 2 ( 3 1 man = 2 children).
⇒y= ... (ii)
24 9 × 16
∴ 12 men will take = 12 days.
Hence, from (i) and (ii), we have 2x2 = 5y2 12

3 2
y 8
5 2
Putting x2 = y in (i), we get y = 2 ⇒y= .
12. 3
1 1 1
+ = ...(i)
2 4 3 A B 12

1 1 1
+ = ...(ii)
7. 4 A C 20
4 10 x – 4 10
1 1 1 1
B + + = ...(iii)
A A B C 8
x – 2 30 2 30
Adding (i) and (ii), we get

2 1 1 8
+ + = ...(iv)
If x is the width of the river then speed at which they A B C 60
travel is proportional to the distances they travel. Subtracting (iii) from (iv), we get
Hence from the diagram, we get
1 8 1 1
410 x − 410 + 230 = − =
= A 60 8 120
x − 410 410 + x − 230
Hence, A alone can complete the whole work in 120
Solving the quadratic equation, we get x = 1000 m. days.

For questions 8 and 9: 13. 4 Let us assume that the work can be completed in n
Let the cruising speed of the plane and the time difference days. Thus,
between A and B be y km/hr and x hours respectively.
Distance between A and B = 3000 kilometers. For, the plane 6 n−8 n
+ + =1
moving from city A to City B: 3000 = (7 – x) × (y – 50). This is 12 36 54
satisfied for x = 1 and y = 550. These are the only values given
n−8 n 1
in the options that satisfy the above equation. ⇒ + =
36 54 2
8. 4 The required difference = 1 hour.
3n − 24 + 2n 1
⇒ =
9. Plane’s cruising speed = 550 km/hr. 108 2

3
10. 4 Let the speeds of Aku and Chika be y m/s and 3y m/s ⇒ 5n = 54 + 24 = 78 ⇒ n = 15 .
5
respectively and the speed at which the river flows
be ‘x’ m/s. As per the question, the possible set of
equations are:

(PQ − 30) 30
= ...(i)
y+x (3y − x)

Page 2 QA - 10
20. 2 Let x hr be the required time to fill the tank.
1 1 5
14. Work done on the first day = + =
8 12 24  1 1 
∴ Given  − x =1
1 1 3  10 20 
Work done on the second day = + =
12 24 24
x
⇒ = 1 ro x = 20
1 1 4 20
Work done on the third day = + =
8 24 24 ∴ The tank can be filled completely in 20 hr.
Work done on the first three days

5 3 4 12 1 21. 3 As A starts puffing, the length of the cigarette that


= + + = =
24 24 24 24 2  
gets burnt in one cycle  A  → B → 
Thus, the work will be completed in 6 days.  gap gap 

= (2 × 3) + 3 + (3 × 3) + 3 = 6 + 3 + 9 + 3 = 21 mm.
15. 3 Let the required number of days be x.
84 mm will get burnt in four such cycles.
 1 1  5
∴x + + =1
 20 30  20  84 
3 = 4 ,
⇒ x = 9.  21 
Time taken in one cycle = 2 + 3 + 3 + 3 = 11s.
16. Let units of work done in an hour by a man, a woman So, total time = 4 × 11 = 44s.
and a boy be m, w and b respectively.
15m = 24w = 36b
22. 3 Let total work be 60 units. 1st tap fills 3units per minute,
⇒ 12w = 7.5m and 6b = 2.5 m
2nd tap fills 4 units per minute and waste pipe empties
Let the number of men required be x.
2.25 (15m × 12 × 8) = (xm + 12w + 6b) × 30 × 6 5 units per minute. So, when opened together all three
⇒ 18m = (xm + 7.5m + 2.5m) will fill (3 + 4 – 5) = 2 units per min. Hence in first 9 min,
⇒ x = 8. 18 units of tank is filled. Since the waste pipe is closed
∴ The remaining 42 units will require
1 1 9
17. 4 In a minute, (A + B) fill + =
12 15 60 42
= 6 min to fill.
3+4
9 27
In 3 min, they fill 3 × = parts of the tank.
60 60
23. 4 The water supply does not start until the water level
27 33 reaches the 5,000-gallon mark, which will take 10 hr
Remaining part = 1 − =
60 60
 5000 
  . Once the level reaches that mark, there is
33  500 
So B will take × 15 = 8 min 15 s
60
an inflow at the rate of 300 gallons per hour and an
outflow at the rate of 500 gallons per hour. So effective
18. 1 Let the slower pipe takes x hr to fill the reservoir.
emptying rate will be 200 gallons per hour. At this rate,
1 1 1 the tank will be emptied in another 25 hr. So the total
+ =
x x−5 6 time taken to empty the tank is 35 hr.
⇒ x2 – 5x = 12x – 30
⇒ x2 – 17x + 30 = 0 24. 4 Total amount of work = 60 man-hours
⇒ (x – 15) (x – 2) = 0 From 11 am to 5 pm, 6 technicians = 36 man-hours
⇒ x = 15hr [3 x ≠ 2] From 5 pm to 6 pm, 7 technicians = 7 man-hours
From 6 pm to 7 pm, 8 technicians = 8 man-hours
19. 2 Let the required time be x hours. From 7 am to 8 pm, 9 technicians = 9 man-hours
Total = 60 man-hours
1 1 1
− = ⇒ x = 21.
3 x 3.5

QA - 10 Page 3
25. 3 Following is the situation as steps (i) to (vi) are On 8th day, part of work remained
followed (according to question statement):
7 1 7 5 2 1
= − = − = =
A B C D 60 12 60 60 60 30

–20 20 2 1
3 >
15 30
90 –90
∴ The work will be completed on 9th day by C and A.
–10 10

–50 50 28. 1 Let the capacity of the tank be 180 units. (LCM of 10,
12, 15 and 18)
–100 100
Amount of water drawn in one minute by
110 –110 Outlet Pipe A = 18 Units
Outlet Pipe B = 15 Unts
Total + 60 30 –40 –50
Outlet Pipe C = 12 Unts
D gets emptied first, it gets emptied in 20 minutes. Outlet Pipe D = 10 Unts
Hence option (3). Let the total time taken to empty the tank with the given
conditions be ‘t’ minutes.
26. 1 Chandra and Mayank complete 63% work in 7 days. Amount drawn by A = 1 × 18 = 18 units
So, they complete 9% work in a day. Amount drawn by B = 2 × 15 = 30 units
Now, since the ratio of their efficiency Chandra and Amount drawn by C = (t–5) × 12 = (12t – 60) units
Mayank is 5 : 4, so one of them completes 5% work a Amount drawn by D = t × 10 = 10t units
day, and other one 4% a day. Obviously, the person ⇒ 18 + 30 + 12t – 60 + 10t = 180
doing 5% work a day is the fastest worker. 96 8
Time taken by the fastest worker = 20 days. 22t = 192 or t = minutes = 8 minutes
11 11

1 1 3 29. 1 Let the work done by a man, a woman and a child in a


27. 3 On 1st day part of work completed = + =
10 20 20 day be m, w and c respectively.
Then, m = 3c and w = 2c
1 1 1 ∴ 20m + 30w + 36c = 60c + 60c + 36c = 156c
On 2nd day part of work completed = + =
20 30 12 Total units of work done in second case
= (15m + 21w + 30c) × 7 × 18
1 1 2 = (45c + 42c + 30c) × 7 × 18 = 117c × 7 × 18
On 3rd day part of work completed = + =
30 10 15 Hence, the required total wages
In three days part of work completed
78
= × 117c × 7 × 18 = Rs. 7,371.
3 1 2 11 156c
= + + =
20 12 15 30
30. 2 If the ratio of diameters of 3 inlet pipes of second tank
22 is 1 : 2 : 3, the ratio of their cross-sectional areas will
In 6 days part of work completed =
30 be 1 : 4 : 9. Suppose cross-sectional area of inlet pipe
of first tank be k. Then the cross-sectional areas of
22 8
Remaining work = 1 − = three pipes of second tank will be 4k, 16k and 36k
30 30 respectively. Total cross-sectional area of pipes of
On 7th day, part of work remained second tank is 56k.
Time taken in filling the second tank is
8 3 16 9 7
= − = − =
30 20 60 60 60 k 5
= 10 × × 2 × 2 = min
56k 7

Page 4 QA - 10
QA - 11 : Numbers - 1 CEX-Q-0212/20
Answers and Explanations

1 3 2 1 3 4 4 2 5 – 6 4 7 2 8 4 9 – 10 1
11 2 12 – 13 2 14 1 15 4 16 1 17 3 18 4 19 3 20 4
21 4 22 – 23 3 24 2 25 – 26 1 27 1 28 2 29 2 30 –

1. 3 Since n3 = n × n × n is odd, it implies that n is odd. This 7. 2 Let the number be N, then we get
is because any number when multiplied by an even 25x2536 = [N × (3×4) + 3×13 – 3×4(12×3)] × (3×8)
number gives even number while product of two odd = 72[4N –131].
numbers is odd. By the same logic, both n2 and n4 are So, the number has to be multiple of 9. Among the
odd. Now (n – 1)(n + 1) = n2 – 1 is even since the choices only (2) is satisfies the condition.
difference of two odd numbers is even. Thus, both III
and IV are true. Hence, (3) is the most appropriate 8. 4 Since x < y < z, then x + y + z = k.
answer. For values of k = 6 or k = 7, there are unique values of
(x, y, z)which are (1, 2, 3) and (1, 2, 4).
2. 1 Let the four consecutive positive integers be x – 1, x, For k = 8, (x, y, z) can be either (1, 2, 5) or (1, 3, 4).
x + 1 and x + 2.
x(x + 1) (x – 1) (x + 2) + 1 (odd)
ccc
= (x3 – x) (x + 2) + 1 9. cb × 3 = = 111 = 37 × 3
= x4 + 2x3 – x2 – 2x + 1 c
= (x2 + x – 1)2. Hence cb = 37
Hence, statements A and C are true. ∴ c × b = 21

3. 4 Take any three odd positive numbers and veryfy the 10. 1 If P1 and P2 are twin-primes then P1 will be of the form
options. 6K – 1, where K is a natural number.
P1 = 6K − 1 and P2 = 6K + 1 .
4. 2 The number of integers between any two integers is
always one less than the obsolute difference between P1 + P2 = 6K − 1 + 6K + 1 = 12K
the integers.
∴ 12 would always divide P1 + P2.
E.g. 3 > 1 > 0. Where 3 = x, 1 = y.
There is one number between 3 and 1 (excluding 3
11. 2 Let the two numbers be x and y.
and 1)
It is given that x × y = 1.
i.e. 3 – 1 – 1 = 1
We know that AM ≥ GM
i.e. x – y – 1.
x+y
⇒ ≥ xy ⇒ x + y ≥ 2 × 1 ⇒ x + y ≥ 2.
5. If XY is the number, the value of the number is 10X + Y. 2
The value of the number formed by reversing the digit
is 10Y + X. 12. In such numbers we cannot have 0 or 1 at unit’s
The difference in the value is 9 (Y – X) which is equal place.
to 18.
When we have 2 in unit's place, we have one such
So Y – X = 2. There are 7 possible pairs of ( X, Y ) … number i.e., 12.
(3, 1) to (9, 7). So, apart from 13, there are 6 When we have 3 in unit’s place, we have two such
possibilities.
number 13 and 23.
Similarly we can say that When we have 9 in unit’s
6. 4 Let the four-digit number be denoted by aabb place, we have eight such numbers. So number of
= 11 × (100a + b) such numbers is
Now since aabb is a perfect square 100a + b should (1 + 2 + ... + 8) = 36
be a multiple of 11. Hence the resulting number has 72 digits
The only pair of values of a and b that satisfy the
above mentioned condition is a = 7 and b = 4. Now
7744 is a perfect square.

QA - 11 Page 1
13. 2 76th natural number = 76 19. 3
We have 7 perfect squares and 3 perfect cubes from Initial count 1 2 3 4 5 6 7 8 9 10 11 12 13 14 15 16 17 18 19 20

2 to 75 in which 64 occur twice (because of being Original seat No. 1 2 3 4 5 6 7 8 9 10 11 12 13 14 15 16 17 18 19 20

both a perfect square and a perfect cube) After count 1 1 2 3 4 5 6 7 8 9 10 11 12


Hence, 9 numbers must have been removed. Original seat No. 1 4 6 8 9 10 12 14 15 16 18 20
The number 76, if we start a series of natural numbers
After count 2 1 2 3 4 5 6 7
from 2, will be the 75th number. Original seat No. 1 8 10 14 15 16 20
If we do not include the above 9 numbers in this series
then 76 becomes the 75 – 9 = 66th number. After count 3 1 2 3
Original seat No. 1 14 16
Subsequently 80 will be the 70th number.
But 81 being a perfect square cannot be included.
Original seat no of the person sitting in the 3rd seat at
Hence, 82 will be the 71st number and subsequently the end is 16.
the 76th number will be 87.
20. 4 Let N = abc & M = cba.
14. 1 The last digits of the 2 three digit numbers can be
(0, 6), (1, 5), (2, 4), (3, 3), (8, 8) an (7, 9) N = a b c
∴ The last digit of product of these numbers can be 0, M = c b a
5, 8, 9, 4 and 3.
S = a + c 2b a + c
∴ Four digits i.e. 7, 6, 2 and 1 cannot be the last digit of
the product of these numbers. For S to be a 3-digit number, a + c must be less than or
equal to 9. As N and M, both are 3-digit numbers,
15. 4 9p = 32p is a perfect square. neither a nor c can be zero.
As 3 is a prime number p must also be a perfect So, the minimum value of (a + c) is 2. The ten’s digit of
square. Let p = k2 the number S is 2 × b. So, it can be 0, 2, 4, 6 or 8. The
Now, 4p is a perfect cube and 4p = (2k)2 prime number 383 qualifies to be S hence,
Minimum possible value of p = 4(22) = 16 S can be prime also.
The expression p2 – 16p + 1 will have minimum possible Option (4) as correct.
value at p = 16 only as any other value of p is much
greater than 16 and correspondingly the expression 21. 4 um + vm = wm
will carry a higher value. u2 + v2 = w2
p2 – 16p + 1 for p = 16 is equal to (16)2 – 16(16) + 1 = 1. Taking Pythagorean triplet 3, 4 and 5, we see
m < min(u, v, w)
16. 1 The 2 digits at unit’s & tens place can be only 2 or 3. Also 1’ + 2’ = 3’ and hence m ≤ min(u, v, w)
∴ The hundredth digit must be 6. So the number can
be 623 or 632. On reversing the digits, the numbers 22. Let the numbers be K, L, K + L,
become 326 or 236. But the difference between 623 ⇒ KL(K + L) = 12(K + L)
& 326 is 297. Therefore, the tens digit of the number
⇒ KL = 12.
will be 2.
⇒ {K, L} = {1, 12}, {2, 6} or {3, 4}
17. 3 Let the two digit number be ab. Hence, product P = 156, 96 or 84
Value of ab = 10a + b ∴ Required sum = 156 + 96 + 84 = 336.
We have,
10a + b + 10b + a = 11(a + b) is a perfect square. 23. 3 999 × D = a1a2a3.
3 11(a + b) is a perfect square.
∴ (a + b) should also be a multiple of 11 but a + b can a1a2a3
⇒D=
not be equal to 22 or more as a and b cannot be more 999
than 9. So D must be multiplied by 1998 as 1998 is a multiple of
∴ a + b = 11 999.
Now we have these 8 solutions.
(2, 9), (3, 8), … …, (9, 2) 24. 2 LCM of 2, 3, 4, 6, 12 = 12
12 6 12 4 12 3 12 12
18. 4 It is obvious that one who chooses the integer 2 3 4 62 121
‘2’ wins. ∴ 34 is greatest.
So winning sequence is
2, (2 × 2 + 1), 2(2 × 2 + 1)+1.…
( )
2 2
or, 2, 5, 11, 23, 47, 95, 191, 383, 767, 1535 25. Let 8 + 2 7 = a + b or a + b + 2a b = 8 + 2 7
∴ B should choose 1535 to ensure his victory.
a2 + b = 8 and a b = 7 . a = 1 and b = 7 satisfies it.

Hence, answer is 1 + 7 .

Page 2 QA - 11
Case II: 2x–72 (1 + 21039–x) = n(n + 1)
x3 Now 2x–72 = 21039–x
2 3
+y 3
26. 1 = 256 or 2 x = 256 . Hence x3 + y3 = 8.
3 ⇒ 2x = 1039 + 72
2− y
⇒ x ≠ integral value.
x2 − xy Hence, x has only one value.
a ×a 2
+ y 2 − xy
= 81 or 3 x = 81 or x2 + y2 – xy = 4.
− y2
a
x = 2 and y = 0 satisfies this. Hence xy = 0 29. 2 3n + 2 + 3n −1
3n + 5 + 3n + 2
27. 1 Let the three consecutive positive integers be equal to
‘n – 1’, ‘n’ and ‘n + 1’. 3n −1 33 + 1
 
⇒ n − 1 + n2 + (n + 1)3 = (3n)2 =
3n + 2 33 + 1
 
⇒ n3 + 4n2 + 4n = 9n2
⇒ n2 − 5n + 4 = 0 3n
∴ n = 1 or n = 4 3 1 1
= n 2 = =
Since the three integers are positive, the value of ‘n’ 3 .3 3.32 27
cannot be equal to 1, therefore the value of ‘n’ = 4 or m
= n – 1 = 3.
30. We have
Hence, the three consecutive positive integers are 3,
(1) 1010 < n < 1011
4 and 5.
(2) Sum of the digits for ‘n’ = 2
Hence, option (1) is the correct choice.
Clearly-
(n)min = 10000000001 (1 followed by 9 zeros and
28. 2 Here 270 + 21039 + 2x or 270 (1 + 2969 + 2x–70)
finally 1)
But 270 is the square of an even number
Obviously, we can form 10 such numbers by shifting
Then 1 + 2969 + 2x–70 has to be the square of an odd
‘1’ by one place from right to left again and again.
number.
Again, there is another possibility for ‘n’
So 1 + 2969 + 2x–70 = (2n + 1)2
n = 20000000000
⇒ 2969 + 2x–70 = 4n2 + 4n So finally : No. of different values for n = 10 + 1 = 11
or 2967 + 2x–72 = n(n + 1) ans.
So it is the product of two consecutive numbers.

Case I: 2967(1 + 2x–1039) = n(n + 1)


Now 2967 = 2x–1039
⇒ x = 2006

QA - 11 Page 3
QA - 12 : Numbers - 2 CEX-Q-0213/20
Answers and Explanations

1 – 2 – 3 3 4 – 5 – 6 2 7 4 8 3 9 – 10 1
11 – 12 1 13 1 14 2 15 1 16 4 17 2 18 – 19 1 20 –

1. N = 25 × 34 × 53 Number of factors
The powers of 2, 3 and 5 that are present in the
l. Product of the factors = (N) 2 = N60
number are (0, 1, 2, 3, 4, 5), (0, 1, 2, 3, 4) and (0, 1, 2,
3).
a. Total number of factors is 6 × 5 × 4 = 120. 2. N = 32 × 22 × 32 × 54 = 22 × 34 × 54
b. For odd factors take only 1 power of 2 i.e. 0 a. Total number of factors of N is (2 + 1) × (4 + 1) ×
power. Hence, required number of factors (4 + 1) = 75. If N is written as a product of two
= 1 × 5 × 4 = 20. numbers, both of them must be factors of N. Here
c. Number of even factors = total factors – odd N is a perfect square and thus one of the factors
factors = 120 – 20 = 100. will be of the form N × N . For other 74 factors,
d. N has 3 prime factors i.e. 2, 3, and 5. Also, 1 is
neither prime nor composite. 74
there will be = 37 pairs. So a total of 38 pairs
So total number of composite factors = 120 – 4 2
= 116. are there.
e. A factor will be a multiple of 5 if it contains at least
one power of 5. So out of the powers mentioned b. Two different ways will be 37 as N × N will
in N, one power of 5 (i.e. 0 power) is not not be allowed now.
considered. c. As N is an even number, it cannot be written as a
So, number of factors 6 × 5 × 3 = 90. product of 2 odd numbers.
f. 45 = 32 × 5. To make a number a multiple of 45 we d. Let N = 2a × 2b. Hence 34 × 54 = a × b. Again a and
need to take at least 2 powers of 3 and at least 1 b both must be the factors of 34 × 54 and for each
power of 5. So powers of 2, 3 and 5 now allowed value of a there must be a unique value of b. So
are (0, 1, 2, 3, 4, 5), (2, 3, 4) and (1, 2, 3). total of 5 × 5 = 25 ways. Out of these 25 ways,
Hence, the number of factors that are divisible by there will be exactly 1 way when both a and b
45 = 6 × 3 × 3 = 54. are equal. In other 24 ways, the unordered ways
g. Perfect squares will have only even powers of 24
the primes. So powers of the primes must be will be = 12. Hence total number of ways is
2
(0, 2, 4), (0, 2, 4) and (0, 2) respectively.
So number of factors that are perfect square is 13.
3 × 3 × 2 = 18. e. As one of them is even so all powers of 2 must be
h. Perfect cubes will have only those powers which there in that number only. Let N = (22 × a) × b. So
are divisible by 3. So powers used to make factors our answer would be same as the total number
as perfect cube are (0, 3), (0, 3) and (0, 3) of factors of 34 × 54 i.e. (4 + 1) × (4 + 1) = 25.
respectively. f. A number N can be written as a product of 2 co-
So, the number of factors that are perfect cube prime numbers in 2P – 1 ways where P is the
= 2 × 2 × 2 = 8. number of prime factors. Hence there are 23 – 1
i. If the unit digit of a number is 0 it must be a multiple = 4 ways.
of 2 as well as 5. So power 0 will be omitted from
powers of 2 and 5. 3. 3 660 = 22 × 3 × 5 ×11
Hence, number of factors with unit digit 0 ∴ Prime factors are 2, 3, 5 and 11.
= 5 × 5 × 3 = 75.
j. For last 2 digits to be 0 the number must contain at 4. Now 350 = 2 × 7 × 52
least 2 powers of 2 and 5 both. Hence such factors Number of even factors = (Total number of factors) –
will be 4 × 5 × 2 = 40. (Total number of odd factors)
k. Sum of the factors is (20 + 21 + 22 +…+ 25)(30 + Number of even factors = 12 – 6 = 6.
31 + … + 34)(50 + 51 + 52 + 53) = 1189188.

QA - 12 Page 1
5. 464 = 24 × 29 = 4(22 × 29) (2a), (5b) – number of sets = 6 × 2 = 12
So, the number of factors of 464 that are divisible by (3a), (5b) – number of sets = 4 × 2 = 8
4 = 3 × 2 = 6. (2a × 3b), (5c) – number of sets = 6 × 4 × 2 = 48
(2a × 5b), (3c) – number of sets = 6 × 2 × 4 = 48
6. 2 If we factorize 1296 we get 1296 = 24 × 34 (3a × 5b), (2c) – number of ses = 4 × 2 × 6 = 48
Therefore the total number of factors is (4 + 1) (4 + 1) ∴ Total number of such sets = 188.
= 25 Combining both the cases, total sets possible
Therefore those numbers that have exactly 3 factors = 188 + 48 = 236.
will be in the form of 22 × 30 or 20 × 32
So there are only 2 such numbers 4 and 9. 13. 1 According to the given condition, the number of factors
of the number is 5.
If a number has an odd number of factors, it has to be
7. 4 Option (1): 999 = 33 × 371
the square of a natural number. Also 5 can be written
Number of factors of 999 = (3 + 1) × (1 + 1) = 8
as a product of two numbers in only one way i.e. 5 ×
Option (2): 1001 = 71 × 111 × 131 1 and thus the number must be of the form a4, where
Number of factors of 1001 a is a prime number. Only two numbers viz. 24 and 34
= (1 + 1) × (1 + 1) × (1 + 1) = 8 satisfy the aforementioned conditions.
Hence, the required sum = 81 + 16 = 97.
Option (3): 1010 = 21 × 51 × 1011
Number of factors of 1010 14. 2 n = 3000 = 23 × 3 × 53
= (1 + 1) × (1 + 1) × (1 + 1) = 8 Pure Factors: 2, 22, 23, 3, 5, 52 and 53
Hence, option (4) is the correct answer. Hence, the required answer = 2 + 4 + 8 + 3 + 5 + 25 +
125 = 172
16n2 + 7n + 6 6
8. 3 Let M = = 16n + 7 + 15. 1 If NX is in the form aP × bQ × cR ……
n n Therefore, number of factors will be (P + 1)(Q + 1)
(R + 1)….
6
For to be an integer, n has to be a factor of 6 i.e. 1, Here (P + 1)(Q + 1)(R + 1)… = 24
n
360 = 23 × 32 × 5, is the smallest number which has
2, 3 and 6. Therefore, n can take 4 values. 24 factors.
Therefore, sum of the digits = 3 + 6 + 0 = 9
9. Simplifying the equation, we get
 3 2 16. 4 All of them are distinct non-zero single digits numbers
 4n + 3n + 2n + 1 + n 
2
so they can take values 1, 2, 3, 4, 5, 6, 7, 8, 9. The
 
maximum value of the sum of any of any three of the
To make this a natural number, the maximum value numbers can be 9 + 8 + 7 = 24. Accordingly
of n is 2. b + c + d = 9 or 18 ...(1)
a + c + d = 8 or 16 or 24 ...(2)
10. 1 Sum of all numbers from 1 to 100 = 5050 a + b + d = 5 or 10 or 15 or 20 ...(3)
Sum of all multiples of 2 = 2550 a + b + c = 23 ...(4)
Sum of all multiples of 5 = 1050 For their sum to be 23, the numbers, a, b and c must be
Sum of all multiples of 10 = 550 9, 8 and 6 only (in any order). Using this (b + c + d)
Hence, required value = 5050 – 2550 – 1050 + 550 ≠ 9, (a + c + d) ≠ 8 and (a + b +d ) ≠ 5 or 10. Now we
= 2000. can write:
b + c + d = 18 ...(5)
11. The total number of factors = (3 + 1)(2 + 1)(1 + 1) = 24. a + c + d = 16 or 24 ...(6)
12 of these factors would give N when multiplied with a + b + d = 15 or 20 ...(7)
one of the 12 other factors. a + b + c = 23 ...(8)
We have deduced (a, b, c ) are (6, 8, 9) in any order.
12. 1 N = 2 6 × 34 × 52 Now (b + c) could be 14 or 15 or 17. Putting these
Case [A]: when none of the elements is 1. values in equation (5) and (7) we get:
The three factors must be some powers of 2, 3 & 5 d = 4 or 3 or 1 ...(From equation (5))
respectively. d = 1 or 6 ...(From equation (7))
Total number of such sets = 6 × 4 × 2 = 48 As d has a unique value, it must be consistent with
Case [B]: when one of the elements is 1. both the above results hence d must be 1. In equation
The other two factors could be of the form (7), (a + b) is 14 or 15 or 17 and d = 1
(2a), (3b) – number of sets = 6 × 4 = 24
⇒ (a + b + d) = 15 only.

Page 2 QA - 12
⇒ (a + b) = 14 No. of such cases = 2 × 2 = 4 ways.
⇒ (a, b) is (6, 8) in any order. ...(9) 5 & (2 o r 4 )
(b + c) = 17 ⇒ (b, c) must be (9, 8) ...(10)
From (9) and (10) it can be seen that b must be 8.
⇒ a = 6 and c = 9 ...(11) 3 1
So (a, b, c, d) are (6, 8, 9, 1) in that order.
(4 or 2)

17. 2 Here again no. of ways = 2 × 2 = 4 ways


So, there are 4 + 4 = 8 nos, in which (1, 3) are at
odd positions. Similarly there are 8 nos. in which
(1, 5) are at odd positions. So, in all there are 16
nos. where 1 is at unit’s place. Similarly there are
le ft m o st digit(L M D ) righ t m ost digit(R M D ) 16 nos. with 3 at unit’s place and 16 more with 5
at unit’s place.
Odd positions can be counted in 2 ways. Summing up all the odd unit’s digits = 16(1 + 3 + 5)
(i) Counting from the LMD-end: = 144
From (i) and (ii) we can now, sum up all (even or
odd) nos. at units place = 72 + 144 = 216
Hence answer is (2)

18. P = (1 × 30) × (2 × 30) × (3 × 30) × (4 × 30) × … ×


o dd (29 × 30) × (30 × 30).
p osition s P = 3030 × 30!
We have 1, 2, 3, 4 & 5 to be filled in these blocks. P = 256 × 3 44 × 537 × 7 4 × 112 × 13 2 × 17 × 19 × 23 × 29.
Odd nos. (1, 3, 5) to be filled in at odd positions.
Other places are to be filled by even nos. (2 or 4) N = 57 × 45 × 38 × 5 × 3 × 3 × 24 = 25 × 35 × 5 2 × 19 2
Let’s count, how many such nos. are there with 2 Let the two numbers be ‘19a’ and ‘19b’ respectively
at the unit’s digit. such that ‘a’ and ‘b’ are relatively prime to each other.
o dd ⇒ (19a) × (19b) = N = 25 × 35 × 5 2 × 19 2

⇒ ab = 25 × 35 × 5 2
Possible pairs (a, b) are
2
(25 × 35 × 52, 1); (25,35 × 52 ); (25 × 35, 22 ); (25 × 52, 35 ) .
Odd nos. can be filled in 3P2 = 6 ways. Therefore, N can be written as the product of two
The remaining two places are to be filled by 2 numbers, such that their HCF is 19, in 4 ways.
nos. (one odd no. left out of 1, 3, 5 & one even i.e.
4) in = 2 ways. Short cut method: Here, the answer has to be a
So, there are 6 × 2 = 12 number with 2 at the power of 2. (Why?)
rightmost place. Similarly, there are 12 such nos.
with 4 at the rightmost digits.
94 13 1 1
The sum of rightmost digits in all such number 19. 1 =3+ =3+ =3+
27 27 27 1
= 12(2 + 4) = 72 2+
13 13
(ii) Now counting from the RMD-end.
Let’s place 1 at the units place and check, how ∴ a = 3, b = 2 and c = 13.
many nos. are possible with (1, 3) at the odd Hence a + b + c = 3 + 2 + 13 = 18.
positions:
20. Number of ways in which a number can be
3 written as sum of two or more consecutive natural
1
numbers is one less than total number of odd factors
(4 or 2) of the number.
Here, 1000 = 23 × 53.
5 & (2 o r 4 ) So number of ways = 3 + 1 – 1 = 3.

QA - 12 Page 3
QA - 13 : Numbers - 3 CEX-Q-0214/20
Answers and Explanations
1 2 2 2 3 – 4 2 5 2 6 4 7 1 8 4 9 4 10 4
11 4 12 3 13 – 14 2 15 2 16 – 17 1 18 – 19 2 20 2
21 1 22 – 23 4 24 4 25 –

1. 2 Since dividing the number by 6, 15 and 17, leaves a 7. 1 Since HCF is 119, the numbers could be taken as 119u
remainder of 1, the smallest such number is the LCM and 119v, where u and v are coprimes.
of 6, 15 and 17 plus 1, viz. 510 + 1 = 511. This number Hence, 119u + 119v = 833, u + v = 7
is also divisible by 7 and hence the required answer is Since the numbers differ by the minimum possible
511. amounts ∴ (u, v) must be = (3, 4)
∴ Numbers are 357 and 476
2. 2 Difference between divisor and remainder
= 35 – 25 = 45 – 35 = 55 – 45 = 10 8. 4 Since 22 is the GCF of the two numbers, let the
LCM of 35, 45, 55 = 5 × 7 × 9 × 11 = 3465 numbers be 22x and 22y respectively such that x and
⇒ Required Number = 3465 – 10 = 3455. y are relatively prime.
22x + 22y = 462 = 22 × 21
3. The number which when divided by 4, 5, 6 and 7 ⇒ x + y = 21
leaves remainders 2, 3, 4 and 5 respectively will of We need to find all pairs of numbers such that they
the form [(LCM of 4, 5, 6, 7) k – (4 – 2)] i.e. 420k – 2, are relatively prime and their sum is 21. (1, 20), (2, 19),
where k is a natural number. (4, 17), (5, 16), (8, 13) and (10, 11) are the pairs that
∴ The required number = 9658. satisfy the aforementioned condition.

4. 2 Note that the difference between the divisors and the 9. 4 Difference between a two-digit number and the number
remainders is constant. with its digit reversed is always a multiple of 9.
2–1=3–2=4–3=5–4=6–5=1 This difference is equal to the HCF only in case of the
In such a case, the required number will always be number 54 (or 45).
[a multiple of LCM of (2, 3, 4, 5, 6) – (The constant Therefore, the value of a + b = 9.
difference)].
LCM of (2, 3, 4, 5, 6) = 60 10. 4 Start checking with option (1). LCM of ab3 and b3 will
Hence, the required number will be 60n – 1. be ab3. Similarly for option (2) also. LCM of ab and ab3
Thus, we can see that the smallest such number is is again ab3. If we examine the options one by one, we
(60 × 1) – 1 = 59 can see that all of them individually when paired up
The second smallest is (60 × 2) – 1 = 119 with b3, will give an LCM as ab3. Therefore the correct
So between 1 and 100, there is only one such number, choice should be none of these.
viz. 59.
11. 4 x – m is the HCF of both expressions, means x – m is
5. 2 Because each word is lit for a second, the factor for both
5 17 41   7 21 49  ∴x – m = 0 ⇒ x = m
LCM  + 1, + 1, + 1 − 1 = LCM  , ,  − 1 Substituting x = m in both expressions
2 4 8  2 4 8 
m2 – bm + a = m2 + cm – d
LCM(7,21,49) 49 × 3
−1= − 1 = 72.5 s a + d = (c + b) m
HCF(2,4,8) 2
a+d
⇒m=
6. 4 The product of two numbers is the product of the LCM b+c
and HCF of the two numbers. If the numbers are x and
y, then x + y = 140, xy = 28 × 168. 12. 3 Vice Presidents of HR, Operation and Sales visit the
Solving for x and y, the numbers are 84 and 56. plant after a gap of 2 days, 3 days and 5 days
respectively. So, VP (HR), VP (Operation) and VP
Alternative method: (Sales) visit the plant on every 3rd, 4th and 6th day
The two numbers are 28X and 28Y, where X and Y respectively.
th
are coprimes. Therefore, CEO meets the VPs on every 12 day (L.C.M.
Also 28(X + Y) = 140 of 3, 4 and 6) i.e. on 3rd, 15th, 27th of January and 8th
So X + Y = 5 of February, but CEO is on leave till 28th of January.
Since LCM 28 × 6 = 168, XY = 6 Hence, CEO will meet all the three VPs on 8th of
So X = 3, Y = 2 February.

QA - 13 Page 1
13. Largest sub-multiple of 840 is 420. e. Number of trailing zeroes is equal to the highest
Now 420 = 22 × 3 × 5 × 7 power of 10. 10 = 2 × 5. As 5 is a bigger prime than
So those three numbers can be 3, 140 and 420. 2, the highest power of 5 will be less than that of 2.
So, the maximum possible sum can be 563.
All the other combinations will leas to a lower sum. 150  150  150 
The power of 5 is  + + 
 5   52   53 
14. 2 1192 = 172 × 72. Since L.C.M. of (X, Y and Z) is (172 ×
= 30 + 6 + 1 = 37. So the number of trailing zeroes
72), power of 17(170, 171, 172) and powers of 7(70, 71,
in 150! will be 37.
72) have to be distributed among (X, Y, and Z)
Number of ways of distributing powers of 17 among
17. 1 n can be only 131 and 133. Please check how many
X, Y and Z is as given below:
times 5, 25 & 125 is there within 131, you will realise
X Y Z that there will be 32 'zeros' in the end of 131!

0 1
17 or 17 17 2 18. The number of zeroes in the number 100! is 24 since
17 0
2 0 1 2
17 17 or 17 or 17 100!  100! 
we have  +  = 24
0
17 or 17 1
17 2  5   52 
1
17 But adding 10 leaves only one trailing zeroes in the
172 170 or 17 1 or 172
number 100! + 10.
17 2 170 or 171 or 17 2 170 or 171 and 172
19. 2 Integers which will not be divisible by n will be all
Total number of ways = (2 + 3 + 2 + 3 + 9) = 19.
prime numbers 3, 5, 7, 11, 13, 17,19 and the composite
Similarly, the number of ways of distributing power of
number 4.
7 among X, Y and Z = 19
∴ Total number of ordered triplets (X, Y and Z) = 19 ×
20. 2 For n = 0 and 1, n! is a perfect square (which is 1).
19 = 361.
Otherwise n! is never a perfect square.
Alternative method: (Between n and 2n there must be at least 1 prime
Power of 17(170, 171, 172) and powers of 7(70, 71, 72) number)
have to be distributed among (X, Y, and Z)
17 2 can be distribeted (X, Y and Z) in 3 3 ways. 21. 1 87! – 60!
However we need to subtract the cases who only = 60! [(87 × 86 × ... × 61) – 1]
170 or 171 have to be distributed which is 23 = 60! (3k – 1)
∴ 17(0, 1, 2) can be distributed in (33 – 23) ways. ∴ Highest power of 3 is highest power of 3 in 60!, i.e.
Similarly for 7(0, 1, 2) = 19 × 19
Total number of ordered triplets  60   20   6 
 3  +  3  +  3  = 20 + 6 + 2 = 28
= (27 – 8) (27 – 8) = (33 – 23) × (33 – 23).      
(here [x] denotes greatest integer less than or equal
15. 2 If there are n numbers, the function h has to be to x)
performed one time less.
22. For 50! – x! to end in 6 zeroes, x! must also end in
16. a. The highest power of 2 is 6 zeroes. There are 5 numbers x = 25 to 29 which
150  150  150  150  150  would all have 6 zeroes in their factorials.
=  2 + 2 + 3 + 4 + …  7  Hence the answer is 5.
  2  2  2  2 
= 75 + 37 + 18 + 9 + 4 + 2 + 1 = 146. 23. 4 V! + (V + 1)! + (V + 2)!
b. The highest power of 3 is = V! + (V + 1) V! + (V + 2)(V + 1) V!
= V!(1 + V + 1 + V2 + 3V + 2)
150  150  150  150 
= + + +  = (V2 + 4V + 4)V! = (V + 2)2V!
 3   32   33   34  Now this will be a multiple of 36 for
= 50 + 16 + 5 + 1 = 72 V = 4, 6, 7, 8, 9, 10, 11, 12, 13 and 14.
c. 6 = 2 × 3 and the highest powers of 2 and 3 are ∴ It is possible for 10 values of V
146 and 72 respectively. So the highest power of
6 is 72 as 72 is lower among the two. 24. 4 P = 1 + 2.2! + 3.3! + … 10.10!
= (2 – 1)1! + (3 – 1)2! + (4 – 1)3! + … (11 – 1)10!
146
d. 48 = 24 × 3. Highest power of 24 is ≈ 25 = 2! – 1! + 3! – 2! + … 11! – 10! = 1 + 11!
4
Hence the remainder is 1.
which is lower than the power of 3 so the highest
power of 48 will be 24. 25. (0, 0) and (1, 1) satisfy the equation

Page 2 QA - 13
QA - 14 : Numbers - 4 CEX-Q-0215/20
Answers and Explanations
1 4 2 1 3 4 4 4 5 3 6 3 7 4 8 4 9 4 10 3
11 4 12 3 13 1 14 3 15 2 16 3 17 2 18 – 19 2 20 3
21 – 22 2 23 1 24 – 25 4 26 3 27 – 28 1 29 – 30 2

1. 4 We know that if a + b + c = 0, then a3 + b3 + c3 = 3abc. Case I:


∴ 723 – 553 – 173 = 3 × 72 × (–55) × (–17) When a – d = 5,
The above number is divisible by both 3 and 17. a = 6, 7, 8 and 9 and corresponding value of
Hence, option (4) is the correct answer. d = 1, 2, 3 and 4
The number of values of A is 4 × 10 × 10 i.e. 400.
2. 1 A number is divisible by 11, if the difference of the sum Case II:
of digits at odd places and sum of digits at even places When a – d = 0, b > c
is either 0 or a multiple of 11. Then number of values of A is 9(9 + 8 + 7 + 6 + 5 + 4 +
⇒ |2 + 5 + 7 + 1 –(3 + a + 9)| is either 0 or a multiple of 11 3 + 2 + 1) i.e. 9 × 45 or 405
⇒ |3 – a| is either 0 or a multiple of 11 Hence number of values of A is 400 i.e. 805.
∴ The value of ‘a’ is 3.
9. 4 (an – bn) is always divisible by (a + b) if n is an even
3. 4 For a number to be divisible by 9, sum of the digits of number. Thus, (76n – 66n) is divisible by (7 + 6) i.e. 13.
the number has to be divisible by 9. It can also be written as (343)2n – (216)2n.
Here, 8 + x + 0 + 1 = 9 + x. or (343n + 216n) (343n – 216n)
Hence, x is a multiple of 9 So, it can also be divisible by 127 and 559.
Since x is a natural number, x = 9.
10. 3 We can write
4. 4 23n – 1 = (23)n – 1 = (8)n – 1 N = (111)2 (62 + a2)
(x)n – 1 is always divisible by x – 1. Since we do not know what is a, the number N is
Therefore, 23n – 1 = (8)n – 1 is divisible by 8 – 1 = 7 definitely divisible by (111)2 = 12321.

5. 3 n3 – n = n(n2 – 1) = (n – 1) n(n + 1) = Product of three 11. 4 The expression (n5 – 5n3 + 4n) can be written as
consecutive numbers. So at least one number is even, (n – 2)(n – 1)(n) (n + 1)(n + 2) which is nothing but the
and one number is a multiple of 3. So the product is product of five consecutive numbers. Therefore the
divisible by 6. expression is divisible by 1 × 2 × 3 × 4 × 5 = 120.

6. 3 45n – 54n = 1024n – 625n 12. 3 The sum of four consecutive two-digit odd numbers
The above expression is always divisible by (1024 – lies between 56 and 384. Since the sum of four two-
625) i.e. 399, which is divisible by 133. digit numbers gives a perfect square on dividing by
Hence, option (3) is the correct answer. ten, the number has to be of the form 10a2, where a is
an even natural number. The possible values of a are
7. 4 Out of the three-digit numbers that are divisible by 55, 4 and 6. Accordingly, the numbers are 37, 39, 41 and
there are four numbers viz. 605, 715, 825 and 935 43 or 87, 89, 91 and 93.
that satisfy the given condition. ∴ 41 is a possible value of one of the odd numbers.

8. 4 A can be written as 13. 1 If k – 4 is divisible by 5, then the smallest k = 9.


1000a + 100b + 10c + d Since k + n2 is divisible by 5, the lease possible value
B can be written as that n can take is 4.
1000d + 100c + 10b + a
So A – B > 0 14. 3 The expression can be expanded and written as
⇒ (1000a + 100b + 10c + d) – (1000d + 100c + 10b + (n – 2) (n – 1) n ( n + 1)
a) > 0 The product of 4 consecutive terms is divisible by 24
⇒ 999a + 90b – 90c – 999d > 0 but not always divisible by 48.
This has to be divisible by 45.
Then, it can be written as 15. 2 Product of first 20 multiples of 30 can be written as
(990 + 9)a + 90b – 90c – (990 + 9)d (30) 20 [20!]. (30) 20 has 20 zeroes and 20! has
Now 9a – 9d has to be divisible by 45. 4 zeroes. Hence total number of zeroes is 24.
So a – d is 5 or 0

QA - 14 Page 1
16. 3 32n + 2 – 8n – 9 = 9 × 32n – 8n – 9 5
22. 2 34 when divided by 4 gives remainder 1.
= 9 × 9n – 8n – 9 = 9[(1 + 8)n] – 8n – 9
= 9 [1 + 8n + nC2 × 82 + nC383 … + 8n] – 8n – 9 45
= 64n + 9(nC2 × 82 + nC383 … + 8n) Units digit of 23 is 2.
= 64[n + 9 × (nC2 + nC38 … + 8n – 2)] = 64K. Cyclicity of 3 is 4:
5 35
17. 2 Sum of the first two natural number in any three 153
=
(16 − 1)
consecutive natural number, which digits sum are in 4 4
the form of 3k + 1, 3k + 2 and 3k + 3 or sum of all the 5
three such numbers would be divisible by 3. 153 when divided by 4 given a remainder –1 or 3.
Hence m must be either of the form 3n or 3n – 1 35
∴ Unit digit of 315 = 7.
18. Let X = AB 5 3 5
34
Now, as per the information given in the questions the Unit digit of 2 × 315 = 4.
value of B could be 1, 5 or 6.
The square of the number obtained by reversing the 23. 1 In the first 5 numbers, the sum of all numbers ending in
digits of AB has last digit as 6, which implies that the the 1, 3, 5, 7 and 9 has unit's digit 5. Similarly, in the
value of A is 6 or 4. next 5 numbers, the sum ends in 5. A similar pattern is
So possible values of AB from the above conclusion followed by subsequent sets of 5 numbers. Hence,
could be 61, 65, 66, 41, 45 and 46, but the square of the sum of 10 sets of numbers each ending in a 5 is 0.
the number BA should also be less than 3000.
So, number of distinct possibilities for X could be 61, 24. 132 ends in 9, 133 ends in 7, 134 ends in 1, thus (134)6
41 and 45. ends in 1

19. 2 Let A = abc, then B = cba 25. 4 To simplify the problem, lets say
Given, B > A which implies c > a … (1) 6007 ≈ 6000
as B – A = (100c + 10b + a) – (100a + 10b + 1) 60002 = 36000000 (8 digits)
B – A = 100(c – a) + (a – c) 60003 = 216000000000 (12 digits)
B – A = 99(c – a) and (B – A) is divisible by 7. Similarly, (6007)3 will also contain 12 digits
As 99 is not divisible by 7 (no factor like 7 or 72), ∴ The 12th digit will be the last digit of (6007)3 and it
(c – a) must be divisible by 7 {i.e., (c – a) must be 7, 72 will depend upon ‘7’
etc.}. As c & a are single digits. (c – a) must be 7 only, 73 = 343
the possible values (c, a) {with c > a} are (9, 2) & ∴ The 12th digit in (6007)3 will be 3.
(8, 1), with this we can write A as
A : abc ≡ 1b8 or 2b9 26. 3 The last two digits of any number in the form of 74n will
as b can take values from 0 to 9, the smallest & largest always be equal to 01.
possible value of are: For example 74 = 2401 and 78 = 5764801.
Amin = 108 and Amax = 299. Hence, option (3) is the correct choice.

20. 3 Work with options 27. 47523 can be written as (500 – 25)23
If A and C are the only persons lying, then B and D say Every term has atleast 2 zeroes and the last term is
the truth. Thus the number has to be one of 30, 60, 90 negative and ends in 25. Hence the number must end
or 120. Since A lies, the number has to be 120. in 75.
It also is in agreement with C lying. Thus A and C could
be the only persons lying. 28. 1 ((30)4)680 = (810000)680
The only person lying could be B as they could have Hence the right most non-zero digit is 1.
thought of the number as 30, 60 or 90 and all of A, C
and D would be speaking the truth. 29. Unit digit of the 22010, 62011, 72012 and 82013 is 4, 6, 1
Similarly, D could be the only person lying as any two-digit and 8 respectively.
divisor of 120 would make A, B and C's statement true. ∴ Required unit digit = 4 × 6 × 1 × 8 = 2.
Could C alone lie? If C lies that means the number
would be 120 which would make A's statement false. 30. 2 Following are the two cases that are possible.
Thus C alone lying is not a possibility. Case I:
P = Q.
21. Let the two-digit number be 10a + b. There are 9 such ordered pairs (P, Q).
According to the question, Case II:
10a + b = 4(a + b) … (i) P = 4, 6 or 8 and Q = 4, 6 or 8.
10b + a = 2(10a + b) – 6 There are 6 such ordered pairs (P, Q).
⇒ 19a – 8b = 6 … (ii) Therefore, there are 9 + 6 = 15 such ordered pairs
From equation (i) and (ii), we get (P, Q).
a = 2, b = 4.
Hence, the number is 24.

Page 2 QA - 14
QA - 15 : Numbers - 5 CEX-Q-0216/20
Answers and Explanations

1 1 2 4 3 3 4 3 5 – 6 4 7 4 8 2 9 2 10 2
11 2 12 2 13 4 14 – 15 2 16 – 17 1 18 – 19 2 20 2
21 3 22 1 23 4 24 2 25 –

1. 1 We can write (37)47 as (38 – 1)47 which when divided


 1421 × 1423 × 1425 
by 19 leaves a remainder of ( – 1)47 or 18. 5. Rem  
 12 

 251000   (26 – 1)1000   (118 × 12 + 5) × (118 × 12 + 7) × (118 × 12 + 9) 


2. 4 Rem   = Rem   = Rem  
 13   13   12 
   

5 × 7 × 9
 (–1)1000  = Rem   = 3.
= Rem    12 
 13  = 1.
 

 41 
6. 4 Rem   = 4
 34413   (–2)413   (–1)413 × 2413  6
 
3. 3 Rem   = Rem   = Rem  
 9   9   9 
     
 42 
Rem  =4
 (–1) 413 3 137 2  6 
× (2 ) ×2  
= Rem  
 9 
 43 
Rem   = 4
 (–1) × (9 – 1)137 × 22   (–1) × (–1)137 × 22   6
= Rem   = Rem  
 9   9 
 44 
Rem  =4
 (–1) × (–1) × 4   6 
= Rem   
 = 4.
 9 
Hence, any power of 4 when divided by 6 leaves a
remainder of 4.
4. 3 1523 = (19 – 4)23 = 19x + (–4)23 where x is a natural
number.
2323 = (19 + 4)23 = 19y + (+4)23 where y is a natural
number.
1523 + 2323 = 19 (x + y) + 423 + (–4)23 = 19 (x + y)
The above expression is multiple of 19. Hence, the
required remainder is 0.

QA - 15 Page 1
10. 2 The number is a series of 1 natural number, 2 natural
(723243 + 318243 ) numbers, 3 natural numbers and so on. Hence to get
7. 4
17 36 digits we need to find n for which 1 + 2 + 3 + … +
n ≥ 36. For n = 8, the sum is exactly 36 which means
(731 – 8)243 + (323 – 5)243 )
= the number will have 12345678 at the end.
17
To divide by 36, we need to divide the number by 4
As 731 and 323 are divisible by 17, the net remainder and 9.
is the same as when (–8)243 + (–5)243 is divided by The rule of 4 is to divide the last 2 digits of the number
17. i.e. 78. 78 divided by 4 gives a remainder of 2.
The rule of 9 is to divide the sum of the digits by 9. The


Rem 
{
 (–8)243 + ( −5)3 × ( −5)240
}  sum of digits is 1 × 8 + 2 × 7 + 3 × 6 + 4 × 5 + 5 × 4 + 6
× 3 + 7 × 2 + 8 × 1 = 120 which divided by 9 gives a
17 
 
  remainder of 3.
Hence, the remainder by 36 is the smallest number
which when divided by 4 and 9 leaves remainders of
{
 −2729 + ( −5)3 × (625)60
= Rem 
}  2 and 3 respectively i.e. 30. Hence answer 30.
17 
 
  11. 2 Let the number be x = 7p + 5.
∴ x2 = (7p + 5)2
⇒ x2 = 49p2 + 70p + 25
{
 ( −2) × (24 )182 − (125) × (612 + 13)60
= Rem 
}  ⇒ x2 = 7 (7p2 + 10p + 3) + 4
17  Hence, the remainder is 4.
 
 
12. 2 Given expression is

{
 ( −2) × (17 − 1)182 − (125) × (17 × 36k + (170 – 1)30 )
= Rem 
}  n! + (n! + 1) + (n! – 2) + (n! + 3) ..... + (n! – 2006)
= 2007 × n! + (1 – 2 + 3 – 4 ..... – 2006)
17 
  = 2007 n! + 1003 × (–1)
 
Now, for n = 1003, the expression 2007 n! – 1003 is
(where k is a natural number) clearly divisible by 1003.
Hence, the remainder is zero. Option (2) is the correct

{ } 
 (−2) × (17 – 1)182 − (17 × 7 + 6) × (17 × 36k + (17 × 10 – 1)30
= Rem
choice.

17  22
.......
  22 = 24k = 42k = (5 – 1)2k
  13. 4
= 5 N + 1 (N is a natural number)
∴ The required remainder = –2 – 6(0 + 1) = –8 or 9. When 5N + 1 is divided by 5, the remainder will be1.

8. 2 P = 30 k + x 14. We have, V256 = (V64)4


∴ P is prime, x cannot be a multiple of 2, 3 or 5. x also ∴ V is of the form 2K + 1.
has to be less than 30. ∴ It is odd.
Therefore possible values of x are 1, 7, 11, 13, 17, 19, Thus, V64 is also odd.
23 and 29. (V64) when divided by 16 will leave a remainder of 1
as fourth power of any odd number divided by 16
9. 2 1366 – 23 can be written as (133)22 – 23 leaves a remainder 1.
Remainder when 133 divided by 183 is 1.
Remainder when (133)22 divided by 183 is also 1. 15. 2 Let V be the number that when divides 34369 and
Remainder when (133)22 – 23 divided by 183 is – 22 31513 leaves the same remainder. V must be a factor
or 161. of 34369 – 31513 = 2856. When 31513 is divided by
2856 it leaves a remainder 97. Hence, any three-digit
factor of 2856, when divides 31513 the remainder will
be 97.

Page 2 QA - 15
22. 1 This question can easily be solved using Euler's
 54k   ( −1)4k   1 Theorem.
16. Rem   = Rem   = Rem   = 1.
 6   6  6 If a and m are co-prime to each other, the Remainder
φ(m)
obtained when a where φ(m) is the number of co-
17. 1 The terms 13, 53, 93, ...., 213 are of the form 4k + 1. primes to m that are less than m, divided by m is 1.
The terms 33, 73, 113, ..., 232 are of the form 4k + 3.
φ(53) = 52 (As 53 is a prime number)
 6 × (4k + 1) + 6 × (4k + 3) 
= Rem  
 4   352   3φ(53) 
Rem  = Rem  =1
 53   53 
   
6 × 1+ 6 × 3
= Rem   = 0.
 4 
 467   466 × 4   4φ(67) × 4 
Rem  = Rem  = Rem 
18. Since 13 is coprime to 100, it follows a power cyclicity 23. 4  67   67   67

     
of 20 in its last 2 digits.
So last two digits of 131982 = last two digits of 132 = 1 × 4 = 4.
= 69.
Hence, the required remainder is 69. 24. 2 This problem can be easily solved using Wilson’s
Theorem.
According to this theorem, when (n – 1)! is divided by
 29202   3202   (33 )67 × 3 
19. 2 Rem   = Rem   = Rem   n, where n is a prime number, the remainder obtained
 13   13   13
      is –1 or n – 1.

 12!   (13 – 1)! 


 (2 × 13 + 1)67 × 3   67  Rem   = Rem   = –1 or 12.
= Rem   = Rem  1 × 3  = 3.  13   13 
 13   13 
   

 16!   16! 
20. 2 Cyclicity for the last two digits of any number is 20. 25. Rem   = Rem  16 × 17 
 272   
Last two digits of 33334442 = Last two digits of 33332
= Last two digits of 332 = 89.
16!
Rem =0
16
21. 3 Let x, a and b, where a and b are co-prime to each
other. Let r1 and r2 be the remainders when x is
16! (17 − 1)!
divided by a and b respectively. Rem = Rem = –1 or 16
17 17
When x is divided by a × b, the remainder obtained will
be the smallest numbers which when divided by a Now using Chinese Remainder Theorem the required
and b leaves remainders r1 and r2 respectively. remainder will be the smallest number that when divided
21 = 3 × 7 by 16 and 17 gives remainders 0 and 16 respectively.
Such smallest number is 16. Hence, the required
464
Rem =1 remainder is 16.
3

464 (43 )21 × 4 (9 × 7 + 1)21 × 4


Rem = Rem = Rem =4
7 7 7
The smallest number which when divided by 3 × 7
leaving remainder 1 and 4 respectively is 4.
Hence, the required remainder is 4.

Note: This theorem is known as Chinese Remainder


Theorem.

QA - 15 Page 3
QA - 16 : Numbers - 6 CEX-Q-0217/20
Answers and Explanations

1 1 2 2 3 2 4 3 5 1 6 – 7 4 8 4 9 – 10 3
11 – 12 3 13 4 14 3 15 4 16 1 17 3 18 3 19 4 20 2
21 1 22 – 23 2 24 3 25 4 26 2 27 3 28 1 29 2 30 –

1. 1 Consecutively dividing 54 by 12, the remainders are 4 9. (412)7 = (2 × 70 + 1 × 71 + 4 × 72)10 = (205)10


and 6 in the reverse order. Now convert it to base 8
Hence, (54)10 = (46)12. 8 2 05

2. 2 Consecutively dividing 142 by 12, the remainders are 8 25 5


11( i.e. B) and 10 (i.e. A ) in the reverse order. 8 3 1
Hence, (142)10 = (BA)12.
0 3
3. 2 (12323)4 ∴ (206)10 = (315)8
Hence, (412)7 = (315)8
 
=  110111011
 1 2 3 2 3 2 10. 3 Convert the part before the decimal.
Divide 110111011 into groups of 3 digits from right to
left.
(1101)2 = 1× 20 + 0 × 21 + 1× 22 + 1× 23 = (13)10
For the part after decimal take negative powers of 2
(110 111 011)2 = (6 7 3 )8 = (P)8
⇒ P = 673. ∴ (0.011)2 = (0 × 2–1 + 1× 2–2 + 1× 2–3 ) = (0.375 )10
10
∴ The answer is 13.375.
4. 3 (ABCD)16 = 163 × 10 + 162 × 11 + 161 × 12 +160 × 13
= 40960 + 2816 + 192 + 13 = (43981)10
11. (12630)x = (3402)10
5. 1 We divide 1153 by 15 and write the remainders and
quotient at appropriate places. (
⇒ (3402 )10 = 0 × x0 + 3 × x1 + 6 × x 2 + 2 × x 3 + 1× x 4 )
10
∴ The number is 51D.
3402 = 3x + 6x 2 + 2x3 + x 4
6. (100)10 = (10)100. Hence, we need two digits. x = 7 satisfies the above equation.

12. 3 389 in base 3 will be written as ‘one followed by 89


7. 4 (1234 )6 = (4 × 60 + 3 × 61 + 2 × 62 + 1× 63 )10 zeroes’. When we add one to it, zero in unit’s place
will become one and we will have 88 zeros.
= (4 + 18 + 72 + 216)10 = (310 )10
13. 4 Since the last digit in base 2, 3 and 5 is 1, the number
8. 4 (1555)10 = (?)16 should be such that on dividing by either 2, 3 or 5 we
should get a remainder 1. The smallest such number is
31. The next set of numbers are 61, 91.
1 6 1 55 5 Among these only 31 and 91 are a part of the answer
16 97 3 choices.

16 6 1 Among these, (31)10 = (11111)2 = (1011)3 = (111)5


0 6 Thus, all three forms have leading digit 1.
Hence, the answer is 91.
∴ (1555)10 = (613)16

QA - 16 Page 1
14. 3 Given that (abc)5 = (cba)7 20. 2 169x + 13y + z = 786
or 25a + 5b + c = 49c + 7b + a Since x, y and z are single digit natural number, the
or 24a = 2b + 48c values of x, y and z must be 4, 8 and 6 respectively.
or 12a = b + 24c Hence, the required answer
As abc is a 3-digit number in base 5, the possible = (4 + 8 + 6)10 = (18)10 = (20)9.
values for a, b and c are 0, 1, 2, 3, 4 only.
The possible solutions of the above equation are: 21. 1 First we look for the leap years during this period.
(a, b, c) = (2, 0, 1) and (4, 0, 2) only ∴ Only 2004 is a leap year and the years 2002, 2003,
a can take value = 2 or 4 only 2005 and 2006 are the ordinary years.
b can take value = 0 only Number of odd days from 3rd July, 2001 to 3rd July,
c can take value = 1 or 2 only. 2006 = 1 + 1 + 2 + 1 + 1 = 6 odd days.
So, 3rd July, 2001 was 6 days before Wednesday, i.e.,
15. 4 The number of zeroes (as counted in the decimal on Thursday.
system) is same as the highest power of 6 that divides Therefore, 1st July, 2001 was Tuesday.
100! i.e. 48.
This, when counted in base 6, is 120. 22. The year 2008 is a leap year and a leap year calendar
repeats itself after 28 years. (If there is no non-leap
century year in between both the leap years.)
16. 1 The product of 44 and 11 in base 10 is 484. 2008 + 28 = 2036
If base is x, then 3414 So, 2036 will have the same calendar as that of 2008.
= 3x3 + 4x2 + 1x1 + 4 × x0 = 484
⇒ 3x3 + 4x2 + x = 480 23. 2 Number of odd days for January = 6
This equation is satisfied only when x = 5. Number of odd days for February = 1
So base is 5. Number of odd days for March = 3
In decimal system, the number 3111 can be written Number of odd days for April = 2
3 × 53 + 1 × 52 + 1 × 51 + 1 × 50 = 406. Number of odd days for May = 3
Number of odd days for June = 2
17. 3 Converting both the numbers in base 10, we get Number of odd days for July = 3
the numbers as 12, 30. Number of odd days for August = 3
So, the HCF of 12 and 30 is 60 and in terms of base 4 Number of odd days for September = 2
it is (12)4. Total number of odd days = 6 + 1 + 3 + 2 + 3 + 2 + 3 +
3 + 2 = 25.
18. 3 P = (222)X = 2X2 + 2X + 2 and ∴ 30th September 2012 will be Tuesday.
Q = (222)Y = 2Y2 + 2Y + 2
It is given that P – Q = 28. 24. 3 Day before yesterday was Monday,
⇒ 2X2 + 2X + 2 – 2Y2 – 2Y – 2 = 28 ⇒ Today is Wednesday.
2 2 ⇒ Day after tomorrow is Friday.
 1  1
⇒ X2 + X – Y2 – Y = 14 ⇒  X +  −  Y +  = 14 ∴ Four days after the day after tomorrow will be
 2   2  Tuesday.
⇒ (X + Y + 1) (X – Y) = 14
Therefore, the values of X and Y are (7, 6) and (4, 2) 25. 4 Number of odd days for January = 2
in that order. But Y cannot equal to 2 as there exists a Number of odd days for February = 1
number 222 in base Y and digit ‘2’ does not exist in Number of odd days for March = 3
base 2. Number of odd days for April = 2
⇒ X = 7, Y = 6 and (Q – X) = 79. Number of odd days for May = 3
Number of odd days for June = 2
19. 4 Given, (xxx)b = x3 Number of odd days for July = 3
⇒ (x × b2 + x × b + x) = x3 Number of odd days for August = 3
⇒ (b2 + b + 1) = x2 Total number of odd days = 3 + 1 + 3 + 2 + 3 + 2 + 3 +
3 = 19.
⇒x≥b
∴ 21th September will be Sunday.
But x is a digit in base b, therefore x < b
Hence, b cannot assume any value.

Page 2 QA - 16
26. 2 Tomorrow ⇒ Friday 29. 2 25th April 2006 was Wednesday.
Friday + 3 = Monday For next wedding anniversary to be on Wednesday,
Monday = 15th number of odd days must be a multiple of 7.
Other Monday ⇒ 22, 29 For an ordinary year, there is 1 odd day but for leap
Therefore, 30th June = Tuesday year, there are 2 odd days.
Hence, after 11 years, the number of odd days will be
27. 3 If February 29th, 2004 was a Sunday, then February 14, which is a multiple of 7.
1st was also a Sunday. ∴ On 25th April 2017 there will be Wednesday.
1st August is (31 + 30 + 31 + 30 + 31 + 1) 154 days
from 29th Feb. 154 is a multiple of 7. So 1st August 30. If a particular year, which is not a leap year, starts
was again a Sunday. with Monday, it will also end with Monday. For minimum
number of days, let’s consider a leap year N and
1st November is (30 + 30 + 31 + 1) = 92 days from 1st
assume its first day is Tuesday.
August. 92 = 7n + 1.
Let Gautam start his visit on the last day of the year
So, 1st November was a Monday. So, option (3) is the
(N – 1).
correct choice.
Year First day Last Day
28. 1 It is given that, 15th September, 1943 was Wednesday. N–1 Monday Monday
As, 1944 is a leap year. N Tuesday Wednesday
Therefore, there are two odd days from 15th September,
N+1 Thursday Thursday
1943 to 15th September, 1944.
So, 15th September,1944 is two days after Wednesday, N+2 Friday Friday
i.e., Friday N+3 Saturday Saturday
Hence, 19th September, 1944 is four days after Friday, N+4 Sunday Monday
i.e., Tuesday. So, on the first day of year (N + 4), Gautam will meet
the given condition.
So, the required number of days = 365 × 4 + 3 = 1463.

QA - 16 Page 3
QA - 17 : Algebra – 1 CEX-Q-0218/20
Answers and Explanations

1 2 2 4 3 – 4 – 5 3 6 2 7 1 8 2 9 3 10 2
11 2 12 2 13 3 14 1 15 2 16 4 17 4 18 1 19 3 20 3

21 3 22 2 23 1 24 1 25 3 26 3 27 4 28 4 29 3 30 4

1. 2 Statement (A), P(2) must be zero. As, 2 is a root of the 4. Since (x + 4) is a factor, we put x = –4 in the given
given polynomial. So it is true. expression.
Statement (B), if 1, 2 and –3 are roots of a polynomial, (A) So, 3(–4)3 + p(–4)2 – 30(–4) + 24 = 0
then it must be in the form of – 192 + 16p + 120 + 24 = 0
P(x) = A(x – 1) (x – 2) (x – 3), where A can take any 16p = 48 ⇒ p = 3.
value. But in this option, A = 1, which is not necessarily
(B) 3x3 + 3x2 – 30x+ 24 = 3(x + 4) (x – 2) (x – 1)
true or there must be fourth power of x in the polynomial.
Thus, roots are –4, 2 and 1.
Statement (C), the fourth root cannot be imaginary, as
imaginary roots exist in pairs. So statement (C) is not
correct. 5. 3 Using the remainder theorem we can conclude that
Statement (D), since product of the roots for this (–a)3 + a(a2) + a(–a) + 4 = 0.
⇒ (a – 2)(a + 2) = 0
E So, a = –2 or 2.
polynomial =
A

12 6. 2 We can factor the left hand side into (a – 2)2 + (c – b)2


So, 1 × 2 × (–3) × fourth root = =6 + (c + 1)2 = 0
2
Because the left hand side consists of a sum of
⇒ fourth root = –1
squares and the right hand side is 0, it must be that
Hence, it is true.
each square is 0.
So, statement (A) and statement (D) are true.
This implies that a = 2; c = –1; b = –1.
So, a + b – c = 2 + (–1) – (–1) = 2.
2. 4 f(x) = x 3 + 3x 2 + 3x

f(x +1) = (x + 1)3 + 3(x + 1)2 + 3(x + 1) 1 1


Given x + = 47 or x 4 + 4 + 2 = 49
4
7. 1
3 2 2
( )
= x + 1 + 3 x + 3 x + 3 x + 1 + 2x + 3 x + 3
x4 x

2
= x 3 + 6x 2 + 12x + 7  1  1
or  x 2 +  = 7 2 or x 2 + 2 = 7
 
 x2  x
Alternative method:
We can solve this question by giving value to x. e.g.
1
put x = 1 then or x 2 + +2=9
f(x + 1) = x3 + 3x2 + 3x x2
⇒ f(2) = 23 + 3(2)2 + 3 × 2 = 26 2
or  x +  = 32 or x + = 3
1 1
put x = 1 in the options
Hence, option (4) is the answer.  x x

3
3. If (x – 3) is a factor then f(3) = 0. So 27 – 45 + 3p + 9 1  1 1 1
x3 + = x +  − 3.x.  x + 
=0⇒p=3 x 3  x x x
If all the roots are integers, then the possible
combinations are 3, 3, –1 or 3, –3, 1. = 33 – 3 × 3 = 18.
Of the two, we find that only the first is possible since
sum of the roots = 5.

QA - 17 Page 1
8. 2 The given expression is symmetrical i.e. if we replace 13. 3 Let, the number of cars = x and number of auto-
a, b and c with each other, the expression remains rickshaws = y
unchanged. So, x + y = 160 … (i)
So, a = b = c and 4x + 3y = 550 … (ii)
Hence, the given expression can be re-written as On multiplying equation (i) with 4 and subtracting
equation (ii), we get y = 90
a2 a2 a2 1 1 1 Thus, there are 90 auto-rickshaws.
+ + = + + =1
2a2 + a2 2a2 + a2 2a2 + a2 3 3 3
14. 1 Let the age of Alia be x years.
∴ The age of Shrishti = (x + 2) years
Alternative method: Now, the age of Pariksha can be either (x + 5) or
We assume some values of a, b and c such that (x – 5) years.
a + b + c = 0 and a ≠ b ≠ c, and find the value of the Case I: The age of Pariksha is (x + 5) years.
The age of Aasma = (x + 5 – 6) = (x – 1) years.
expression that is given. So let a = 1, b = –1 and
Case II: The age of Pariksha is (x – 5) years.
c = 0.
The age of Aasma = (x – 5 – 6) = (x – 11) years.
a2 b2 c2 Hence, in both the cases Aasma is the youngest
So we find that + 2 + 2
2a + bc 2b + ac 2c + ab
2
person.
1 1
= + + 0 = 1.
2 2 15. 2 Since thief escaped with 1 diamond,
Before 3rd watchman he had (1 + 2) × 2 = 6
Before 2nd watchman he had (6 + 2) × 2 = 16
9. 3 x 2 + 5y 2 + z2 = 4yx + 2yz Before 1st watchman he had (16 + 2) × 2 = 36.
(x 2 + 4y 2 – 4yx) + z2 + y 2 – 2yz = 0
Alternative method:
2 2 Use the options and try.
(x – 2y) + (z – y) = 0
It can be true only if x = 2y and z = y 16. 4 According to the information given in the question,
Cost of 1 dozen apple + cost of 1 kg orange = Rs.20
Alternative method: Cost of 1 dozen orange + cost of 1 kg apple = Rs.16
Option (1) cannot be the answer as L.H.S. has z2 and Cost of 1 dozen orange + cost of 1 dozen apple
R.H.S has z = 3 × 12 = Rs.36
Option (2) cannot be the answer as they are ∴ Cost of 1 kg orange + cost of 1 kg apple = 0
contradictory conditions. Which is not possible. Hence, the data is inconsistent.
So, option (3) is the only possibility.
17. 4 Let there be x mints originally in the bowl.
10. 2 a3 + b3 + c3 – 3abc
= (a + b + c)(a2 + b2 + c2 – ab – bc – ca) 1
Sita took , but returned 4. So now the bowl has
3
(a3 + b3 + c 3 − 3abc)
∴ =a+b+c
(a2 + b2 + c 2 − ab − bc − ca) 2
x + 4 mints.
3
Here, a = 243, b = 257, c = 500
∴ a + b + c = 243 + 257 + 500 = 1000 1
Fatima took of the remainder, but returned 3.
4
11. 2 Since x2 cannot be less than 0 for any value of x, so
32 
| x 2 + 1|≤ 1 will hold only when x = 0. So the bowl now has  x + 4  + 3 mints.
4  3 
Hence, (x + 2)(x + 1)(x)(x − 1)(x − 2)L = 0 Eshwari took half of remainder that is

12. 2 Let the two-digit number be 10y + x. 1 3  2  


  x + 4  + 3 
So, x + y = 7 and y = x + 3 2  4  3  
On solving these equations we get,
She returns 2, so the bowl now has
x = 2 and y = 5
⇒ Required number = 52. 1 3  2  
  x + 4  + 3 + 2 = 17 ⇒ x = 48
2  4  3  
Alternative method:
Use the options and apply the given conditions.

Page 2 QA - 17
Short cut: Alternative method:
Since Sita was the first person to pick and she picks Use the options to solve the question.
e.g. if we pick option (2) which says each child must
1
up of the mint, but if you see the options, none of have received 900 chocolates.
3
Using this in the condition given in the question for
the option is a multiple of 3. child 1.

18. 1 3x + ky = 1 and 6x + 3y = 2 shall have infinite solutions 1


900 = 100 + (T − 100) {where T is total number of
10
3 k 1 3
if = = or k = chocolate}
6 3 2 2
⇒ T = 8100
Trying this in the conditions it satisfies.
19. 3 2x and 40 are even numbers. Therefore, 3y must be So, option (2) is correct and option (1) & (3) cannot be
an even number. There are 6 even multiples of 3 from correct simultaneously.
0 to 40. Hence, there are 6 values of (x, y) that satisfy
the equations. 23. 1 From given data, we can say that
They are (17, 2), (14, 4), (11, 6), (8, 8), (5, 10), (2, 12). f(x) = k(x – 1) (x – 2) (x – 3) ......(x – 49) + x + 3
So, f(50) = k49! + 53.
20. 3 Given equation is x + y = xy
⇒ xy – x – y + 1 = 1
24. 1 It is given that p + q + r ≠ 0 , if we consider the first
⇒ (x – 1)(y – 1) = 1
x – 1 = 1 & y – 1 = 1 or x – 1 = –1 & y – 1 = –1 option, and multiply the first equation by 5, second by
Clearly (0, 0) and (2, 2) are the only pairs that will –2 and third by –1, we see that the coefficients of x,
satisfy the equation. y and z all add up-to zero.
Thus, 5p – 2q – r = 0
21. 3 Given that 30X + 20Y = 1000 No other option satisfies this.
⇒ 3X + 2Y = 100 ...(i)
Let K pens be gifted away. 25. 3 Let x be the number of males in Mota Hazri.
⇒ 35X + 25(Y – K) = 1000 Chota Hazri Mota Hazri
7X + 5Y – 5K = 200 ...(ii) Males x – 4522 x
2(3X + 2Y) + X + Y – 5K = 200 Females 2(x – 4522) x + 4020
200 + X + Y – 5K = 200 x + 4020 – 2(x – 4522) = 2910 ⇒ x = 10154
⇒ X + Y = 5K ∴ Number of males in Chota Hazri = 10154 – 4522
= 5632
A. X + Y = 25 and 3X + 2Y = 100 gives Y
= – 25 (not possible). 26. 3 Let the number of questions answered correctly be x,
B. Not possible as (X + Y) has to be a multiple of 5. that of answered wrongly be y and that of left
C. X + Y = 45 and 3x + 2Y = 100, X = 10 and Y = 35. unattempted be z.
Thus, x + y + z = 50 …(i)
Therefore, only C can be correct.
y z
And x – – = 32
22. 2 Let the total number of chocolates in the box be x. 3 6
Number of chocolates taken by Child 1 The second equation can be written as,
= 100 + 0.1(x – 100) = 90 + 0.1x 6x – 2y – z = 192 …(ii)
Number of chocolates taken by Child 2 Adding the two equations, we get,
= 200 + 0.1(x – (290 + 0.1x) = 171 + 0.09x.
242 + y
As each child receives the same number of chocolates, 7x – y = 242 or x =
7
therefore 90 + 0.1x = 171 + 0.09x
Therefore, the value of x = 8100. Since x and y are both integers, the minimum value of
Number of chocolates received by each child y must be 3.
= 90 + 0.1x = 900.

QA - 17 Page 3
29. 3 Let the student attempted a, b and c number of
27. 4 Given that F(x) = ( x + 2 )( x + 1)( x − 1)( x − 2 )
questions in paper A, B and C respectively.
Putting x = P, we have So, a + b + c = 20 …(i)
and 20a + 25b+ 30c = 520 …(ii)
F (P ) = (P + 2 )(P + 1)(P − 1)(P − 2 )
On multiplying equation (i) by 20 and subtracting it
Now P is in the form 6K ± 1 where K is a positive from equation (ii), we get
5b + 10c = 120
integer.
⇒ b + 2c = 24
F (6K + 1) = (6K + 3 )(6K + 2 )(6K )(6K − 1) So, c b
0 — 24
= (36 )(2K + 1)(3K + 1)(K )(6K − 1) …(i)
1 — 22 
 rejected
F (6K − 1) = (6K + 1)(6K + 2)(6K )(6K − 3 ) 2 — 20
3 — 18 
= 36 (6K + 1)(3K + 1)(K )(3K − 1) …(ii) 4 — 16
Please note that the value of K ≥ 17 and expression F M M
(6K + 1) and F (6K – 1) always bear the factor 10. 10 — 4
Hence, 360 is the correct choice. 11 — 2
12 — 0
28. 4 All the three equations are symmetrical. So, the (We don’t need to write all the values, as c will increase
solutions exist for the conditions x = y = z, x = y = –z, in the step of 1, so b will decrease in the steps of 2)
x = –y = z and –x = y = z. But here a + b + c = 20, so b cannot be more than 16.
Case I: i.e. there are 9 solutions.
When x = y = z Hence, the student can score 520 marks in 9 ways.
then all the three equations give x = y = z = ±1 Now, the values that a can take are 0, 1, 2, 3, 4, 5, 6,
i.e. two solutions 7, 8. So, the maximum number of questions attempted
Case II: by the student in section A is 8.
When x = y = –z
then all the three equations give x = y = –z = ±2 30. 4 There are two equations to be formed 40 m + 50 f
i.e. x = y = –z = 2 ⇒ (x, y, z) = (2, 2, –2)
= 1000
or x = y = –z = –2 ⇒ (x, y, z) = (–2, –2, 2)
250 m + 300 f + 40 × 15 m + 50 × 10 f = A
i.e. two solutions
850 m + 800 f = A
Case III:
m and f are the number of males and females A is
When x = –y = z
amount paid by the employer.
then all the three equations give x = –y = z = ±2
i.e. x = –y = z = 2 ⇒ (x, y, z) = (2, –2, 2) Then the possible values of f = 8, 9, 10, 11, 12
or x = –y = z = –2 ⇒ (x, y, z) = (–2, 2, –2) If f = 8
i.e. two solutions m = 15
Case IV: If f = 9, 10, 11 then m will not be an integer while f = 12
When –x = y = z then m will be 10.
then all the three equations give –x = y = z = ±2 By putting f = 8 and m = 15, A = 18800. When f = 12 and
i.e. –x = y = z = 2 ⇒ (x, y, z) = (–2, 2, 2) m = 10 then A = 18100
or –x = y = z = –2 ⇒ (x, y, z) = (2, –2, –2) Therefore the number of males will be 10.
i.e. two solutions
Hence, total 8 solutions are possible.

Page 4 QA - 17
QA - 18 : Algebra - 2 CEX-Q-0219/20
Answers and Explanations

1 – 2 1 3 2 4 1 5 3 6 2 7 4 8 3 9 1 10 2
11 5 12 1 13 – 14 1 15 2 16 4 17 1 18 4 19 2 20 1
21 2 22 3 23 3 24 2 25 4 26 4 27 3 28 4 29 2 30 3
31 1 32 2

1. Multiplying (x + 3) ( 2x + 5) we get 2x2 + 5x + 6x + 15 4. 1 For equation ax2 + (a + 1)x + 1 = 0 to have equal roots,
= 2x2 +11x + 15. we have
Comparing coefficients we get b = 11. ⇒ (a + 1)2 – 4a = 0 ⇒ a = 1.

2. 1 Quadratic equation having roots (4, 3) is 5. 3 Let roots are α,β.


(x – 4) (x – 3) = 0
⇒ x2 – 7x + 12 = 0 ... (i) 1 1 (α + β)2 − 2αβ
( α + β) = 2
+ 2
=
Quadratic equation having roots (3, 2) is α β (αβ)2
(x – 3) (x – 2) = 0
⇒ x2 – 5x + 6 = 0 ... (ii) ( −a)2 − 2 × (1)
⇒ −a =
Picking the coefficient of x from (i) and the constant (1)2
term from (ii), we get the required equation
x2 – 7x + 6 = 0 ⇒ a2 + a − 2 = 0
⇒ (x – 6) (x – 1) = 0
⇒ a = −2 or 1.
∴ x = 1, 6
Hence, actual roots are (6,1). ⇒ (3) is correct.

Alternative method: 2a + 3
Since constant = [3 × 2] and coefficient of 6. 2 Sum of the roots = α + β = − = −1
x = [–4x – 3x] = –7 a +1
Since quadratic equation is ∴ 2a + 3 = a + 1
x2 – (Sum of roots)x + Product of roots = 0 or a = –2
or x2 – 7x + 6 = 0 3a + 4
Solving the equation, Product of the roots = αβ =
a +1
(x – 6)(x – 1) = 0 or x = (6, 1).
3 × (−2) + 4 − 2
3. 2 ax2 + bx + 1 = 0 = = =2
− 2 +1 −1
For real roots

b2 − 4ac ≥ 0 1 13
x 3
7. 4 Put = y and solving y + = we get y =
∴ b2 − 4a(1) ≥ 0 1− x y 6 2

2
∴ b2 ≥ 4a or
3
For a = 1, 4a = 4, ∴ b = 2, 3, 4
a = 2, 4a = 8, ∴ b = 3, 4 x 9 4
Subsequently, = or
a = 3, 4a = 12, ∴ b = 4 1− x 4 9
a = 4, 4a = 16, ∴ b = 4 9 4
or x = or
∴ Number of equations possible = 7. 13 13

QA - 18 Page 1
8. 3 N= (6 − N), where N = 6 − 6 − 6...∞ i.e. (α + β) =
−b
→ + ve [where α & β are roots]
a
⇒ N2 = 6 – N ⇒ N = –3 or 2 ⇒ N = 2
⇒ b → –ve [ Q a is –ve]
N < 0 cannot be the answer, since any number is by Now, if x = 0, then y = c.
definition positive. From graph, at x = 0, y is +ve.
⇒ c is +ve.
Alternative method:
14. 1
6 = 2.4 approximately.
The answer will be slightly less than that.
So, with this logic all the options got eliminated except
option (3).

α β
1 2−x
9. 1 x= = –1
1 3 − 2x 1
2+
2−x

⇒ 3x − 2x 2 = 2 − x ⇒ 2x 2 − 4x + 2 = 0

⇒ x 2 − 2 2x + 1 = 0
OR
2 2± 8−4 2
⇒x = = 2 ± = 2 ±1
2 2
The value of x is less than 1, hence 2 − 1 is the valid
answer.

10. 2 Given that f(x) = ax2 + bx + c


Also, f(5) = –3f(2) ⇒ f(5) + 3f(2) = 0
⇒ (25a + 5b + c) + 3(4a + 2b + c) = 0 α, α
⇒ 37a + 11b + 4c = 0 …(i)
–1 +1
Also, as 3 is a root of f(x) = 0, thus, f(3) = 0.
Therefore, 9a + 3b + c = 0 …(ii)
Using equation (i) and (ii), we get that a = b
Therefore, c = –12a
⇒ f(x) = a(x2 + x –12) = a(x + 4) (x – 3)
Therefore, the other root of f(x) = 0 is –4.
The graph of the quadratic equation, (in both the above
11. 5 f(x) = a(x2 + x –12) cases) when the coefficient of x2 is positive, is given
Therefore, the value of a + b + c cannot be uniquely above. So, f(1) > 0 and f(–1) > 0
determined.
15. 2 Let f(x) = ax2 + bx + c.
If a > 0, then f(x) will be an upward parabola and f(1)
12. 1 Let f(x) = ax 2 + bx + c must be less than zero, since x = 1 is between the
At x = 1, f(1) = a + b + c = 3 roots of the quadratic.
At x = 0, f(0) = c = 1 If a < 0, then f(x) will be an downward parabola and
The maximum of the function f(x) is attained at f(1) must be greater than zero, since x = 1 is between
b a–2 the roots of the quadratic.
x= – =1= Hence, a(a + b + c) is definitely less than zero.
2a 2a
⇒ a = –2 and b = 4 16. 4 Since f(x + y) = f(x) + f(y) + xy
∴ a(x + y)2 + b(x + y) + c = ax2 + bx + c + ay2 + by +
∴ f(x) = –2x2
+ 4x + 1 c + xy
Therefore, f(10) = –159 ⇒ 2axy + c = xy + 2c
which is possible if c = 0 and a = 1/2
13. Since figure is drawn to scale, and it shows that co-
efficient of x2 is –ve (curve opens downward), the 5
sum of the roots is positive. Q a+b+c=3⇒b= 2

Page 2 QA - 18
x2 5 ⇒ p = –a2/4
So, f(x) = + x
2 2
3a2
So, 2a − a − =0
2
10
1  n(n + 1)(2n + 1)  5  n(n + 1)  4
Now, ∑ f(n) = 2  6 + 2 2 
  
n =1
5a2 4
⇒ − a = 0 ⇒ a = 0,
(n + 8)n(n + 1) 4 5
=
6
−4
Put n = 10 ∴ p = 0,
25
10
18 × 10 × 11  4 4
∑ f(n) = 6
= 330. Sum of all possible real values of p = 0 +  −  = −
 25  25
n =1

17. 1 The maximum value of f(x) must be 0 and this maximum 21. 2 Let us say the roots are α, β, γ, δ and given that sum
value occurs for x = 2. of the roots α + β + γ + δ = 4 and product of roots
αβγδ = 1.
5 Since α, β, γ and δ are positive, the only possible
Let, f(x) = –a(x – 2)2, f(0) = –10, a =
2 values of α, β, γ and δ is α = β = γ = δ = 1, because the
Hence, product of these four roots is maximum.
∴ α = β = γ = δ = 1.
5 5
f(x) = − (x − 2)2 ⇒ f(–2) = − (–2 – 2)2 = –40. ∴ a = αβ + βγ + γδ + αδ + βδ + αγ = 6,
2 2 –b = αβγ + δβγ + αγδ + αβδ
⇒ –b = 4 ⇒ b = –4
18. 4 f(x) = 2x2 + 7x – 5
 7  22. 3 Let the three roots of this cubic equation be α, α and
⇒ f(x) = 2  x 2 + x  − 5 β. We can write:
 2 
(x – α)(x – α)(x – β) = x3 – A.x2 + Bx – C = 0
 7 7 
2
7
2
or x3 – (2α + β)x2 + (α2 + 2αβ)x – β.α2 = x3 – Ax2 +
⇒ f(x) = 2  x 2 + 2 × x +    − 5 − 2 ×   Bx – C
 4  4   4

⇒ A = 2α + β
 7 89
2 B = α2 + 2αβ
⇒ f(x) = 2  x +  − C = α2.β
 4 8
Option (1): If at least one of α and β is an even
 7
2
number, then C will be an even number. If only β is
As  x +  ≥ 0 , even, then B will be an odd number. Hence, (1) is
 4
incorrect.
−89 Option (2): If α is an even number and β is an odd
Minimum value of f(x) =
8 number, then B will be an even number but A will be an
Maximum value of f(x) = +∞ odd number. Hence (2) is incorrect.
Option (3): If A is an even number then β must be an
19. 2 The given equation is (1 – p) x2 + 4x + p = 0 even number. Hence, C must be an even number. Hence
It’s discriminant 16 – 4 (1 – p) p or 16 – 4p (1 – p) (3) is correct.
is positive as 0 < p < 1.
 –4  23. 3 Take an example of f(x) where one of the roots is less
Also, sum of roots   and product of roots than –1 and the other is more than 1 and cross check.
 (1 – p )  If a > 0, f(1)and f(–1) both are negative.
 p  If a < 0, f(1) and f(–1) both are positive.
  are negative and positive in sign respectively.
1– p  So, in either case f(1) x f(–1) > 0.
Therefore, roots of the given equation are real and
(a + b + c)(a – b + c) > 0
negative.
Hence, (2) is the correct choice. (a + c)2 – b2 > 0
(a + c)2 > b2
20. 1 Let ‘a’ be the common root for both the equations. (a + c)2/b2 > 1

Then a must satisfy both the equations,


24. 2 Let roots be α and α 2 .
i.e., 2a2 – a + 3p = 0 and a2 – a – p = 0
⇒ 2a2 – a + 3p = a2 – a – p
2
( )
Given, α + α = −p and (α ) × α = q
2

⇒ a2 + 4p = 0 or α + α 2 = −p and α 3 = q

QA - 18 Page 3
Q a quadratic equation with roots α and β is given by
( )
3
⇒ α + α2 = ( −p)3
x2 – (α + β) x + αβ = 0
or (α )3 + (α 2 )3 + 3(α )2 × (α 2 ) + 3(α )(α 2 )2 = −p3 ⇒ x2 – 3x + 2 = 0
2
or p3 − q(3p − 1) + q2 = 0 ⇒x+ = 3.
x

25. 4 Case I: For x <1, the equation is:


x2 + x = 0. The roots are x = 0 & x = –1. 29. 2 The given equation is x6 + 4x2 = 30.
Case II: x ≥ 1, the equation is: Now, consider the function f(x) = x6 + 4x2.
x2 – x + 2 = 0. There are no real roots. This is a symmetric function about the Y axis as well
So the equation has exactly two roots: x = 0 & as an increasing function as we go from 0 to +∞ or if
x = –1. we go from 0 to −∞ .
Hence, only option (4) is correct. Since this is an increasing function, there will only one
value of x between 1 and 2 for which the value of the
26. 4 Let y = n3 – 7n2 + 11n – 5 function is 30. Similarly, there will be only value of x
At n = 1, y = 0 between – 1 and – 2 for which the value of the function
∴ (n – 1) (n2 – 6n + 5) is 30.
= (n – 1)2 (n – 5) Hence, the number of real roots of the equation
Now (n – 1)2 is always positive. x6 + 4x2 = 30 is 2.
For n < 5, the expression gives a negative quantity.
Therefore, the least value of n will be 6. 30. 3 y2 = x2
Hence, m = 6. 2x2 – 2kx + k2 – 1 = 0
D=0
⇒ 4k2 = 8k2 – 8
2 + 4x − 60
27. 3 (x2 − 5x + 5)x =1 ⇒ 4k2 = 8
Case – I: k2 = 2 ⇒ k = ± 2 with k = + 2 gives the equation
When (x2 – 5x + 5)0 = 1 −b 1
So, x2 + 4x – 60 = 0 = 2x2 – 2 2x + 1 = 0; root is: =+
2a 2
x = – 10, 6
but with k = − 2, the equation is
i.e. two values
1
Case – II: = 2x2 + 2 2x + 1 = 0 root is: −
2 2
When (1)x + 4x − 60 = 1
as this root is –ve, will reject k = − 2.
So, x2 – 5x + 5 = 1
Only answer is: ⇒ k = + 2 only.
x2 – 5x + 4 = 0
x = 1, 4
31. 1 Since, A > 0, f(0) will be less than zero when the
i.e. two values
product of the roots α and β (i.e. αβ) is negative.
Case – III:
(–1)even = 1 For, α = −4, − 3, − 2 or − 1, β can be any of 1, 2 or 3.
So, x2 – 5x + 5 = –1 and x2 + 4x – 60 must be even. Total number of pairs (α, β) = 4 × 3 = 12
Now, x2 – 5x + 5 = –1
x2 – 5x + 6 = 0 For β = −3, − 2 or − 1 , α can be either 1 or 2.
x = 2 or 3 Total number of pairs (α, β) = 3 × 2 = 6
For x = 2
Hence, total number of distinct pairs = 12 + 6 = 18.
x2 + 4x – 60 is even
For x = 3
32. 2 f(–5) = 0 ⇒ 25a – 5b + c = 0 … (i)
x2 + 4x – 60 is odd
f(14) = f(56) ⇒ (14)2a + 14b + c = (56)2a + 56b + c
we cannot take x = 3
⇒ b = –70a
i.e. only 1 value
Putting this value in equation (i), we get
Hence, total 5 values of x are possible.
c = –375a.
So, f(x) = a(x2 – 70x – 375)
28. 4 If α + β = 3 and α3 + β3 = 9
then α = 1 and β = 2, or α = 2 and β = 1 are possible. f(0) a( −375) 5
∴ = = .
f(10) a(102 − 70(10) − 375) 13

Page 4 QA - 18
QA - 19 : Algebra - 3 CEX-Q-0220/20
Answers and Explanations

1 3 2 4 3 1 4 4 5 2 6 – 7 – 8 2 9 1 10 4
11 3 12 1 13 2 14 2 15 3 16 3 17 3 18 1 19 3 20 1
21 – 22 – 23 1 24 3 25 2

1. 3 All the terms in which the power of (–b) will be odd, 6. A. Number of terms = 20 + 3 – 1 C = 22 C
2 – 1 2
will have a negative sign.
Therefore, the power of (–b) can be 1, 3, 5…29. 22 × 21
= = 231
Hence, required number of terms is 15. 2
B. (a + b + c)20 = {(a + b) + c}20
15
 2 3 = 20C0 (a + b)20 × c0 + 20C1 × (a + b)19 × c1 + …
2. 4 x + 
 x = 20C0 (a + b)20 × c0 + 20C1 × {19C0a19b0 + 19C1a18b1
Let the (r + 1)th term be independent of x. + 19C2a17b2 + … } × c + …
From the above expression the coefficient of
r
3 a17b2c is 20C1 × 19C2 i.e. 3420.
∴ Tr +1 = 15Cr (x 2 )15 −r ×  
x C. To find the sum of the coefficients, we need to
substitute a = b = c = 1.
⇒ 2(15 – r) – r = 0
∴ The required sum = 320.
⇒ r = 10
∴ 11th term is independent of x.
7. A. All the powers of x starting from 0 to 40 are
3. 1 Expansion of (x + y)100 + (x – y)100 present in the expansion of (1 + x + x2)20. Hence,
= (100C0 x100 y0 + 100C1 x99y1 + ... + 100C100 x0y100) the number of terms in the expansion is 41.
+ (100C0 x100(–y)0 + 100C1x99(–y)1 + 100C1 x98(–y)2 +
B. {(1 + x) + x2}2 = 20C (1 + x)20 + 20C (1 + x)19x2 +
....+ 100C100x0(–y)100) 0 1
20C (1 + x)18x4+ …
Clearly, all the terms containing (–y), (–y)3 ...(–y)99 will 2
get cancelled. From the above expression, the coefficient of x4
⇒ (x + y)100 + (x – y)100 = 2[100C0 x100 + 100C2 x98y2 ... is 20C × 20C + 20C × 19C + 20C × 18C .
0 4 1 2 2 0
+ 100C100]
All terms in the expansion of (x + y)100 + (x – y)100
8. 2 Here (1 – x + x2)5 = [1 – x (1 – x)]5
contain only non-odd powers of x
∴ Sum of the coefficients = (1 + 1)100 + (1 – 1)100 = 1 −5 C1 × 1× x (1 − x ) +5 C2 × x2 (1 − x )2
= 2100.

−5 C3 × x3 (1 − x ) +5 C4 × x 4 (1 − x ) – ...
3 4
4. 4 To find the sum of the coefficients of all the powers of
20 Only 3rd, 4th and 5th terms will give the terms of x4
x, we put x = 1 in the given expression i.e.  4x – 
1
.
 x In 3rd term, it is 5 C2 × x2 × x2
Hence, the required sum is 320.
In 4th term, it is −5 C3 × x3 ( −3 × x )
5. 2 Given expression is (1 – x2 + x3) (1 + x)10
= (1 – x2 + x3) (1 + 10C1x + 10C2 x2 + 10C3x3 + 10C4x4 + …
+ 10C7x7+ …+ x10) In 5th term, it is 5 C4 × x 4 × 1
∴ Coefficient of x7 = 10C7 – 10C5 + 10C4 So, the coefficient of x4 is
10.9.8 10.9.8.7.6 10.9.8.7 10 + (– 10) × (– 3) + 5 i.e. 45
= − + = 78.
3.2.1 5.4.3.2.1 4.3.2.1

QA - 19 Page 1
9. 1 A. Not always true. To get the maximum value, vz has to be the smallest
Example: If x = 3 & y = 4; a = 0 and b = –10 ( false) negative value and u has to be the highest negative
But if x = 3 & y= 4; a = -10 and b = 0 (true) value. Thus, vz has to be –2 and u has to be –0.5.
B. Not always true. For example, if x = 1 and a = –10
vz −2
(false) but if x = 4 and a = 3 (true) Hence, the maximum value of = = 4.
C. True only if c > 0 u −0.5

14. 2 Amount of rice bought by the first customer


x −1 5
10. 4 < ⇒ 7x – 7 < 15 ⇒ 7x < 22 ⇒ x < 3.333…
3 7  x 1
=  +  kgs
2 2
5 x+4
Again, < ⇒ 25 < 7x + 28 ⇒ –3 < 7x
7 5  x 1 x −1
Amount of rice remaining = x −  +  = kgs
⇒ x > –3/7 2 2 2
Hence, x = 0, 1, 2, 3 Amount of rice bought by the second customer
1  x − 1 1 x + 1
11. 3 From the first and the third equations, = × + = kgs
20X < 500 – 21X 2  2  2 4
Amount of rice remaining
8
⇒ 41X < 500 ⇒ X < 12
41  x − 1  x + 1 x − 3
= −  = 4 kgs
From the second and the third equations,  2   4 
23(X – 1) > 500 – 21X
1  x − 3 1 x +1
Amount of rice remaining = × + = kgs
⇒ 44X > 523 ⇒ X > 11
39 2  4  2 8
44
As per the information given in the question
Since X is an integer, X = 12
x +1 x − 3
= because there is no rice left after the
Alternative method: 8 4
Use answer choices. third customer has bought the rice.
One should pick 2nd option first as it is there in two Therefore, the value of x = 7 kgs.
options. If we try 2nd option first and it gives Hence, option (2) is the correct choice.
contradiction then two options gets eliminated.
So, take X = 11 and Y comes out to be 269 which Alternative method:
gives contradiction. Now, try the third option which is Take the values of x and try. We should take odd
the answer. values of x to get the integral values.
Take x = 5, which contradicts then take x = 7, which
p 10 5 satisfies the condition hence, option (2) is the answer.
12. 1 Maximum value of = =
q 12 6
15. 3 P(P – 3) < 4P – 12
p 3 1 P(P – 3) < 4(P – 3)
Minimum value of = = or P(P – 3) – 4(P – 3) < 0
q 21 7
or (P – 3)(P – 4) < 0
5 1 29 Hence, 3 < P < 4.
So, difference = − = .
6 7 42
Alternative method:
Use answer choices and get the answer.
13. 2 u is always negative. Hence, for us to have a minimum
vz
value of , vz should be positive. Also, for the least 16. 3 2x 2 + 2x − 3 x − 3 < 0
u
value, the numerator has to be the maximum positive 2 x (x + 1) − 3(x + 1) < 0
value and the denominator has to be the smallest (x + 1)(2x − 3 ) < 0
negative value. In other words, vz has to be 2 and u
has to be –0.5. 3
> x > −1
vz 2 2
Hence, the minimum value of = = –4.
u −0.5

Page 2 QA - 19
17. 3 We have 21. Let, y = (x – 1)(x – 2)(x – 3)(x – 4) > 0
x2 – 5x + 4 ≤ 0 The points where y = 0 are x = 1, 2, 3, 4
x2 – 4x – x + 4 ≤ 0 +ve – ve +ve – ve +ve
(x – 4) (x – 1) ≤ 0 1 2 3 4
⇒ 1≤ x≤4
For any value of x in the set ( −∞,1) ∪ (2,3) ∪ (4, ∞ ),
but, x ≥ 3
⇒ value of x the value of y would be +ve.
3≤x≤4
22. (x + 10)(x + 7)(x + 4)(x – 4)(x – 7) < 0
– ve +ve – ve +ve – ve +ve
Alternative method:
Option (2) cannot be the answer as it is given that – 10 –7 –4 4 7
x ≥ 3. for –ve value of the given expression, x = 5, 6, – 5,
Pick x = 5, which contradicts –6 and all the integers less than –10.
So, option (1) and (5) are eliminated So, x can take infinitely many values.
take x = 4, which satisfies hence, option (3) is the
answer. (X + Y)
23. 1 It is very apparent that the answer is (a), as
2
18. 1 x2/3 + x1/3 – 2 ≤ 0 ⇒ x2/3 + 2x1/3 – x1/3 – 2 ≤ 0
is the average of X and Y and should always lie
( )( )
⇒ x1/ 3 − 1 x1/ 3 + 2 ≤ 0 ⇒ – 2 ≤ x1/3 ≤ 1 between X and Y.
⇒–8≤x≤1
Alternative method:
Take any value of X and Y and try with the options.
Alternative method:
e.g. try X = 1 and Y = 0
We can use the options.
take x = –1 which satisfies the equation.
24. 3 Since, nC0 + nC1 + nC2 + … + nCn = 2n
So, option (3) and (4) eliminated.
Now, take x = 8 which contradicts the equation. and nCr = nC(n – r)
So, option (2) got eliminated.
So, nC0 + nC1 + nC2 + … to middle term = 2(n – 1)
Hence, answer is option (1).
∴ 21C0 + 21C1 + 21C2 + … + 21C10 = 220
19. 3 If 9x2 + 3ax + (a + 5) > 0 for all values of x,
⇒ 21C1 + 21C2 + … + 21C10 = (220 – 1) [Q 21C
0 = 1]
then the discriminant of this quadratic expression must
be negative. Again,
∴ (3a)2 – 4(9) (a + 5) < 0 10C + 10C + … + 10C = 210
0 1 10
⇒ a2 – 4a – 20 < 0
⇒ (a – 2)2 < 24
⇒ 10C1 + 10C2 + … + 10C10 = (210 – 1)
So, from the given expression in the question
⇒ – 2.89 < a < 6.89
(220 – 1) – (210 – 1) = (220 – 210).
⇒ a = –2, –1, 0, 1, 2, 3, 4, 5, 6
Hence, there are 9 such integral values.
25. 2 f(x) = (x – 10)(x + 10)(x – 9) (x + 9) ...(x – 1) (x + 1) < 0
+ – + – + – + – + – + – + – + – + – + – +
x 2 − 17x + 72
20. 1 We have ≥0 –10 –9 –8 –7 –6 –5 –4 –3 –2 –1 1 2 3 4 5 6 7 8 9 10
2x 2 + x + 18
p
x= .
or x 2 − 17x + 72 ≥ 0 (Q 2x 2 + x + 18 > 0) q

or (x – 9)(x – 8) ≥ 0 −19 −15 −11 −7 −3 3 7 11 15 19


So, x = , , , , , , , , , .
or x ≥ 9 or x ≤ 8 2 2 2 2 2 2 2 2 2 2

QA - 19 Page 3
QA - 20 : Algebra - 4 CEX-Q-0221/20
Answers and Explanations

1 – 2 1 3 4 4 3 5 4 6 4 7 2 8 4 9 2 10 2
11 – 12 4 13 3 14 2 15 2 16 3 17 1 18 3 19 3 20 2
21 1 22 3 23 3 24 2 25 2 26 3 27 4 28 2 29 4 30 4

1. A. Number of elements in A × B = 5 × 4 = 20 7. 2 f(x).f(y) = f(xy)


B. No one-one function is possible from A to B Given, f(2) = 4
because number of elements in set A is greater We can also write,
than the number of elements in set B. f(2) = f(2 × 1) = f(2) × f(1)
C. Since set A contains 5 elements and set B contains or f(1) × 4 = 4
4 elements, so we will first divide 5 elements of
⇒ f(1) = 1
set A into four groups i.e. 10 ways.
Now we can also write,
Now, these four groups can be paired with the 4
elements of set B in 4! ways.  1  1
f(1) = f  2 ×  = f(2) × f  
So, number of onto function from A to B = 10 × 4!  2 2
= 240.
 1  f(1) 1
⇒ f  = = .
2. 1 In case of F1, a is paired with 1 and m both, which  2  f(2) 4
violates the condition for being a function. So, F1 is not
a function. Whereas in case of F4, c (an element of x)  1 
8. 4 3f ( x + 2 ) + 4f   = 4x
does not belong to any value of set Y. So, it is not a  x + 2
function. F2 and F3 satisfy all the conditions, so option Putting x = z – 2, we get
(1) is correct.
 1
3f ( z ) + 4f   = 4z – 8 …(i)
z
3. 4 If y is real, 9 – x > 0 ⇒ (3 + x) (3 – x) > 0
2
1
Now replacing z with in the above equation, we get
⇒ –3 < x < 3 z
 1 4
3f   + 4f ( z ) = − 8 …(ii)
4. 3 If y is real, 15 − x − 2x ≥ 0 ⇒ x + 2x − 15 ≤ 0
2 2 z z
From (i) and (ii),
⇒ (x + 5)(x − 3) ≤ 0 ⇒ −5 ≤ x ≤ 3
1 16 
f ( z ) =  – 8 – 12z 
7 z 
1 3 1  16 
5. 4 Substituting x =
4
and from the options, we find f ( x + 2) =  – 8 – 12(x + 2)
2 7  (x + 2) 
that the given condition is satisfied. 1 16  52
f (4 ) =
 – 8 – 12 × 4  = − .
 1 3 7 4  7
Q f  x,  = Hence, option (4) is the right choice.
 2 4
So, x + y can be greater than 1 or less than 1 as well. 9. 2 f(1995) = f(15 × 133) = f(3) + f(5) + f(7) + f(19)
We need to check by options. = 1 + 1 + 1 + 1 = 4.

6. 4 f(g(x)) = x ⇒ f(310x – 1) = x 10. 2 Given that g(x + 1) + g(x – 1) = g(x) ... (i)
So, 210(310x – 1) + 1 = x
So, g(x + 2) + g(x) = g(x + 1) ...(ii)
⇒ 210 × 310x – 210 + 1 = x
Adding equations (i) and (ii), we get
1 − 210 1 − 2 −10 g (x + 2) + g (x – 1) = 0
⇒ = x ⇒ x = 10 .
10 10
1− 2 3 3 − 2 −10 ⇒ g(x + 3) + g(x) = 0
⇒ g(x + 3) = –g (x);
So, p = 3.

QA - 20 Page 1
11. The given function can be plotted as 14. 2 Checking with options:
Option (2):
Y = 4 – 3x
6 f3 (–x) = –f2 (–x) = –f1(x) ⇒ f1(x) = –f3 (–x), for all x.

–6
4 15. 2 F(1) + F(2) + … + F(n) = n2 . F(n)

2x
⇒ F(1) + F(2) + … + F(n – 1) = (n2 – 1) . F(n)

Y=
2 Also, F(1) + F(2) + … + F(n – 1) = (n – 1)2 . F(n – 1)
4 Hence we can say (n – 1)2 F(n – 1) = (n2 – 1) . F(n)
3 F(n) (n − 1)2 n − 1
⇒ = 2 =
–6 –4 –2 2 4 6 F(n − 1) n −1 n +1

–2 F (n ) F (n − 1) F (2 )
Now, × × ... ×
F (n − 1) F (n − 2 ) F (1)

–4 (n − 1) (n − 2 ) (n − 3 ) 1
= × × × ... ×
( )
n + 1 n n − 1 3
F(n) 1× 2
–6 ⇒ =
F(1) n(n + 1)
(1) For x = 10 1× 2 2
⇒ F(2005) = × 2006 =
f(x) = 2x – 6 2005 × 2006 2005
⇒ f(10) = 2 × 10 – 6 = 14.
(2) The minimum value of f(x) is at x = 2, which is –2. Alternative method:
(3) The maximum value of the given function can either Putting n = 2, we have F(1) + F(2) = 4 × F(2).
be at x = –5 for f(x) = 4 – 3x or at x = 10 for f(x) 2006
= 2x – 6 i.e. we will check both the possibilities. Thus F(2) =
3
So, at x = –5
Putting n = 3 we have F(1) + F(2) + F(3) = 9 × F(3).
f(–5) = 4 – 3(–5) = 19
and at x = 10 4 1 1
Thus F(3) = × = of 2006
f(10) = 2 × 10 – 6 = 14. 3 8 6
Thus maximum value of f(x) in the given range Putting n = 4 we have F(1) + F(2) + F(3) + F(4)
is 19. 9 1 1
= 16 × F(4). Thus F(4) = × = of 2006
6 15 10
12. 4 If x is an integer, [x] = x. Thus in general we can say that
1 1
∴ will not be a real number when x is an F(n) = × 2006
| x 2 − [x]2 | (sumof all natural numbers till n)
integer. 2 2
Thus F(2005) = × 2006 = .
2005 × 2006 2005
13. 3 f1f2 = f1(x)f1(–x)
16. 3 Assume f(x) = xn + 1
 –x 0 ≤ –x ≤ 1  1 1
 and f   = +1
f1(–x) =  1 –x ≥ 1
 x  xn
0 otherwise

 –x –1 ≤ x ≤ 0
 1
x
(  1
So, f ( x ).f   = xn + 1 
x n ) 
+ 1


=1 x ≤ –1
0
 otherwise (
= 1 + xn + 
x
)
 1
n

+ 1

f1f1(–x) = 0, for all x
 1
= f (x ) + f  
2 x
Similarly, f2f3 = –(f1(–x)) ≠ 0 for some x

f2f4 = f1(–x). f3 (–x) Now f ( x ) = xn + 1 = 9

= –f1(–x) f2 (–x) ⇒ xn = 8 ⇒ n = 3
∴ f ( x ) = x3 + 1
= –f1(–x) f1(x) = 0, for all x
Hence, f(x) = 43 + 1 = 65.

Page 2 QA - 20
Solving the above inequality the only positive integral
17. 1  1 +  2  +  3  = 1× 3.
      value of I that satisfies the equation is I = 3, and the
corresponding value of f for this value of I is 0.2.
 4 +  5 +  6 +  7 +  8 = 2× 5 So, x = 3 + 0.2 = 3.2.
         
Hence, there is only one integral value of x that satisfies
the given equation.
 9  +  10  + L  15  = 3 × 7
     
nth term is n × (2n + 1) = 2n2 + n 21. 1 Here, if x is an integer, then
[x] = x
n(n + 1) (4n + 5)
and Sn = 2 ∑ n2 + ∑ n = So, x = x + 1, which is not possible.
6
Put n = 18 If x is in the form of Integer (I) + Fraction (F)
S18 = 4389 then [x] = I
So, I = I + F + 1 = (I + 1) + F, which is again not possible.
 361 = 19 Hence, no solution.
 
Total sum = 4389 + 19 = 4408. 22. 3 Given that 11[y] + 23{y} = 250 ...(i)
Now 0 ≤ {y} < 1
18. 3 Only the last 3 terms have numbers greater than or So, 0 ≤ 23{y} < 23.
equal to 1 inside the [ ] sign. The last three terms are: Comparing the above with (i) -
227 <11[y]< 250 ...(ii)
 1 38   1 39   1 40 
 4 + 50  +  4 + 50  +  4 + 50  As [y] is always an integer the only possible values of
      [y] in (ii) are 21 and 22. (this is because only multiples
Each of these terms are equal to 1. of 11 between ‘227 and 250’ are 231 and 242)
All previous terms have numbers that lie between 0 250-231 19
and 1 and therefore, are equal to zero. when [y] = 21, {y} = =
23 23
Hence, the sum of the given terms is 3.
19 19
Subsequently y = [y] + {y} = 21 + or y = 21 .
23 23
19. 3 [x]2 + 4{x} = 2x
Also, when [y] = 22,
Let [x] = I, {x} = f,therefore x = I + f 250 – 242 8
{y} = = .
23 23
2
I2 + 4f = 2I + 2f ⇒ 2f = 2I − I
8 8
Subsequently y = [y] + {y} = 22 + or y = 22 .
2
2I − I 23 23
∴0 ≤ <1
2 So, there are exactly two possible solutions for the

Possible values of I = 0, 1 and 2 19 8


equation, y = 21 and y = 22 .
If I = 0, then f = 0: x = 0 23 23
If I = 1, then f = 0.5: x =1.5 and if I = 2, then f = 0: x =2
Therefore, x has three real values. 23. 3 Case I:
x is an integer
20. 2 Let x = [x] + {x} = I + f, where [x] = I denotes the integral Then x2 = [x2] and x = [x], So x = 1, 2, 3, 4, 5 are five
solutions
part of x and {x} = f denotes the fractional part of x.
Case II:
x = 1 + k or 2 + k or 3 + k or 4 + k, where k is a fraction
2 [x ] = 5x + 2
2
0<k<1
for x = 1 + k
⇒ 2I2 = 5I + 5f + 2 12 + k2 + 2k – [1 + k2 + 2k] = k2 (or) 2k = [k2 + 2k], k = 0.5
for x = 1.5, this equation is satisfied
2I2 − 5I − 2 for x = 2 + k
⇒f= 4 + k2 + 4k – [4 + k2 + 4k] = k2 (or) 4k = [k2 + 4k]
5
k = 0.25, 0.5, 0.75
There are 3 solutions x = 2.25, 2.5, 2.75
2I2 − 5I − 2 Similarly, we get for x = 3 + k (5 solutions)
⇒0≤ <1
5 For x = 4 + k (7 solutions)
In all, 5 + (1 + 3 + 5 + 7) = 21 solutions.

QA - 20 Page 3
24. 2 Here, x > 1 and x < 2 is obvious.
1 5 + 3x
3 f 5 (x) = f(f 4 (x)) = =
Now, for x = , expression = 1 + 3 + 4 + 6 = 14  3 + x  8 + 5x
1+ 
2  5 + 3x 
∴ x = 1.5 satisfies for x = 1.5, [3x] = [4.5] = 4
5 1+ 2 1
But as soon as x = , [3x] = 5 it would not satisfy So, f(2) × f2(2) f3(2) f4(2) f5(2) = =
3 (8 + 5 × 2) 6
5 (Note: here we don’t need to write all the term, as
So, x should be less than .
3 denominator of one term is getting cancelled by the
numerator of next term.)
25. 2 We’ll verify the given statements by putting values
of n. 28. 2 Here –r is negative.
So, So, the given expression becomes
Statement I: Put n = 1 So, x1 = 1 f1(–2) + f2(–2) + f3(–2)
= –1 + f(–1) + f3(–2) = –1 + 0 + f(0)
1 1 3 1
So, 2n ++ = + = 2 (true) 1
4 2 2 2 = –1 + =0.
Again, n = 3 So, x3 = 2 1+ 0
Thus option (2) is correct.
1 1 5 1
2n + + = + = 3 (true)
4 2 2 2 29. 4 f(x) = f(x – 1) f(x – 3)
It is always true. f(4) = –1.1 = –1
Similarly, statement II is also true. f(5) = –1.1 = –1
But in case of statement III, if we put n = 1 f(6) = –1.–1 = 1
So, x1 = 1 f(7) = 1.–1 = –1
and xn is the smallest integer greater than f(8) = –1.–1 = 1
f(9) = 1.1 = 1
1 1 3 1 f(10) = 1.–1 = –1
2 × 1+ − = − = 1, which is not true.
4 2 2 2 f(11) = –1.1 = –1
Thus, only two statements are true. f(12) = –1.1 = –1
f(13) = –1.–1 = 1
f(14) = 1.1 = –1
4x f(15) = –1.–1 = 1
26. 3 Since, f(x) = x Here, after every multiple of 7, we are getting two
4 +2
one’s.
4( ) 1− x
2 Since 694 = 7k + 1 and 695 = 7k + 2,
⇒ f(1 – x) = (1− x )
= x So, f(694) + f(695) = 1 + 1 = 2.
4 +2 4 +2
So, f(x) + f(1 – x) = 1
1 + f(x) f(x + a) − 1
30. 4 f(x + a) = ⇒ f(x)
 1   1996  1 − f(x) f(x + a) + 1
⇒ f +f =1
 1997   1997 
x ↔ (x + a)
 2   1995  f(x + 2a) − 1 1 + f(x + a)
f +f = 1 ..... and so on
 1997   1997  f(x + a) =
f(x + 2a) + 1
⇒ f(x + 2a) =
1 − f(x + a)
So, required sum = 998.
1 + f(x)
1+
1 − f(x) −1
1 ⇒ f(x + 2a) = =
1 + f(x) f(x)
27. 4 Since 2 is a non-negative number, so f(x) =
(1 + x) 1−
1 − f(x)
1 (1 + x) Again, x ↔ (x + a)
f 2 (x) = f(f(x)) = =
1 (2 + x) −1 f(x) − 1
1+ =
(1 + x) ⇒ f(x + 3a) =
f(x + a) f(x) + 1
1 (2 + x) Again, x ↔ (x + a)
f 3 (x) = f(f 2 (x)) = =
 1 + x  (3 + 2x) 1 + f(x)
1+ 
 2 + x  −1
f(x + a) − 1 1 − f(x)
f(x + 4a) = =
1 (3 + 2x) f(x + a) + 1 1 + f(x)
f 4 (x) = f(f 3 (x)) = = +1
 2 + x  (5 + 3x) 1 − f(x)
1+ 
 3 + 2x  f(x + 4a) = f(x)
⇒ Period of f(x) is 4a.

Page 4 QA - 20
QA - 21 : Algebra - 5 CEX-Q-0222/20
Answers and Explanations

1 1 2 3 3 4 4 4 5 3 6 3 7 1 8 4 9 4 10 4
11 2 12 – 13 – 14 4 15 4 16 1 17 4 18 4 19 3 20 4
21 – 22 2 23 4 24 2 25 2 26 1 27 4 28 3 29 1 30 1

1. 1 A. |x + y| ≥ |x – y| is not always true. –8 and less than 2 satisfy the given inequality.
Let x = 4 and y = 2, it follows the given condition Therefore, 9 integer values of x satisfy the inequality.
but for x = 4 and y = –2, it doesn’t follow the
condition . 5. 3 Let us solve the question taking the opposite of
B. This condition is not followed whatever is stated
C. |a × x| = a|x| this holds true only for non-negative I. If a is negative, then b has to be negative, since
values of a, so it is not always true. b < a. but |b| can be greater than |a|.
D. This is always true. Hence, (I) is not necessarily true.
Hence, only one statement is true.
II. If b is positive, then a also has to be positive since
a > b. Now, when a > b, and both are positive |a|
2. 3 7 + 3|x| = 13 ⇒ 3|x| = 6 has to be greater than |b|, which contradicts the
⇒ |x| = 2 given condition. Hence, b cannot be positive.
So, x = ±2 So (II) is necessarily true.
III. If c is negative, then b has to be negative since
Alternative method:
b < c, but |b| will be greater than |c|.
Use the options and try. Thus, c cannot be negative and hence (III) is also
true.
3. 4 | x | +2x = −4
If x > 0, then 3x = −4 (not possible) a2
6. 3 To make minimum, a must be minimum and b must
If x < 0, then x = – 4 b2
Now, x + y + z = 5 be maximum. Also, a and b are +ve integers, so from
Therefore, y + z = 9 and 2 | y | + z = 7 the given inequations
If y > 0, then y + z = 9 and 2y + z = 7, but this gives a = 1, 2, 3, 4, 5 and b = 1
a negative value of y, so not possible. a2
If y < 0, then y + z = 9 and z – 2y = 7, but this gives a So, minimum value of = 1.
b2
positive value of y, which is not possible
Hence, there is no solution set for the given system 7. 1 The intervals given in option (2), (3) and (4) contains
of equations.
large negative numbers for which the given inequation
does not hold true.
Alternative method:
x + y + z = (given) So, there must be a lower limit, which is –7/2.
But options (2) and (3) do not satisfy this condition. Hence, option (1) is correct.
Now, the check option (1), which does not satisfy
2|y| + z = 7 8. 4 |3x| + 7|x| ≤ 60
Hence, option (4) is correct For x ≥ 0
3x + 7x ≤ 60
60
4. 4 [x + 3 ] < 5 x≤ ⇒x≤6
10
For x < 0
−5 < [x + 3] < 5
−3x + 7( − x) ≤ 60
If we take x = 2, [x + 3] = 5 and if we take x = –8
⇒ –10x ≤ 60 ⇒ x ≥ –6
[x + 3] = –5.
⇒ value of x is –6 ≤ x ≤ 6
Therefore all integer value of x which are greater than

QA - 21 Page 1
Alternative method: (3) The graph of |x – y| + |x + y| = 10 is given as
The inequation given in the question contains ≤ but
only option (4) contains equals to sign. So, option (4) y=5
is correct.
x = –5 x=5
For questions 9 and 10:
Let |x| = m and |y| = n y = –5
m + n = 7 and m2 + n2 = 25
or m2 + (7 – m)2 = 25 So, required area = 10 × 10 = 100 sq. unit.
Solving above equation we get m = 3 or 4 and hence n = 4 or
14. 4 || 2x − 19 | −7 | < 5
3 or x = ± 3 or ± 4 and y = ±4 or ± 3
−5 < | 2x − 19 | −7 < 5
9. 4 It doesn’t matter what value is assigned to x and y ⇒ 2 < | 2x − 19 | < 12
among ±3 and ±4 because the expression is x3 + y3 ⇒ − 12 < 2x − 19 < −2 or 2 < 2x − 19 < 12
Cleary a total of four values are possible for x3 + y3. ⇒ 7 < 2x < 17 or 21 < 2x < 31
This will happen as (x, y) is retrieved from the following
sets. 7 17 21 31
⇒ <x< or <x<
(– 3, – 4) or (+ 3, + 4) or (– 3, + 4) or (3, – 4) 2 2 2 2
Therefore, there are 10 integer values of x that
10. 4 Here, x and y can be +ve and –ve both. satisfy the inequality i.e. x = 4, 5, 6, 7, 8, 11, 12, 13,
So, value of |x – y|2 cannot be determined uniquely. 14 and 15.

15. 4 Let a2 = b3 = c5 = d6
11. 2 |2x – [x]| = |x + (x – [x])| = 4
Therefore, a = d3, b = d2 and c = d1.2
Here, x – [x] = fractional part of x = {x} Logd(abc) = Logd(d6.2) = 6.2
i.e. |x + {x}| = 4
This will only be satisfied for x = 3.5 and x = 4
1
Hence, only two solutions are possible. 16. 1 Since log b = logb a
a

12. (1) The minimum value will occur at x = 2, which is 3. So, the given expression becomes
(2) The minimum value will occur at 2 ≤ x ≤ 4, which logn 2 + logn 3 + logn 4 + … + logn 43
is 9.
Since logn 1 = 0 and loga p + loga q = loga pq ,
13. (1) The graph of the function |x| + |y| = 10 is given 1
below So, logn 43! =
log43! n

10
17. 4 Since log2x + logx 2 = = log2y + logy2 and x ≠ y ,
(0 , 1 0) 3

10
which means if we convert log2x + logx 2 = into a
3
(– 10 , 0 ) (1 0, 0)
quadratic equation, then x will take two values, same
would be for y. i.e. we have to find the sum of the
(0 , – 10 ) roots.
10
So, log2x + logx 2 =
3

1 1 10
So, area = 4 × × 10 × 10 = 200 sq. unit log2x + =
2 log2 x 3

(2) The given region is same as previous one, but the 3(log2 x)2 − 10(log2 x) + 3 = 0
origin is shifted. So, area remains unchanged,
which is 200 sq. unit −( −10) 10
So, log2x + log2y = = .
3 3

Page 2 QA - 21
n 22. 2 log10 x − log10 x = 2logx 10
18. 4 Q log bn = log b
am m a
 x 
log10   = logx 100
1 1 1 1  x
⇒ + + + ........ +
log3 9 log9 9 log27 9 log n 9
3 log10 100
∴ log10 x =
log10 x
1 1 3 n
= + + + ...... + 1 2
2 1 2 2 ∴ log10 x =
2 log10 x
1 2 3 n n(n + 1)
⇒ + + + ..... + =
∴ (log10 x ) = 4
2 2 2 2 4 2

Alternative method: ∴ log10 x = ±2


Take n = 1 ∴ log10 x = 2 or log10 x = −2
1 1
Then the answer should be = , put in the ∴ 102 = x or 10 −2 = x
log3 9 2
1
options. 1st and 2nd option get eliminated. Now put ∴ x = 100 or x =
100
n = 2 and 3rd option gets eliminated. So, answer is
option (4).
23. 4 logxy = a.logzy = b.logzx = a × b

19. 3 log7 log5 ( x+5 + x =0 ) logxy logxy


a= and b =
For the equation to be 0, x + 5 + x must be equal to logzy logzx

5. i.e. log5 ( x + 5 + x = 1. ) logxy  logxy 


⇒ a×b = × z
 logx 
Putting x = 4, satisfies the equation. logzy

x y  logx   logx 


P = logx   + logy   
k k
20. 4
y x y  y
 logk   logk 
= × {For some base k}
= logx x – logx y + logy y – logy x  logy   logkz 
k
= 2 – logx y – logy x  z  x
Let, t = logx y  logk   logk 
2
1  1  logx 
3
⇒P= 2−t− = − t −
( )
3
 =  ky  = logxy = (ab)3
t  t  log 
 k 
Which can never be positive, out of given option it
can’t assume a value of +1. So (4) is ans. So, ab – a3b3 = 0
or, a × b(1 – a2b2) = 0
21. (1) Let y = 250 ⇒ ab = ±1
log10 y = = 50 × log10 2 = 50 × 0.301 = 15.05 Only option (4) does not satisfy.
y = 1015.05 = 100.05 + 15 = 1015 × 100.05
Since (10)0.05 is any number of the form a.bc…
log x log y logz
So, number of digits in y = 15 + 1 = 16. 24. 2 = = = K , say. Let B is base.
b−c c −a a−b
(2) let y = 2–50
log10 y = –50 log10 2 = –50 × 0.301 Then logB x = K(b − c) ⇒ x = BK(b − c) and
⇒ y = 10–15.05 = 10–16 + 0.95 = 10–16 × 100.95
Here, 100.95 ≈ 8.88… y = BK(c − a) and z = BK(a − b).
10 0.95 Adding, we get
So, 1016 = 0. 0
{ …… 8 …
15 zeroes logB x + logB y + logB z
i.e. 15 zeroes. = {K(b − c) + K(c − a) + K(a − b)} = 0

QA - 21 Page 3
27. 4 When a > 0, b < 0,
or log (xyz) = 0 ⇒ (xyz) = B(0) = 1
B ax2 and –b |x| are non negative for all x,
⇒ xyz = 1 i.e. ax2 – b|x| ≥ 0
Option (1): xyz = 1, is correct. ∴ ax2 – b |x| is minimum at x = 0 when a > 0, b < 0.
Option (2): xa.yb.zc
a b c 28. 3 | 2x − 7 | + | x − 5 | = 14
= BK(b − c)  × BK(c − a)  × BK(a −b )
     
If x ≥ 5, ⇒ 2x − 7 + x − 5 = 14
=B [
K a(b − c) + b(c − a) + c(a −b)]
⇒ 3x = 26
= BK(0) = B0 = 1
Option (2) is correct. 26
∴x =
b+c c +a a +b
3
Option (3): x .y .z

Bk(b + c)(b − c)  × Bk(c + a)(c − a)  × Bk(a +b)(a −b)  7


If ≤ x < 5, ⇒ 2x − 7 + 5 − x = 14
      2
K (b2 − c 2 ) + (c 2 –a2 ) + (a2 –b2 )
=B   ⇒ x = 16
But x lies between 3.5 and 5, hence x cannot be
= B0 = 1
equal to 16
Option (3) is correct.
Option (4): is wrong as the expression evaluates to 1 7
as in (3) and not zero. If x < , ⇒ 7 − 2x + 5 − x = 14
2
In all, options (1), (2) and (3) are correct.

−2
log x logy log z ⇒x=
25. 2 Let = = = k1 3
a2 + b2 + ab b2 + c 2 + bc c 2 + a2 + ac
Therefore, there are two real values of x that satisfy
and x(a–b) .y (b – c) . z(c–a) = k2 the equation.
(a – b )log x + (b – c )log y + (c – a )log z = log k 2
29. 1 We will go by options.
{ (
⇒ k1 (a – b ) a2 + b2 + ab + (b – c ) b2 + c 2 + bc ) ( ) We put x = 1.5, which doesn’t satisfy the given
equation. So option (2) and (3) are rejected.

(
+ (c – a ) c 2 + a2 + ac )} = logk2 Again, we take x = –2, which again do not satisfy the
given equation.

( )
Hence, the only solution set is {–1, 1}, which is
⇒ log k 2 = k1 a3 − b3 + b3 − c 3 + c 3 − a3 option (1).

⇒ logk 2 = 0 ⇒ k 2 = 1
log18 log2 + 2log3
Hence, (2) is the answer. 30. 1 log12 18 = a ⇒ =a⇒ =a
log12 2log2 + log3
26. 1 If 0 < a < 1 and loga x > 0, then 0 < x < 1. Let, log 2 = x and log 3 = y
(x 3 − 1) x + 2y y 1 − 2a
So, 0 < log7 <1 =a⇒ =
(x3 + 1) So,
2x + y x a−2

x3 − 1
⇒1 < <7 4log2 4
x3 + 1 Now, log24 16 = =
log3 + 3log2 log3
+3
log2
x3 − 1
But is always less than 1, for all positive
x3 + 1
4 4(a − 2) 4a − 8
values of x. = = =
 1 − 2a  (a − 5) a−5
So, no solution. 3+ 
 a−2

Page 4 QA - 21
QA - 22 : Algebra - 6 CEX-Q-0223/20
Answers and Explanations
1 – 2 4 3 3 4 2 5 1 6 3 7 2 8 3 9 4 10 3
11 3 12 2 13 1 14 2 15 1 16 3 17 – 18 – 19 1 20 3
21 3 22 3 23 1 24 2 25 1 26 2 27 3 28 – 29 1 30 4

1. (1) The given series is in the form of 3n + 2, Putting there value of a and c in equation (1), we get
where n = 0,1,2...,49
15th term from last is same as (50 – 15 + 1) = 36th b2 = b y / x b y / z = b y / x + y / z
term from start, which is 3 × 35 + 2 = 107. Which is possible only when
(2) For being divisible by 15, a number must be 1 1 2
y y
divisible by 3. Since all the numbers of this series 2= + ⇒ + =
x z x z y
are in the form of 3n + 2, which is not divisible by
2. So, no term would be a multiple of 15. Hence option (1) is the correct choice.
(3) The nth term of the series is (3n + 2)
(4) The sum of the series is 6. 3 The equation of the given line is
50 ax + by + c = 0
[2 × 2 + 49 × 3] = 25 × 151 = 3775.
2 ⇒ ax + ary + ar 2 = 0 ⇒ x + ry + r 2 = 0 … (i)
2
(i) intersects the curve x + 2y = 0 at the points whose
2. 4 The middle term is the average of all the five numbers
ordinates are given by
75 –2y2 + ry + r2 = 0 or,
= = 15
5
So, a1, a2, a3, a4, a5, = 11, 13, 15, 17, 19 ⇒ 2y 2 − ry − r 2 = 0 … (ii)
Now, smallest term of other series = 2 × 19 = 38 ∴ Required sum of y-coordinates = Sum of the roots
So, the sum is 38 + 40 + 42 + 44 + 46 = 42 × 5 = 210.
r
of equation (ii) =
3. 3 Since 452 = 2025 2
So, will have to remove 45 perfect squares of the
sequence. 1 2 3 4 5
⇒ x – 45 = 2015 7. 2 V= − + − + −
So, x = 2060.
2 4 8 16 32
V 1 2 3 4
= − + − + ....
4. 2 Let a be the first term and r be the common ratio. 2 4 8 16 32
Then, a + ar = 12
V 1 1 1 1 1
Also if T1, T2 … be the terms of the GP, then V+ = − + − + − .....
T1 = 2(T2 + T3 + T4 + …) 2 2 4 8 16 32
⇒ a = 2(ar + ar2 + …) 1 1
1 2 2
or, 3V = 2 = 2 =
1
or, V = × =
( 2)
2ar 1 3
⇒a= ⇒ 3r = 1 ⇒ r = 2 1− − 1 3 3 3 9
1− r 3 2

12 12
∴a = = =9 8. 3 The number of elements in successive subsets are
1+ r 1
1+ 1, 4, 9, 16 ....(i.e. 12, 22, 32, 42, 52, ..)
3 Sum of squares of first n natural numbers
9 27 n (n + 1)(2n + 1)
Hence, S = = . =
1 2 6
1−
3 Number of elements in S1 to S36 subsets is
5. 1 Given that
36 (37 )(73 )
b2 = ac …(1) = 6 × 37 × 73 = 16206
ax = by = cz …(2) 6
So, fifth element of subset S37 = 16206 + 5 = 16211.
or a = b y / x and c = b y / z

QA - 22 Page 1
9. 4 The number of terms in the 1st , 2nd, 3rd and 4th brackets
x
are 1, 3, 5, 7 respectively. S(1 − x 2 ) = 2 +
So, the no of terms in the 37th bracket will be 1− x
1 + 36 × 2 = 73. 2–x
⇒S=
1st term in the 2nd bracket is 71, that in the 3rd and 4th are (1 – x)3
71+3, 71+3+5 respectively.
So, the 1 st term of the 37 th bracket will be
71+3+5+L+36 terms = 7(1 + 35) × 36 = 71296 14. 2 Given S = 7 + 13 + 21 + 31 + ......... 9901
6 12 20 30 9900
71296 [ 773 – 1]
Hence, the required sum is:  1   1   1   1 
6 = 1 +  + 1 +  + 1 +  + … +  1 + 
 2 × 3  3 × 4  4 × 5 99 × 100 
10. 3 Let, the infinite G. P. be a, ar, ar2, ...
1 1 1 1 1 1 1 1 
3 = 98 +  − + − + ..... + − + − 
a a 2 3 3 4 98 90 99 100 
So, = 14 ⇒ = 143 = 392 × 7
1− r (1 − r)3
1 1 9849
= 98 + − = .
a3 2 100 100
and = 392 Hence, (2) is the correct option.
1 − r3
a3 a3 1 + r + r2
⇒ = ×7 ⇒ =7 1 1 1 1
(1 − r) 3 3
(1 − r ) (1 − r)2 15. 1 + + +L+
1× 3 × 5 3 × 5 × 7 5 × 7 × 9 95 × 97 × 99
⇒ r = 2 or 1/2
Here, r ≠ 2 (for an infinite G.P.) 1   1   1 1    1 1   1 1  
=   1 −  −  −   +  −  −  −   +
So, r = 1/2 ⇒ a = 7. 8   3 3 5   3 5 5 7 

11. 3 Let, a = 2, b = 4 and c = 8  1 1  1 1  


… +  −  − −  
For ap = bq = cr   95 97   97 99  
2p = 4q = 8r
This is possible only if P = 3 q = 3/2 and r = 1
1  1  1 1   1 6400 800
Thus, options (1), (2) and (4) do not satisfy the =  1 −  −  −  = × = .
condition. 8  3 97 99   8 9603 9603
So, option (3) is the possible answer.
16. 3 Put the values and note that the sum comes out like
12. 2 For x, y and z to be in G.P.
3 5 7
y2 = xz − + + ···
2 log y = log x + log z 2 6 12
i.e. log x. log y and log z are in A.P.
1 1 1 1 1 1 1 1
So, (log x + 1), (log y + 1), (log z + 1) are also in A.P. = 1+ − − + + − − + + ··· = 1
2 2 3 3 4 4 5 5
1 1 1
⇒ , and are in H.P.
1 + log x 1 + log y 1 + logz 17. A. Since tn = 3n(n + 2) – 12n
and n = 1, 2, 3 …
So, to get the smallest term, we put n = 1.
1
13. 1 Coefficient of xn = (n + 1)(n + 4) Thus, t1 = 3(1 + 2) – 12 = –3.
2 B. tn = 3n(n + 2) – 12n = 3n2 + 6n – 12n
tn = 3n2 – 6n
S = 2 + 5x + 9x 2 + 14x3 + .... So, their sum upto n terms is given by
xS = 2x + 5x 2 + ..... n
2
S(1 – x) = 2 + 3x + 4x + 5x + .... 3 ∑ t n = 3 Σ n2 − 6 Σ n
n =1

Let, S1 = S(1– x) ⇒ S1 = 2 + 3x + 4x + ... 2


 n(2n + 1)(n + 1)   n(n + 1) 
= 3  − 6 2 
 6   
xS1 = 2x + 3x 2 + ...
n(n + 1)(2n − 5)
Sn =
S1(1 – x) = 2 + x + x 2 + .... 2

x 10 × 11 × 15
S1(1 − x) = 2 + Now, S10 = = 825.
1− x 2

Page 2 QA - 22
18. tn = n(n + 1) (n + 2) = n3 + 3n2 + 2n 21. 3 a1 = 1, an + 1 – 3an + 2 = 4n
3
⇒ Σ t n = Σ n + 3 Σ n + 2Σ n 2 an+1 = 3an + 4n –2
when n = 1 then a2 = 3 + 4 – 2 = 5
2 when n = 2 then a3 = 3 × 5 + 4 × 2 – 2 = 21
 n(n + 1)   n(n + 1)(2n + 1)  2n(n + 1)
Sn =   + 3 + from the options, we get an idea that a n can be
 2   6  2
expressed in a combination of some power of 3 &
We put n = 10, to get
S10 = (55)2 + 3(385) + 110 = 3025 + 1155 + 110 = 4290. some multiple of 100.
(1) 399 – 200; tells us that an could be: 3n – 1 – 2 × n; but
19. 1 it does not fit a1 or a2 or a3.
(2) 399 + 200; tells us that an could be : 3n – 1 + 2 × n;
1 1 1 1 1 1
S = 1+ + + 1+ 2 + 2 + … + 1+ + again, not valid for a1, a2 etc.
12 22 2 3 20072 20082 (3) 3100 – 200; tells 3n – 2n: valid for all a1, a2, a3.
(4) 3100 + 200; tells 3n + 2n: again not valid.
1 1 n4 + 2n3 + 3n2 + 2n + 1 So, (3) is the correct answer.
Tn = 1 + 2
+ 2
=
n (n + 1) n2 (n + 1)2
22. 3 According to the statements given in the question we
n2 + n + 1 1 can write here
= = 1+
n2 + n n2 + n S = a1 + a2 + a3 + ... + an … (i)
Since a2, a3, a4 ... an–1 are the arithmetic means of first
2007 2007
1 1  1
S= ∑ Tn = 2007 + ∑  n − n + 1 = 2008 − 2008 . three consecutive terms, next three consecutive
n =1 n =1 terms(starting with a2) etc.
∴ S2 = a2 + a3 + a4 + ... + an–1 … (ii)
Alternative method:
a1 + a2 a2 + a3 a + an
Also, S1 = + + ... n −1
1 1 1 1 1 1 2 2 2
S = 1+ + + 1+ + + ... + 1 + +
12 22 22 32 (n − 1)2 n2
1
we can solve the question by writing the generated ⇒ S1 = [a + 2(a2 + a3 + ...an −1) + an ] … (iii)
form of options 2 1

1 Note that in equation (i) if a1 and an are excluded, then


As, option (1) as n − the rest of the series is nothing but S2.
n
Therefore, we can write the series as
1 S = a1 + S2 + an ⇒ a1 + an = S – S2 … (iv)
Option (2) is (n − 1) −
(n − 1)
1 1
1 S1 = [a + 2S2 + an ] ⇒ (a 1 + an ) = S1 − S2 ...(v)
Option (3) is (n − 1) − 2 1 2
n
From (iv) and (v), we have
1
Option (4) is n − S = 2S1 – S2 = 2 × 1375 – 690 = 2060.
(n − 1)

1 1 3 3(1) − 2 1
Now, first term is + = 1+ 23. 1 V1 = 2 , V2 = 1, V3 = =
12 22 2 2 2
and putting n = 2, only option (1) gives the answer.
 3Vn−1 − Vn− 2 
20. 3 Given a1 = 81.33 and a2 = –19 as we have Vn = 
 2 
Also
ai = ai – 1 – ai – 2, for j ≥ 3
⇒ a3 = a2 – a1 = –100.33
a4 = a3 – a2 = –81.33 V4 =
(2)
3 1 −1
=
1
, V5 = 4 2()
3 1 −1 1
=
2 4 2 8
a5 = a4 – a3 = 19
a6 = a5 – a4 = +100.33
a7 = a6 – a5 = +81.33 1
Thus, we observe that Vn =
a8 = a7 – a6 = –19 2n − 2
Clearly onwards there is a cycle of 6 and the sum of
terms in every such cycle = 0. Therefore, when we 1 1 1
add a1, a2, a3... upto a6002, we will eventually be left with ∴ V9 = = =
9−2 7 128
2 2
a1 + a2 only i.e. 81.33 – 19 = 62.33.

QA - 22 Page 3
24. 2 Since the given terms are in A.P. 28. log 3 + log 3 − log 3 + log 3 – ......
32 33 34 35
(1− x) x
So, 2 × log4 (2 + 1) = log2 (5 × 2 + 1) + 1
1 1 1 1 1 1 1 1
log2 (2(1− x)
+ 1) = log2 (5 × 2x + 1) + log2 2 = − + − + − + − + ........
1 2 3 4 5 6 7 8
⇒ 21− x + 1 = (5 × 2x + 1)2
1 1 1 1
= + + + + .........
2 1× 2 3 × 4 5 × 6 7 × 8
⇒ + 1 = 10 × 2x + 2
2x
≈ 0.5 + 0.083 + 0.03 + 0.017 + 0.01 + ....... ≈ 0.7.
Let 2x = y
2
So, + 1 = 10 × y + 2 2 2 2 2
y 8  12   16   20 
29.1 Sn =   +   +   +   …
2 + y = 10y2 + 2y  
5  5  5  5 
2 −1
⇒y= or
5 2 42  2
Sn = 2 + 32 + 42 + 52 + L112 
25  
2  1
i.e. 2x = x
 as 2 ≠ − 
5 2 16  2
Sn = 1 + 22 + 32 + 42 + L112 − 1
x = 1 – log2 5. 25  

25. 1 a + d, a + 4d, a + 8d
16  11 × 12 × 23  16 16
(a + 4d)2 = (a + d) (a + 8d) = × − 1 = × 505 = × 101
a2 + 16d2 + 8ad = a2 + 9ad + 8d2 25  6  25 5
8d2 = ad
⇒ m = 101.
a = 8d, d ≠ 0
a + 4d 12d 4 30. 4 If we assume (1, 7) as the first block, (1, 1, 7, 7) as the
r= = =
a+d 9d 3 second block and so on till mth block, then T5001 will be in
the mth block.
26. 2 Let the sequence be x, y, 3, z, 18 There are 2 terms in the first block, 4 terms in the
So, 3z = 18 ⇒ z = 6
second block and so on.
and 3y = z ⇒ 3y = 6 ⇒ y = 2
and xy = 3 ⇒ x = 3/2. ⇒ 2 + 4 + 6 + 8 + ..... + 2m ≥ 5001
⇒ 2(1 + 2 + ..... + m) ≥ 5001

4 9 16 25 2m (m + 1)
27. 3 S = 1+ + + + + ....... ⇒ ≥ 5001 ⇒ m(m + 1) ≥ 5001
7 7 2 73 7 4 2
S 1 4 9 16 25 Least possible value of m that satisfy the above is m
= + + + + + ........
7 7 7 2 7 3 7 4 75 = 71.
⇒ T5001 lies in 71st block.
6S 3 5 7 9 11
⇒ = 1 + + 2 + 3 + 4 + 5 + .......... Total number of terms in the first 70 blocks = 70 × 71
7 7 7 7 7 7 = 4970 terms.
6S 1 3 5 7 9 11 In the 71st block, there will be 71 one’s and 71 seven’s.
⇒ = + + + + + + .......... Therefore, the sum of first 5001 terms of the series
49 7 72 73 74 75 76
= (8 + 16 +.... 70 terms) + 1 × 31
6S 6S 2 2 2 2 = 8(1 + 2 + ..... + 70) + 31
⇒ − = 1 + + 2 + 3 + 4 + ..........
7 49 7 7 7 7
70 × 71
= 8× + 31 = 19911
  2
62 2 1  4
⇒ × S = 1+  = All the terms from T4971 to T5041 are ‘1’ and all the terms
72 7 1 1  3
− from T5042 to T5112 are ‘7’.
 7 
⇒ T5040 + T5042 + T5044 + T5046 = 1 + 7 + 7 + 7 = 22.
72 4 49
⇒ S= × =
62 3 27

Page 4 QA - 22
QA - 23 : Algebra - 7 CEX-Q-0224/20
Answers and Explanations

1 3 2 2 3 4 4 3 5 2 6 – 7 – 8 – 9 4 10 2
11 4 12 4 13 3 14 3 15 4 16 5 17 3 18 2 19 4 20 3

21 4 22 5 23 3 24 4 25 2 26 3 27 4 28 2 29 3 30 2

1. 3 Here, from options. ⇒ y 2 + (x − 5) y + x 2 − 5x + 3 = 0


if a = 0
As it is given that y is a real number, the discriminant
⇒ x2 + 1 = 0 (not possible)
for above equation must be greater than or equal to
if a = 1 ⇒ x2 + x + 1 = 0
zero.
−1 ± 3 i Hence, (x − 5)2 − 4(x 2 − 5x + 3) ≥ 0
⇒ x= , which is imaginary (not possible)
2
if a = 2 ⇒ 3x 2 − 10x − 13 ≤ 0 ⇒ 3x2 − 13x + 3x − 13 ≤ 0
⇒ x2 + 2x + 1 = 0 ⇒ (x + 1)2 = 0  13 
⇒ x ∈  −1, 
So, roots are – 1 and – 1.  3
This is a possible minimum value.
13
Largest value that x can have is .
Alternative method: 3
For real roots,
Alternative method:
a2 – 4 ≥ 0
x+y+z=5
⇒ (a – 2) (a + 2) ≥ 0
On squaring both sides, we get
a ∈ ( −∞,–2] ∪ [2, ∞ )
Since, x2 + ax + 1 = x2 – (α + β) x + αβ = 0 x= 19 − (y 2 + z2 )
and α and β are negative Here, we have to maximise x and y ≠ z ≠ 0 because
So, a must be positive ⇒ amin. = 2.
xy + yz + xz = 3.
2. 2 As we have to maximise one integer. So, x must be less than 19 , only option (3) represents
So, we will assume the numbers are 1, 2, 3, 4....,14, a a number less than 19 .
(where a is maximum)
⇒ 1 + 2 + 3 + ....+ 14 + a = 15 × 13 5. 2 Let, it is divided into two parts x and y.
⇒ 7 × 15 + a = 15 × 13 So, x + y = 15
⇒ a = 15(13 – 7) = 15 × 6 = 90.
x x y y y 1
+ + + +
2 2 3 3 3 ≥  x y  5
2 3
3. 4 To get the minimum value, we will take first 9 natural ⇒
5  22 × 3 3 
numbers and 10th number (M) will be maximum such  
S to maximise R. H. S.
that could be maximized.
M
x y
S = 1 + 2 + 3 + ... + 9 + M =
S = 49 + M 2 3
Here, if we take M = 100 ⇒ x:y=2:3
S 149 Hence, x = 6 and y = 9.
⇒ = = 1.49
M 100
if we further increase the value of M, 6. a + b + c = 10
We know that
S
→1 AM(a, b, c) ≥ GM(a, b, c)
M
S a+b+c
So, minimum value of ≈ 1. ⇒ ≥ (abc)1/3
M 3

4. 3 xy + yz + zx = 3 10 1000
⇒ ≥ (abc)1/3 ⇒ ≥ abc
⇒ xy + (y + x)z = 3 3 27
⇒ xy + (y + x)(5 − x − y) = 3
1000
⇒ x 2 + y 2 + xy − 5x − 5y + 3 = 0 Hence, the maximum possible value of abc is .
27

QA - 23 Page 1
7. a + b + c = 10
(a – 1) + (b – 2) + (c – 5) = 2 p q r s
12. 4 , , and are all positive numbers and for
We know that q r s p
AM(a – 1, b – 2, c – 5) ≥ GM(a – 1, b – 2, c – 5) positive numbers.

(a − 1) + (b − 2) + (c − 5) ≥ A.M. ≥ G.M.
⇒ {(a − 1) × (b − 2) × (c − 5)}1/3
3 p q r s 1
+ + +  p q r s 4
2 So, q r s p
≥ ⋅ ⋅ ⋅ 
⇒ ≥ {(a − 1) × (b − 2) × (c − 5)}
1/3
3 4 q r s p
8
⇒ ≥ {(a − 1) × (b − 2) × (c − 5)} or,
p q r s
+ + + ≥4
27 q r s p
Hence, the maximum possible value of (a – 1)(b – 2)
8
(c – 5) is . 13. 3 Since a, b, c and d are positive real numbers.
27
1
8. Since, a + b + c = 6 So, a + b + c + d ≥ (abcd) 4
4
and AM ≥ GM
⇒ a+b+c+d≥4
a a a b b 1 Again,
+ + + + +c  3 2 5
3 3 3 2 2 ab c
So, ≥ 3 2  (a + 1) + (b + 1) + (c + 1) + (d + 1)
6 3 2  ≥
4
1
[(a + 1)(b + 1)(c + 1)(d + 1)]1/ 4
6  a 3b 2 c  5
⇒ ≥ 3 2 
6  3 2  a+b+c +d
⇒ + 1 ≥ [(a + 1)(b + 1)(c + 1)(d + 1)]1/ 4
4
⇒ 3 3 22 ≥ a 3b 2 c
Since, we have to minimise RHS, so LHS must be
So, maximum value of given expression is 108.
minimum for minimum value of LHS
a+b+c+d=4
9. 4 x + y + z + w = 29
(x – 1) + (y + 3) + (z – 1) + (w – 2) = 28 4
+ 1 = [(a + 1)(b + 1)(c + 1)(d + 1)]
1/ 4
Applying A.M. ≥ G.M., ⇒
4
(x − 1) + (y + 3) + (z − 1) + (w − 2)
16 = [(a + 1)(b + 1)(c + 1)(d + 1)]min.
4
1
≥ ((x − 1)(y + 3)(z − 1)(w − 2) ) 4 Alternative method:
Maximum value of (x – 1) (y + 3) (z – 1) (w – 2) For minimum value a = b = c = d = 1
So, (1 + a) (1 + b) (1 + c) (1 + d) = 2 × 2 × 2 × 2 = 16.
4
 28 
=   = 2401 . 14. 3 Since x + y = 1
 4 
So, for the minimum value of given expression x = y =
1/2
10. 2 The possible sets for distinct integers (x, y, z) are
We put this value on the given expression.
(1, 2, 8); (1, 3, 7); (1, 4, 6); (2, 3, 6); (2, 4, 5).
So, the maximum value of (xyz + xy + yz + zx) occurs 2 2
1  1  2 2
for x = 2, y = 4 and z = 5. So,  + 2  +  + 2  = (2.5) + (2.5) = 12.5
Hence, the maximum value of given expression 2  2 
= 2 × 4 × 5 + 2 × 4 + 4 × 5 + 5 × 2 = 78.
15. 4 f(x) is a quadratic function and the coefficient of x2 is
11. 4 2a + b + 3c + 2d = a + a + b + c + c + c + d + d 1 + 1 – 1 – 1 + 1 + 1 = 2 > 0.
Applying A.M. ≥ G. M., Therefore, the graph of f(x) is an upward-pointing
1 parabola, and the minimum value of the function is
a+a+b+c +c +c +d+d
8
≥ a2bc 3d2 8 ( ) attained at its vertex. The given function is also
symmetric about x = 6.
2a + b + 3c + 2d ≥ 16 So, the vertex must be at x = 6.
Therefore,
For 2a + b + 3c + 2d to be minimum possible f(6) = 1 + 1 – 4 – 4 + 9 + 9 = 2(1 + 9 – 4) = 12
a=b=c=d=2 Therefore the minimum value of the function is 12.
⇒ a + 2b + 3c + 4d = 20.

Page 2 QA - 23
16. 5 f(x) = max (2x + 1, 3 – 4x) 20. 3 Given pqr = 1
So, the two equations are y = 2x + 1 and y = 3 – 4x
1 1
Their point of intersection would be ⇒ pq = and = qr
2x + 1 = 3 – 4x r p
1
⇒ 6x = 2 ⇒ x = 1 1 1
3 + +
y 1 + p + q−1 1 + q + r −1 1 + r + p −1
y=2 x+ 1
q r 1
= + +
(0 ,3) 1 + q + pq 1 + qr + r 1 + r + qr

(1 /3,5/3 ) qr r 1 1 + r + qr
= + + = = 1.
1 + qr + r 1 + qr + r 1 + r + qr 1 + r + qr
y=3 –4 x
x Alternative method:
(-1 /2,0) (0 ,3/4) Putting x = y = z = 1, we get
1 1 1
+ +
1 + p + q−1 1 + q + r −1 1 + r + p −1
1
So when x ≤ , then f(x)max = 3 − 4x
3 1 1 1 1 1 1
= + + = + + = 1.
1 1+ 1+ 1 1+ 1+ 1 1+ 1+ 1 3 3 3
and when x ≥ , then f(x)max = 2x + 1
3
1
Hence, the minimum of this will be at x = 1
3 21. 4 x − 1− x =
5
5
i.e. y = .
3 x + 1 − x = a, say
On squaring and adding, we get
17. 3 f(x) = min(x 2 − 4,4 − x 2 )
1
Case 1: x2 – 4 > 4 – x2 ⇒ 2x2 > 8 2(x + 1 – x) = + a2
25
x < – 2 or x > 2
Case 2: x2 – 4 < 4 – x2 1 49
⇒ a2 = 2 − =
–2<x<2 25 25
Therefore,
7
4 − x 2
− 3 ≤ x ≤ −2 or 2 ≤ x ≤ 3  ⇒a= (as a > 0)
f(x) =  5
2

 x − 4
−2≤ x≤2 
⇒ maximum value of the function is 0.
1 4 1
22. 5 + = , n < 60
m n 12
18. 2 It is given that abc = 4.
We know, (a + b + c) will be minimum when a, b, c are
1 1 4 n – 48
constants and a = b = c. ⇒ = – =
As a, b, c are in arithmetic progression, m 12 n 12n
⇒ a + b + c = 3b
12n
This will be minimum when a = b = c, b3 = 4. ⇒m=
n – 48
⇒ b = 41/ 3 = 22 / 3 Positive integral values of m for odd integral values of
n are for n = 49, 51 and 57.
19. 4 T = {3, 11, 19, 27, ...., 451, 459, 467}
Therefore, there are 3 integral pairs of values of m
Here, we will be getting 470 if we add first term and
last term, or second term and second last term and so and n that satisfy the given equation.
on.
23. 3 Given equation is x + y = xy
(467 − 3)
Now, number of elements in the set = + 1 = 59. ⇒ xy – x – y + 1 = 1
8 ⇒ (x – 1)(y – 1) = 1
59 + 1
So, = 30 elements are such that they will not ∴ x – 1 = 1 and y − 1 = 1 or x − 1 = –1 & y – 1 = –1
2
give their sum 470. Clearly, (0, 0) and (2, 2) are the only pairs that will
satisfy the equation.

QA - 23 Page 3
24. 4 a+d+b
y ≥ (adb)1/ 3 … (iv)
3
Multiplying (i), (ii), (iii) and (iv), we get
(a + b + c)(b + c + d)(c + d + a)(a + d + b) ≥ 34 (abcd)
3 S1 × S2 × S3 × S4 ≥ 405 .
f (x ) = x+3 (0,3) 3
5 f (x ) = x–3
2 27. 4 Simplifying the expression, we get
(x – 7)2 + (y – 3)2 = 82 + 12 or 72 + 42
To get the maximum value of 3x + 4y we find that x will
(−5,0) (2,0) (20 /3.0 ) be equal to
x y 3x + 4y
15 4 61
The graph of f(x) is shown in bold in the above figure.
We can break-up the definition of f(x) in different 14 7 70
regions of the x-axis. 8 11 68
 3 11 10 73
− 2 x + 3 x≤0
We are neglecting negative solutions since we are looking
 for largest value. Hence, the maximum value is 73.
3 20
f (x ) =  x + 3 0<x≤
5 3
28. 2 Number of samosas = 200 + 20n,
 3 20
 2x−3 x≥ n is a natural number.
 3 Price per samosa = Rs. (2 – 0.1n)
3 15 27 Revenue = (200 + 20n)(2 – 0.1n) = 400 + 20n – 2n2
Now; f(7) = (7) – 3 = and f(–7) = For maxima, n = 5
2 2 2
405 ⇒ Maximum revenue will be at (200 + 20 × 5)
⇒ f (7 ) × f ( −7 ) = .
4 = 300 samosas

Alternative method: For questions 29 and 30:


 −21 −25  To maximise Shabnam’s return we need to evaluate all the
 27  27
f ( −7 ) = max  −3, + 3  = max  ,2  = . given options in the question number 7. Assume Shabnam had
 2 5   2  2 one rupee to invest. Let the return be denoted by ‘r’.
 21 21   15 36  15 Consider the option (30% in option A, 32% in option B
f (7 ) = max  −3, + 3  = max  ,  = . and 38% in option C): If the stock market rises, then
 2 5   2 5  2
r = 0.1 × 0.3 + 5 × 0.32 – 2.5 × 0.38 = 0.653
27 15 405
∴ f ( −7 ) × f (7 ) = × = . If the stock market falls, then
2 2 4 r = 0.1 × 0.3 – 3 × 0.32 + 2 × 0.38 = – 0.197
Consider option (100% in option A): This will give a return
25. 2 Vy2 – 6Vy + 5V + 1 > 0 of 0.1%.
⇒ V(y2 – 6y + 5) + 1 > 0 Consider option (36% in option B and 64% in option C):
⇒ V(y2 – 6y + 9 – 4) + 1 > 0 If the stock market rises, then
⇒ V(y – 3)2 – 4V > (–1) r = 5 × 0.36 – 2.5 × 0.64 = 0.2
If the stock market falls, then
⇒ V × Y > (–1), where Y = (y – 3)2 – 4 r = – 3 × 0.36 + 2 × 0.64 = 0.2
It should be noted that Consider option (64% in option B and 36% in option C):
−4 ≤ Y < ∞ for any real value of y. If the stock market rises, then
Objective is to find the minimum value of V. If we take r = 5 × 0.64 – 2.5 × 0.36 = 2.1
any negative value of V and multiply it with a very high If the stock market falls, then
positive value of Y, it would be lesser than –1(means r = – 3 × 0.64 + 2 × 0.36 = –1.2
even more negative). Thus, V must be non-negative. Consider option (1/3 in each of the 3 options): If the
for V = 0, V × Y = 0 for any Y. stock market rises, then
Thus, V = 0 is the minimum value. r = 0.1 × 0.33 + 5 × 0.33 – 2.5 × 0.33 = 0.858
If the stock market falls, then
26. 3 S1, S2, S3 and S4 are (a + b + c), (b + c + d), (a + d + c) r = 0.1 × 0.33 – 3 × 0.33 + 2 × 0.33 = –0.297
and (a + b + d). As all the sums are positive, using the We can see that only in option (36% in option B and 64% in
AM ≥ GM property, we get, option C), Shabnam gets an assured return of 0.2%
a+b+c irrespective of the behaviour of the stock market. So right
≥ (abc)1/ 3 … (i)
3 option for questions number 13 is (0.20%) and question number
b+c+d 14 is (36% in option B and 64% in option C).
≥ (bcd)1/ 3 … (ii)
3 29. 3
c +d+a
≥ (cda)1/ 3 … (iii) 30. 2
3

Page 4 QA - 23
QA - 24 : Algebra - 8 CEX-Q-0225/20
Answers and Explanations

1 – 2 – 3 1 4 – 5 2 6 3 7 4 8 3 9 3 10 3
11 4 12 2 13 1 14 2 15 – 16 – 17 4 18 1 19 1 20 –
21 3 22 4 23 2 24 2 25 2 26 3 27 3 28 3 29 4 30 2

1. (1) Distance between A and B Possible points of P is (0, – 2), (2, 0), (–2, 2)
2 2 Note here, the points must be laying on the line parallel
= {3 − ( −5)} + (7 − 13) = 64 + 36 = 10 unit.
to either RQ or SR.
(2) The mid-point of AB
3 + ( −5) 3. 1 Equation of the line passing through (0, 5) and (3, 0) is
X= = −1
2 5x + 3y – 15 = 0
The region below the line can be represented by
7 + 13
Y= = 10 which is (–1, 10) 5x + 3y − 15 ≤ 0 …(i)
2
The line passes through (0, 0) and (5, 5) is y = x
1:2 For the region above that y ≥ x ... (ii)
A B
(3) (3 , 7 ) m c n (– 5, 1 3) The right-hand side region of Y-axis is represented by
(x, y) x≥0 ... (iii)
Hence, total shaded region is represented by the
1( −5) + 2(3) 1
X= = combination of equations (i), (ii) and (iii).
1+ 2 3

1(13) + 2(7) 4. (1) Required equation of line is


Y= =9
1+ 2 y – 1 = – 3 (x – 2)
y – 1 = – 3x + 6
So, C ≡  1 ,9  3x + y = 7
3 
1
(4) Equation of line through AB (2) Slope of line perpendicular to y = – 3x + 7 is .
3
13 − 7
y–7= (x − 3) So, required equation of line is
−5 − 3
– 8y + 56 = 6x – 18 1
y–2= (x – 1)
6x + 8y = 74. 3
3y – 6 = x – 1
3y = x + 5.
2. 4
5. 2 It is given that
3
x + 2y = 9 ...(i)
2 3x + 5y = 5 ...(ii)
S 1 3 × (i) – (ii) gives
R
y = 22
–4 –3 –2 –1 1 2 3 4 Therefore, equation (i) gives x = 9 – 44 = –35.
–1
Q Since the lines are concurrent the above values of x
–2
and y should satisfy ax + by = 1
–3 ∴ 22b – 35a = 1
–4

QA - 24 Page 1
9. 3 By careful observation we can say that the given
6. 3 Y triangle is a right-angled triangle as shown below.
(2 , 4 ) (– 2k, k) (k, k)
L’
X
O
(3 , – 2)
(k, – 3k)
L
1
Area of this triangle = × 3k × 4k = 24
2
y 2 − y1 ⇒k=2
The equation of the line L is y − y1 = (x − x1) So, the orthocentre would be the vertex at which
x 2 − x1
right-angle is form, which is (k, k) ≡ (2, 2).

−2 − 4 10. 3 Let ≡ (–a, –b), B ≡ (0, 0), C ≡ (a, b) and D ≡ (a2, ab)
i.e. y − 4 = (x − 2)
3−2
Here, AB = a2 + b2
or y – 4 = –6 (x –2) or 6x + y = 16
So, the co-ordinate of the point where the line L cuts and BC = a2 + b2

8  AC = 2 a2 + b2
the X-axis is  , 0  .
3  So, A, B and C must be collinear as AB + BC = AC.
⇒ Option (1), (2) and (4) are incorrect.
New line L’ is perpendicular to the line L. Hence, option (3) is correct.
So, m1 m2 = –1, Here m1 = –6
Alternative method:
1 The points A(–a –b), B(0,0) and C(a, b) lie on the same
∴m2 = (where m1 is slope of line L and m2 is the
6 line.
slope of the line perpendicular to L)
a>b
1
The equation of new line L’ is y = x + c. b (a ,b)
6
–a
8  (0 , 0 ) a
But this is passing through  , 0  .
3 
(– a, b)
–b
1 8 −4
So, 0 = × + c ⇒ c =
6 3 9
Hence, the equation of new line is 3x – 18y = 8. Thus, Option (1), (2) and (4) are incorrect.
Hence, option (3) is correct.
7. 4 3x + 4y + 3 = 0 …(i) 11. 4 The co-ordinates of point P can be given as,
3x + 4y + 12 = 0 …(ii) P(x, 5x + 3)
Put y = 0 in equation (i),
∴ x = –1 P (x, 5 x + 3) Y = 5x + 3
∴ (–1, 0) lies on the line (i) perpendicular distance
between lines (i) and (ii) is same as perpendicular
distance of (–1, 0) from the line (ii)
M
3( −1) + 4(0) + 12 9 9
∴ Required distance = = = .
32 + 42 25 5
Q (3 , – 2)

Since, M is the mid-point of PQ, the co-ordinates of M


8. 3 The vertex of rhombus must lie on the line is given by
x – y + 1 = 0 or 7x – y – 5 = 0.
x+3 5x + 3 − 2
Only option (3) satisfy the given condition. X= and Y =
2 2

Page 2 QA - 24
C are getting counted twice, so, number of points
5 1
Thus, Y = x+ laying on the triangle = 41 + 41 + 41 – 3 = 120.
2 2
This is the required equation of line. (ii) Equation of line BC is given by
2y + 3x = 120
2 2 2y = 3 (40 – x)
12. 2 The circle x + y + 2 gx + 2 fy + c = 0 has x intercept
2 C (0 , 6 0)
if f 2 > c and has y intercept if g > c .
To keep the origin outside the circle c > 0.
All the conditions are satisfied by option (2).

13. 1 Here, distance between the centres

= (5 − 2)2 + (3 − ( −1)2 = 5 , which is less than the


radius of smaller circle.
So, shortest distance between the circles = radius of A (0 , 0 ) B (4 0, 0)
larger circle – (radius of smaller circle + distance
between their centres) = 12 – (6 + 5) = 1 unit.

14. 2
Here, R.H.S. is a multiple of 3.
So, LHS must be a multiple of 3
Let y = 3b
Similarly, x must be a multiple of 2
let x = 2a
r
So, a + b = 20
Now, number of whole number solutions of this
equation gives the number of integral points laying
on the line BC, which is 21.
The shortest distance will be the difference between Number of integral points laying on the line AB =
the distance between the centre and the line and the 40 (except B)
radius. Number of integral points laying on the line AC =
Centre of the circle is at O(1, 0) and r = 3 units. 60 (except C)
But point A is getting counted twice.
[8 (1) + 6 (0 ) − 48] So, required points = 21 + 40 + 60 – 1 = 120.
⇒r +d=
82 + 62
(2) In side the triangle
⇒ r + d = 4 units (i) We will count the number of points laying on the
⇒ d = 4 – r = 1 unit. lines y = 1, y = 2, y = 3, y = 39 and also inside the
triangle.
15. (1)
(i) On the triangle y
C (0 , 4 0)
y
C (0 , 4 0)

x
A (0 , 0 ) B (4 0, 0 )
x
A (0 , 0 ) B (4 0, 0 )

Number of points on y = 1 is 38
Number of points on y = 2 is 37 and so on
No. of lattice points on AB = 41 So, number of point inside the triangle ABC
No. of lattice points on AC = 41 = 1 + 2 + 3 + ...... + 37 + 38
Now, the equation of line BC is given by x + y = 40.
38
So, number of whole number solutions of this = [1 + 38] = 19 × 39 = 741.
equation will give the number of points laying on 2
the line BC, which is 41. But the vertices A, B and

QA - 24 Page 3
(ii) To solve this, will draw a rectangle as shown So, the possible graph can be drawn as
below.
D C x+y=–3
C (0 , 6 0) D (4 0, 6 0)

A B
x+y=3
A (0, 0) B (40 , 6 0)
x–y=5 x–y=–5
Now, total number of points inside the rectangle Since, distance between two parallel lines is given
= (41 – 2) (61 – 2) by
= 39 × 59 = 2301
Out of these points, 19 lie on the diagonal BC C1 − C2
(inside the rectangle, as solved earlier)
a2 + b2
Thus, number of points inside
2301 − 19 5 − ( −5) 10 3 − ( −3) 6
∆ABC = = 1141. So, AB = = and BC = =
2 2 2 2 2

10 6
16. (A) The graph of the given functions is given as Thus, area of ABCD = × = 30 sq. unit.
2 2
|x|=4 ⇒ x=±4
|y|=3 ⇒ y=±3
17. 4 Given curves can be drawn as
y
y = |x|
y=3
A C (3 ,3)
( −3 ,3 )
x
x=–4 x=4 (3 ,0)
y=–3
(−3 ,0)
O
So, required area = 8 × 6 = 48 sq unit.
( −3 , −3 ) D (3 , −3 )
B
(B) |x – 3| = 4 ⇒ x = 7 and x = – 1,
|y + 3| = 5 ⇒ y = 2 and y = – 8 y = −|x|

y The area generated by the three curves is the sum of


y=2 area of the triangles OAB and OCD.

1 
x = 2  × (6 ) × 3  = 18 square units
x=–1 2 
y=–8
x=7 18. 1 |x + 2y| + |x – 2y| = 6
On replacing x by –x and y by –y in the above equation,
So, required area = 8 × 10 = 80 sq. unit. we get the same equation. This implies that the curve
is symmetric in the quadrants of X-Y plane
(C) Here, In the first quadrant where x, y > 0
|x + y| = 3 ⇒ x+y=±3 |x + 2y| + |x – 2y| = 6
and |x – y| = 5 ⇒ x–y=±5
(x + 2y) + (–x + 2y) = 6 ; 2y > x
The first two lines are parallel to each other, and =
other two lines are parallel to each other.  (x + 2y) + (x − 2y) = 6 ; 2y < x

 3
y = ; 2y > x
= 2
 x = 3 ; 2y < x

Page 4 QA - 24
Y
III.
x– 2 y =0
( )
0, 3
2

X
(3 , 0 )

IV.

3 9
Area in the first quadrant = × 3 = square unit
2 2
Total area of |x + 2y| + |x – 2y| = 6

9
= 4× = 18 square unit.
2

19. 1 Y 21. 3 Y L 1: 5x – 6 y =1
(0 ,2.4)
(0 ,1)

X (a, a 2)
(1 .5,0) (3 ,0)
X
O
Area of the shaded region L 2 : x + 2y = 3

1 12 3
= 3 × − 1×  = 2.85 sq. unit. L 3 : 2x + 3 y = 1
2 5 2
For line L1: 2x + 3y = 1, origin O and point (a, a2) are on
the opposite sides of the line, so their signs are
opposite.
L1(0, 0) : 2.0 + 3.0 – 1 = –1 < 0
20. I.
∴ L1(a, a2) = 2a + 3a2 – 1 > 0
⇒ 3a2 + 3a – a – 1 > 0
⇒ (a + 1) (3a – 1) > 0
1
⇒ a < –1 or a > ... (i)
3
For line L2: x + 2y = 3, L2(0, 0) has same sign as
L2(a, a2)
L2(0, 0) : 0 + 2.0 – 3 = –3 < 0
∴ L2(a, a2) : a + 2a2 – 3 < 0
⇒ 2a2 + 3a – a – 3 < 0
II. ⇒ (2a + 3) (a – 1) < 0

3
⇒− < a <1 ... (ii)
2
For line L3: 5x – 6y = 1, L3(0, 0) has same sign as
L3(a, a2)
L3(0, 0): 5 × 0 – 3 × 0 – 1 = –1 < 0
∴ L3(a, a2) = 5a – 6a2 – 1 < 0
⇒ 6a2 – 5a + 1 > 0
⇒ (3a – 1) (2a – 1) > 0

QA - 24 Page 5
26. 3 Here, any integral value of y, length OP will be integer.
1 1
⇒a< or a > ... (iii)
3 2 P (0 , y)
From (i), (ii) and (iii), we get

 3  1 
a ∈  − , − 1 ∪  , 1 . B (7 , 0 )
 2  2 
A (– 10 , 0 ) O (0 , 0 )

22. 4 In the graph, y-axis is horizontal and x-axis is vertical


(opposite of convention). The graph cuts y-axis
(x = 0) at two places. Hence, it will be quadratic equation
for y. At y = 0 (x-axis), the value of x is near to – 20
(from the graph). Now, AP = 10 2 + y 2 ⇒ AP2 = 102 + y2
Possible equation is x = 2y2 + 3y – 19 and BP = 72 + y 2 ⇒ BP2 = 72 + y2
Here, by careful observation, we can say that we
23. 2 The negative slope of a straight line passing through need to find the Pythagorean triplet, which satisfy the
−30 above two equations.
(0, 30) is given by , where p is a positive number. So, y must be 24.
p
Which will give AP = 26 and BP = 25
The number of possible values of p will be the number Thus, possible values of P are (0, 24) and (24, 0)
of possible straight lines. i.e. two points are possible.
So, p = 1, 2, 3, 5, 6, 10, 15, 30
i.e. 8 straight lines are possible. 27. 3 We are to find the solutions for f(f(x)) = 15.
From the graph, f(4) = 15 and f(12) = 15.
24. 2 For the curves to intersect, log10 x = x–1 The required solutions will be those values of x for
which f(x) = 4 and f(x) = 12.
1 From the graph, the value of function f(x) is 4 at four
Thus, log10 x = or x x = 10
x different values of x, i.e. –8, 1, 7.5 and 10.
The value of the function f(x) is 12 at three different
This is possible for only one value of x such that
points, i.e. 3, 5 and 11.
2 < x < 3.
Hence, the given equation has 7 solutions.

25. 2 kx – 5y + 7 = 0 ⇒ 5y = kx + 7
28. 3 Y
and k2x – 5y + 1 = 0 ⇒ 5y = k2x + 1
On equating these two equations, we get
kx + 7 = k2x + 1 Q (a , 6 )
⇒ x(k2 – k) = 6 = 1× 6 = 2 × 3 = 3 × 2 = 6 × 1
When x(k2 – k) = 1 × 6 M (a/2, 5)
P (0 , 4 )
then x = 1 and k2 – k = 6 ⇒ k = 3, – 2
kx + 7 k + 7
Since, y = =
5 5
S (b , 0 )
⇒ y = 2 or 1
X
When x(k2 – k) = 2 × 3
1 ± 13
then x = 2 and k2 – k = 3 ⇒ k = , k is not an The co-ordinates of the point M are
2
integer. a +0 6 + 4 a 
 ,  or  ,5 
when x(k2 – k) = 3 × 2  2 2  2 
then x = 3 and k2 – k = 2 ⇒ k = 2, –1 6–4 2
Slope of the straight line PQ is =
3k + 7 a–0 a
Since, y = ⇒ y is not integer for any value of k. a
5 ⇒ Slope of the straight line MS = –
2
When x(k2 – k) = 6 × 1 ⇒ x = 6 and k2 – k = 1
y–5 a
1± 5 ⇒ Equation of the straight line MS is =–
a 2
⇒k= , which is not an integer. x–
2 2
Thus, only two values of k are possible. 2
a a
⇒y+ x =5+
2 4

Page 6 QA - 24
As point S(b, 0) lies on it, we must have; 30. 2 (PL + PN) will be minimum if P lies on LN and (PM + PO)
will be minimum if P lies on OM.
2
a a  10 a  ⇒ PL + PM + PN + PO will be minimum if P is the point
0+ × (b ) = 5 + ⇒ b= + 
2 4  a 2 of intersection of the diagonals of quadrilateral LMNO.
For a = 2, –2, 10 and –10; b is an integer.
LN = (–5 – 0)2 + (0 – 5)2 = 5 2
29. 4 Co-ordinates of points after halving their abscissas
and doubling their ordinates infinite times: MO = (1 – (–1)2 + (–1 – 5)2 = 2 10
A (0, High)
⇒ PL + PM + PN + PO = LN + MO = 5 2 + 2 10
B (0, 0)
C (0, High)
D (0, High)
E (0, High)
Now, the closest point would be the points which had
the value of their ordinates closest at the beginning.
Hence, the points A and C should be closest.

QA - 24 Page 7
QA - 25 : Geometry - 1 CEX-Q-0226/20
Answers and Explanations

1 2 2 2 3 4 4 3 5 100 6 2 7 3 8 3 9 3 10 2
11 4 12 3 13 81 14 3 15 1 16 3 17 45 18 1 19 3 20 1
21 2 22 2 23 10 24 4 25 4

1. 2 By using basic proportionality theorem,


4. 3 E
AB = DE ⇒ 2 = 1.5 or EF = 3 cm
BC EF 4 EF

2. 2 ∠CBF = ∠EBF = 50.


C
Also, because AC || EF, ∠CBF = ∠BFE = 50. 96°
In triangle EBF, ∠BEF = 180 – 50 – 50 = 80. x
180 – 2y
1
Hence ∠GEF = of 80 = 40°. x y y
2 A
D B

C Using exterior angle theorem

B
∠A + ∠B = 96°
50 i.e. x + y = 96° …(i)
50 G Also x + (180° – 2y) + 96° = 180°
A ∴ x – 2y + 96° = 0
50 ⇒ x – 2y = –96° …(ii)
a a F Solving (i) and (ii),
y = 64° and x = 32°
E ∴ ∠DBC = y = 64o

5. 100 If DE = BE, then ∠DBE = ∠BDE = a (say)


3. 4 D If EF = EC, then ∠EFC = ∠ECF = b (say)
Hence, ∠DEB = 180° – 2a and ∠FEC = 180° – 2b.
10 0° Also, a + b + 40° = 180° or a + b = 140°.
55 ° ∠DEF = 180° – ∠DEB – ∠FEC
H 25 ° = 180° – 180° + 2a – 180° + 2b
35 ° E
55 ° = 2(a + b) – 180° = 100°.
35 °
90 ° A
90 °
G 40
F

Since HD || GE, D
a
∠ DEG = 80° F
∴ ∠ HEG = 80° – 25° = 55° b
Also HE || GF,
a b
∴∠FGE = 55°.
B E C

QA - 25 Page 1
6. 2 A P
Trian gle T 2
C 10 10

Q R
S
D 16
B
The above figure represents the case given in the Since both are isosceles triangles, therefore
question. If AC = CD and also ∠ ABD = 90°, we can perpendicular dropped from the vertex to the unequal
say that C is the centre of the circle passing through side also bisects the side.
A, B and D with AD as the diameter. So, the radius BD = 6 ⇒ AD = 8 ( Q ∆ABD → right angled triangle)
must be AC, CD or BC. As given that BC = AB, we can
conclude that AB = BC = CA. 1
So, area of ∆ABC = × 12 × 8 = 48 sq. unit
Hence, ∆ ABC is an equilateral triangle and ∠DAB 2
must be 60°. Now, PS = 6
1
7. 3 Possible sides of different triangles can be as follows: ⇒ Area of ∆PQR = × 6 × 16 = 48 sq. units
2
(2, 3, 4), (2, 4, 5), (2, 5, 6), (3, 4, 5) and (3, 4, 6)
Area of ∆T1 Area of ∆ ABC 48
8. 3 Let AC = 2x, where x is any natural number. = = =1
Area of ∆T2 Area of ∆ PQR 48
∴ AB = 1 AC = 2x −1; but AB is a perfect square
2 11. 4 Let the angle between sides c and b be α .
⇒ x – 1 is even ⇒ x is odd.
1
Sum of two sides of a ∆ is greater than the third side So, area of triangle is × c × b × sin α = 1 .
⇒ AB + AC > BC ⇒ 3AB > 295 ⇒ AB > 98.33
2
⇒ 2x – 1 > 98.33 …(i) 2
Also, AC – AB < BC Here b = .
c sin α
⇒ 2x – 2x – 1 < 295
⇒ 2x – 1 < 295 To find minimum value of b, c and sinα must be
…(ii)
The only satisfying value for equations (i) and (ii) is maximum. sinα can be maximum 1 and c can maximum
x=9
be equal to b. So, b2 = 2 and b = 2 is the minimum
∴ AB = 28 = 256 and AC = 29 = 512
value of b.
∴ Perimeter = 256 + 512 + 295 = 1063.

9. 3 Let the three sides are 3a, 4a and 5a. 12. 3 For maximum possible area, the figure should be as
symmetrical as possible. In this case, it should be an
1 1 isosceles triangle;
Area of triangle = × side1 × altitude1 = × side2
2 2
A
1
× altitude2 = × side3 × altitude3
2
15 0 15 0
Hence, 3a × altitude1 = 4a × altitude2 = 5a × altitude3.
1 1 1
altitude1 : altitude2 : altitude3 = : : = 20 : 15 : 12.
3 4 5
B C
D w all
A 20 0
10. 2 Trian gle T 1
BD = 100 ml ( Q⊥ from A to BC also bisects BC)
10 10
In ∆ ABD

B C AD = 1502 − 1002 = 50 5
D
12 1
Area of ∆ ABC = × 50 5 × 200 = 5000 5
2

Page 2 QA - 25
13. 81 Let area of ABC is a square units. P
2a
The area of BPC will be .
3

2 2a 4a A B
Similarly, area of BPQ will be of = .
3 3 9
2 4a 8a
Area of BRQ will be of = .
3 9 27 D C
Hence ∠APC = 60 – 15 = 45.
2 8a 16a
Area of BRS will be of = .
3 27 81 18. 1 A
16a
a must be a multiple of 81 for to be an integer.
81
So, minimum area of ABC is 81 square units. D E

14. 3 A

B C
F
1
E Here, DE || BC and DE = BC = 12 cm
2
∴ Sum of perimeters of triangles so formed
B D C  1 1 
= 72 + 36 + 18 + … ∞ = 72 1 + + + K ∞ 
AF : FB = 1 : 2  2 4 
BD : DC = 7 : 4
ar(∆ABE) : ar(∆AEC) = 7 : 4 1 1
= 72 × = 72 × = 144 cm
Let, ar(∆ABE) = 7a 1 1
1−
⇒ ar (∆AEC) = 4a 2 2
Again, AF : FB = 1 : 2
So, ar(∆AFE) : ar(∆FBE) = 1 : 2 19. 3 Let P, B and H be the perpendicular, base and
hypotenuse of the triangle.
7a
⇒ ar(∆AFE) = [ Q ar(∆ABE) = 7a] 1
3
So, P2 + B2 = H2. Also PB = 40 or PB = 80.
Thus, ar(∆AFE) : ar(∆AEC) = 7a/3 : 4a 2
⇒ FE : EC = 7 : 12 P + B + H = 40 i.e. P + B = 40 - H.
Squaring both sides, we get
15. 1 If p2, q2 and r2 are three sides of a triangle then sum of P2 + B2 + 2PB = 1600 + H2 – 80H.
any two sides of the triangle must be greater than Putting the value of PB, we get H = 18 cm.
third i.e. p2 + q2 > r2. This too is a property of acute
angles triangle with sides p, q and r. Hence answer 20. 1 B
(1).
4 5°

 3
16. 3 The height of an equilateral triangle of side 'a' = a 20
2 1 20 °
1 5°
A
 3 24 3 0° 6 0°
C
It is given that, a = 12 ⇒ a = = 8 3 cm
2 3
Therefore, area of the equilateral triangle
D
= 3 2 3
( )
2
a = × 8 3 = 48 3 cm2 . In ∆ ADC, let AD = x
4 4
AD
17. 45 AP = PB = BC. So, in triangle PBC, ∠PBC = 150. Also tan 60° =
CD
Also, ∠BPC = ∠PCB = 15.
AD x
CD = =
3 3

QA - 25 Page 3
∆ ABD is an isosceles right angle triangle, so AD = BD Case II: Let x be the largest side.
X < 15 + 8 i.e. x < 23.

( ) For an obtuse angled triangle, x2 > 152 + 82 i.e. x > 17.


x 20 3
⇒x= + 20, x = = 10 3 + 3
3 3 −1 Possible values of x are 18, 19, 20, 21 and 22.
Hence, total possible values of x are 10.
21. 2 Initial position of the ladder: If x is the height at
which top of the ladder is above the ground. 24. 4 A

25 m
x
5 cm h 5 cm

7m
9 0°
⇒ x = 252 − 72 = 24 meters B C
D
Final position of the ladder: Let ladder moves away b /2 b /2
y meters from the wall.
1
Area ( ∆ABC) = × b × h = 12 sq.cm
2
⇒ b × h = 24 …(i)
y Also, in right triangle ABD,
(24 – ) 25 m
2 2
 b
(5)2 =   + h2
 2

(y + 7) m ⇒ b2 + 4h2 = 100 …(ii)

  48 
2
 2 2 ⇒ b2 +   = 100 …(iii)
(25) = ( y + 7)2 +  24 − 2 
y  b
∴ b = 6 and 8, are the roots of the above equation.
⇒ y = 8 meters
25. 4 Let area of triangle BHC be a square units. Hence
22. 2 Let ∠A = a. area of BHA will be 3a. Therefore, area of triangle
In triangle ABC, AB = BC. Hence, ∠BCA = a. ACD and ADE will be 8a and 12a respectively.
∠CBD = ∠A + ∠BCA = 2a. Let area of triangle AHG is x square units. Area of
(Exterior angle is sum of two remote interior angles). CHD will be 2a and hence area of HDG will be 6a – x.
In triangle BCD, BC = CD. BH Area of ABH Area of DBH
Hence ∠BDC = ∠CBD = 2a. Now = = .
HG Area of AHG Area of DHG
∠DCE = ∠A + ∠BDC = a + 2a = 3a.
In triangle DCE, ∠DEC = ∠DCE = 3a. Hence,
3a
=
3a
. So, x = 3a.
In triangle AGF, AG = GF. Hence, ∠A = ∠AFG = a. x 6a − x
∠FGC = ∠A + ∠AFG = 2a. Now, area of triangle DGE will also be 6a.
In triangle GFE, EF = FG. Hence, ∠FGE = ∠FEG = 2a. Now assuming the area of triangle AGF as y, area of
∠EFD = ∠A + ∠FEA = a + 2a = 3a. GFE will be 6a – y.
In triangle FED, ∠EFD = ∠FDE = 3a. BG Area of ABG Area of BGE
Now, in triangle ADE, Now, = = .
∠A + ∠ADE + ∠AED = a + 3a + 3a = 180. GF Area of AGF Area of EGF
6a 12a
180 Hence, = .
Hence a = = 26 (approx.) y 6a − y
7
Solving this we get y = 2a. So, required ratio is 3 : 2.
23. 10 Case I: Let 15 be the largest side.
15 < 8 + x i.e. x > 7.
For an obtuse angled triangle,
152 > 82 + x2 i.e. x2 < 161.
Possible values of x are 8, 9, 10, 11 and 12.

Page 4 QA - 25
QA - 26 : Geometry - 2 CEX-Q-0227/20
Answers and Explanations

1 1 2 2 3 1 4 4 5 1 6 3 7 2 8 2 9 3 10 2
11 4 12 2 13 1 14 2 15 2 16 1 17 2 18 2 19 4 20 4
21 1 22 1 23 2 24 115° 25 2

1. 1 D Here ∠ACB = θ + [180° – (2θ + α )] = 180° – (θ + α )


So here we can say that triangle BCD and triangle
ABC will be similar. ∆BCD ~ ∆BAC
A Hence, from the property of similar triangles
3m
E
AB 12 AC 12
2m = ⇒ AB = 16 ; = ⇒ AC = 8
h 12 9 6 9
B C ∴ AD = 7
F
5m SADC = 8 + 7 + 6 = 21 ; SBDC = 27
In ∆BCD, we have
21 7
BF h Hence, r = =
= ...(i) 27 9
BC 3
In ∆CAB, we have
4. 4 A D
CF h
= …(ii) θ
CB 2 E 9 0°θ
Adding (i) and (ii), we get
BF CF h h BF + FC h h
+ = + ⇒ = +
BC CB 3 2 BC 3 2 θ 9 0°θ
Now, BF + FC = BC B C
h h If AC = 13 and AB & BC are integers then their possible
Hence, + = 1 ⇒ h = 1.2 meters.
3 2 values are 5 & 12 or 12 & 5 respectively.
Taking one of the cases

P 2x Q
2. 2 Case I: AB = 5, BC = 12
In ∆ABC
O if ∠A = θ then ∠ACB = 90 – θ
In ∆BEC, ∠EBC = θ ( Q ∠BEC = 90)
S R In ∆BCD, ∠D = 90 – θ ( Q ∠DBC = θ)
3x
If PQ = 2x, then RS = 3x. Therefore, ∆ABC & ∆BCD are similar
2
Area of ∆POQ PQ2  2x 
2 Area of ∆ABC  5  25
4 = =
= =  =
Area of ∆SOR SR2  3x  9 Area of ∆BCD  12  144

(Since triangles are similar.) Case II:


AB = 12
BC = 5
3. 1 A 2
Areaof ∆ABC  12  144
=  = .
θ Areaof ∆BCD  5  25
D
5. 1 ∆ADB ~ ∆BDC
9
AD BD
6 ∴ =
BD DC
α θ
B C ⇒ BD2 = AD × DC = 8 × 2 ⇒ BD2 = 16 ⇒ BD = 4 cm.
12

QA - 26 Page 1
6. 3 Triangle ABC and Triangle QRC are similar. If ABC is P
a right angle triangle of Pythagoras triplet (3 : 4 : 5) 10. 2
then QRC has to be the same. Let us take QR as 3k
so BR will also be 3k as its a square. And RC will be
17 10
8 A F B
4k. So BC = 7k. Thus 7k = 8 & k = .
7
8 24
So side of square 3k will be 3 × = . R
7 7 Q D E C
2 21
 24  576
So, area =   = . PE is an altitude from vertex P to side QR.
 7  49
Let PE = h, so
7. 2 Triangles PQR, PAB & DCR are similar. Triangle PQR 1
× 21 × h = S(S − a)(S − b)(S − c)
is a Pythagoras triplet of 3 : 4 : 5 and so do the other 2
triangles. So, DR will be 5k = 6 and k = 1.2. So, = 24(24 − 17)(24 − 10)(24 − 21)
CR = 4k, CR = 4.8. Similarly, PA = 2.4. Thus,
AC = PR – PA – CR = 25 – 4.8 – 2.4 = 17.8. 1
⇒ × 21× h = 84 ⇒ h=8
2
M Let a be side of square ABCD.
8. 2 P Q
∆PAB ~ ∆PQR
D PF AB 8−a a
= ⇒ =
A PE QR 8 21
T N 168
⇒ 29a = 168 ⇒ a =
C 29
4 × 168
B Perimeter of square ABCD = 4a =
R 29
S O = 23.17 (approx.)
In PQRS, PM = OR and PM || OR,
∴ PMRO is a parallelogram 11. 4 We can use the formula for the circum radius of a
In ∆SRC triangle:
a×b×c
SO SB R=
= 4 × (Area of the triangle)
OR BC
∴ SB = BC = x (say) a×b×c a×c
or R = =
1  2 × AD
x 4 ×  × b × AD 
Also AD = DQ = x ⇒ CN = 2 
2
In ∆SRN 17.5 × 9
= = 26.25 cm.
SN2 = 64 + 16 2×3
2
 5x  8 A P B
= 80 ⇒ x = . 12. 2
 2  5

9. 3 PQ || AC θ
CQ AP 4
∴ = =
QB PB 3
QD || PC
K
PD CQ 4
∴ = =
DB QB 3 -θ
90 90

θ

PD 4
As = D Q C
DB 3 ∠PKQ = 90° (Angle in a semi-circle)
4 PQ QD PD PQ2
∴ PD = PB ∆PQD ~ ∆PKQ ; = = ⇒ PK =
7 PK KQ PQ PD

AP AP 7 AP 7 4 PQ = 4 units, PD = PQ2 + DQ2 = 42 + 22


∴ = = × = × = 7 : 3.
PD 4 PB 4 PB 4 3 4×4 8
7 = 20 = 2 5 ⇒ PK = 2 5 = 5

Page 2 QA - 26
13. 1 A 17. 2 In ∆PQR, PX divides the triangle into two equal areas.
1
Area of triangle OQX = (Area of triangle PQR)
6
D 1
Area of triangle QRY = of PQR
9 0-θ 2
C 1 1
9 0-θ
Area of triangle XZR = (RQY) = (Area of triangle
θ O 2 8
PQR)
B
 1 1
Area of quadrilateral OYZX =  −  of PQR
2 8
BD DO BO 3
∆BDO ~ ∆BAC ; = = Area = of PQR
AB AC BC 8
⇒ DO = 4, BC = 9, BO = BC – OC = 9 – 4 = 5 Ratio of QOX
=4:5
Ratio of quadrilateral OYZX
⇒ BD = BO2 − DO2 = 52 − 42 = 9 = 3
3 4 5 36 27 18. 2
⇒ = = ⇒ AC = , AB = A
AB AC 9 5 5
1 1 36 27
Hence, area of ∆ABC = × AC × AB = × × O
2 2 5 5 P Q
= 19.44. S

14. 2 P B C
R
Area of ∆PQS = 80, as AR is the median
Area of ∆PQR = 240 as RO is the median which
Y divides centroid in ratio 2 : 1
T Area of ∆ABC = 960

19. 4 Since AB||CD, AC||DE and FI || BE the ∆ABC ~ ∆AFG ~


Q R
X ∆DHI ~ ∆DCE.
Let PX & QY meet at T. Thus T will be centroid and 4
PT:TX = 2 : 1. ⇒ Ratio of sides of ∆AFG and ∆ABC =
7
So, PT = 12.
16
1 1 ∴ Ratio of area of ∆AFG and ∆ABC =
49
Now Area of Triangle PQY = × QY × PT = × 21
2 2 ⇒ area of quadrilateral FGCB = 33 square unit
× 12 = 126. Area of ∆PQR = 2 × ∆PQY = 252 Since JL = KM, so let y = 3 units.
36
⇒ Ratio of area of ∆DHI and ∆DCE =
15. 2 X 81
ar( ∆DHI) 36
⇒ =
ar(WFGCB) 33
= 12 : 11
P
R 20. 4 No right angled triangle can have an altitude to the
hypotenuse more than half the length of the
hypotenuse. For example any triangle formed in a
Y Z semicircle with diameter as the base is always a right
Q
As YP is angle bisector it will divide XZ in ratio of triangle. The maximum height it can have is the radius.
corresponding sides i.e. XY : YZ = 6 : 5.
21. 1 A
So, XP = 6 and PZ = 5. As XQ is angle bisector for
triangle XYZ, then XR will also be the angle bisector
of XYP, so it will divide YP in ratio of corresponding
sides i.e. XY : XP = 2 : 1
E F
r=
(a + b − c )
16. 1 For a right angle triangle in-radius and G
2
c
circumradius R = .
2
Using these relation we get 2(R + r) = b + a B C
2
As AB = AC, GB = GC = 2 cm.

QA - 26 Page 3
1 where ‘s’ is the semi-perimeter of the triangle.
GE = GF = cm [Since ‘G’ is the centroid of the r 1
PR
2 Also, R = =4 3 ⇒ =
1 5 2 R 3 +1
∆ ABC and GE = GB] ; ∴ EB = cm
2 2
24. 115° A
⇒ AB = AC = 10 cm
A

O I

B C
B 1 D 1 C
∠ BOC = 180° – ∠ BAC
AD = height = 10 − 1 = 3 cm
∴ ∠BAC = 50°
1
∴ Area = × 3 × 2 = 3 cm2 . 1
2 Hence, ∠BIC = 90° + ∠BAC = 115°.
2
22. 1 A
25. 2

3 0° 3 0°
2r 4 y 3
P R
90 – θ θ B D C
Let BC = x and AD = y.
9 0– θ
BD AB 4
As per Bisector Theorem, = =
DC AC 3
X 4x 3x
90 – θ Hence, BD = ; DC =
7 7
θ

Q S 16x 2
(4)2 + y 2 –
∠PXR = 90° (Angle in a semi-circle) In ∆ABD, cos30° = 49
∆PQR ~ ∆RPS 2× 4× y
PQ QR PR
= = 3 16x 2
PR PS RS ⇒ 2× 4× y× = 16 + y 2 –
2 49
⇒ PR2 = PQ × RS ⇒ PR = PQ × RS
16x 2
23. 2 P ⇒ 4 3y = 16 + y 2 – ...(i)
49

9x 2
9 + y2 –
Similarly, from ∆ADC, cos30° = 49
12 cm 2×3× y

9x 2
60 ° ⇒ 3 3y = 9 + y 2 – ...(ii)
49
Q R Now (i) × 9 – 16 × (ii), we get
4 3 cm
Since, the ratio of length of PQ to QR is 3 and the 12 3
36 3y – 48 3y = 9y 2 – 16y 2 ⇒ y =
measure of angle PRQ is 60 degrees, therefore PQR 7
is a right angled triangle right angled at Q. Alternative method:
Let the inradius and the circumradius of the triangle be
Area of ∆ABC = Area of ∆ ABD + Area of ∆ ADC
‘r’ and ‘R’ respectively.
1  ⇒
1 1 1
× 4 × 3 sin60º = × 4 × y sin30º + × 3 × y × sin30º
∆  × PQ × QR  2 2 2
r= =  2  = 4 3 ,
s 1 PQ + QR + PR
( ) 3 +1 12 3
2 ⇒ 12 3 = 4y + 3y ⇒ y = .
7

Page 4 QA - 26
QA - 27 : Geometry - 3 CEX-Q-0228/20
Answers and Explanations
1 1 2 4 3 3 4 1 5 2 6 4 7 3 8 6 9 48 10 3
11 38 12 3 13 4 14 3 15 3 16 2 17 90° 18 40° 19 3 20 2
21 40 22 3 23 2 24 4 25 4

O 4. 1 It is very clear, that a regular hexagon can be divided


1. 1 4 5° into six equilateral triangles. And triangle AOF is half
B C
of an equilateral triangle.
A Hence, the required ratio = 1 : 12.

5. 2 A B
a
a
a
a a
O a
360° F C
Internal Angle of an octagon = 180° − = 135o a a a
8 a a
The two base angles of the ∆OBC are (180° – 135°) a
= 45° each. E D
So, the internal angle of the star The regular hexagon can be divided into 12 triangles
= ∠BOC = 180° – (45° × 2) = 90°. of equal area, say a
Area of ∆AOF : Area of ∆AED : Area of ∆AEC : Area
2. 4 P Q of ∆FEC = 2a : 4a : 6a : 4a = 1 : 2 : 3 : 2.

W R 6. 4 Let the number of sides be x and 2x.


θ 360 360
Exterior angles will be and and their
x 2x
V S difference is given as 45.
So x = 4, thus sides will be 4 & 8.

U T
7. 3 In a regular octagon, each angle is equal to 135.
∠WPQ = Interior angle of a octagon = 135° Area of hexagon = area of rectangle PQYX + 2(Area
In ∆VWP, ∠WVP = ∠WPV of triangle PWX)
(Angles opposite to equal sides)
PX = 2 2 and XY = 2 units
and ∠VWP = 135o , ∴ ∠WVP + ∠WPV = 45°
Area of rectangle PQXY = 4 2
45°
∴ ∠WVP = = 22.5° Area of triangle PWX = 2
2
Required area = 4 2 + 4
∴∠PVQ = 45° − ∠WVP = 45° − 22.5° = 22.5°
∴ ∠WPQ − ∠PVQ = 135° − 22.5° = 112.5°. 8. 6 We know, for a ‘n’ sided polygon the sum of all internal
2 × (n − 2)π
3. 3 In this kind of polygon, the number of convex angles angles = .
2
will always be exactly 4 more than the number of
concave angles. 2 × (8 − 2)π
So for an octagon this sum = = 6π. .
NOTE : The number of vertices have to be even. Hence 2
the number of concave and convex corners should Now if, 8 angles are acute, then the sum will be <
add up to an even number. This is true only for the 8 × π 
  = 4π.
answer choice (3).  2 

QA - 27 Page 1
So, it cannot be true.
If 7 angles are acute then the sum of those 7 angles 11. 38 B 4 C

7 × π 
is <   = 3.5π. 13
 2  5 5
Then, the rest angle must be greater than (6π – 3.5 π)
= 2.5 π which can’t exist. A D
12 E 4
If 6 angles are acute, then the sum of those 6 angles
16
6 × π
is <   = 3π and the sum of other 2 angles > BE is ⊥ar to AD
 2  BE = CD = 5
(6π – 3π) = 3π.
AE = 132 − 52 = 144 = 12
So it can exist.
Hence the answer is 6. So, ED = 4
Also, ED = BC = 4
Therefore, perimeter of quadrilateral ABCD
9. 48 = 13 + 4 + 5 +16 = 38

6 0° 12. 3 Construction: Join DE


A E
B A B

D C
180 1 1
Interior angle of regular hexagon is 4 ×
6
= 120 and Area of ∆CED = × CE × DF = × 30 × 20
2 2
its external angle will be 60. On the other hand B is = 300 sq. cm
an interior angle of a regular pentagon, so B is 3 ×
1
180 Area of ∆CED = [Area of rectangle ABCD]
= 108. 2
5
Hence, area of rectangle ABCD = 600 cm2
By parallel lines property B = A + 60 and A = B – 60
= 108 – 60 = 48.
13. 4 D Q C
10. 3 A B
L C
R

K D
o B
A
P
J E
In ∆CRQ and ∆APR,
∠CRQ = ∠ARP (Vertically opposite angle)
I F
∠RQC = ∠RPA (Alternate angles)
H G ∴ ∆CRQ is similar to ∆ARP
A regular polygon ABCDEFGHIJKL of 12 sides can be AR AP
divided into 12 congruent triangles. ∴ =
Let O be the centre of circle circumscribing the RC CQ
polygon. AP 3
=
1 2 PB 2
Hence, Area of polygon = 12 × × r × sin 30° = 3r2 3
2 ∴ AP = AB
Area of circle = πr2 5
CQ 7
3r 2 3 =
Ratio of area of polygon to circle = = QD 3
πr 2 π

Page 2 QA - 27
7 7 Perimeter of APRBQA is
∴ CQ = CD = AB = AP + PR + RB + Arc AB
10 10
2π × 10
3 = X + 10 + Y + = 17 + 5π
AB 4
AR AP
∴ = = 5 6
RC CQ 7 =
AB 7
10 17. 90° D C
θ
AR AR 6 θ
∴ = =
AC AR + RC 13
A B
E
14. 3 A E B
∠DCE = ∠CEB = θ (CD || AB)
∠EBC = 180 – 2θ, EB = CB (Isosceles ∆CEB)
Hence, AD = AE
∠EAD = 2θ (∠EAD + ∠EBC = 180)
H F
180 − 2θ
Now, ∠AED = ∠ADE = = 90 − θ
2
Therefore, ∠DEC = 180 – (90 – θ) = 90°
D G C
Now, since the diameter of the circle is 10 cm, it will be 18. 40° A B
the diagonal of the rectangle. Thus, the sides of the
rectangle would be 8 cm and 6 cm. EFGH will be a x
rhombus, the length of whose diagonals are 6 cm and
8 cm.
4x + 5
Thus, the area is given by 3x 2y
1 1 D C
× d1 × d2 = × 8 × 6 = 24 sq. cm
2 2 3x + 4x + 5 = 180° (AD || BC)
x = 25°
P Now, x + 3x + 2y = 180° (Sum of the angles of D)
P 1 y = 40°
P+ + L∞ 1 − P 2 1
2 2 = ×
15. 3
A
A + + L∞
=
2A ( 2 −1 ) 2A 19. 3 Let a and b be the sides of rectangle.
2 2
a2 + b2 = (a + b) − a
( )
than
2P ( 2 + 1) 2 × 4a 2 +1 5
= = 16a b 8
2A 2 × a2 ⇒ = 6b ⇒ = .
5 a 15

=
2 ×2 ( 2 +1 ) =
(
2 2+ 2 )
a a 20. 2 R
S A
c
d c
D
16. 2 B d A0
b
Y B a b
a
Q R P C Q

Let 2a, 2b, 2c and 2d are the areas of four other


10 – Y
rectangles as shown in the figure above. Since AB,
BC, CD and AD are the diagonals of these rectangles,
so it will divide them into two equals parts.
A X P 10 – X C
Now, 2a + 2b + 2c + 2d + A0 = A
Let AP & BR be X & Y respectively. A − A0
2(10 – X) + 2(10 – Y) = 26 a+b+c+d=
2
(Perimeter of rectangle CPQR) Here, area of quadrilateral ABCD = a + b + c + d + A0
X+Y=7
PR = CQ = 10 (diagonal of rectangle) A + A0
= .
2

QA - 27 Page 3
Alternative method: x
We will try to make symmetrical figure by making all OC = OD = (OCD is an isosceles ∆)
2
the rectangles as squares as shown below. Drop a ⊥ar from F to AO
Let the area of each square be 4 sq. unit. ∆GCF will be an isosceles ∆.
Then A0 = 4 sq. unit and A = 36 sq. unit
Now, will reject the area of triangles ABS. BCP, CDQ x
and ADR from the larger square. GC = GF =
2
S A R In ∆OGF

D 2
 x 
( )
2
Radius = OF = GO2 + GF2 = 2x + 
 2
B
x2 5x 2
P C Q = 2x 2 + = = 2.5 x
2 2
1
⇒ Area of quadrilateral ABCD = 36 − 4 × ×4×2
24. 4 Here SQ = 8 unit and height of trapesium = 8 unit.
2
= 20 sq. unit and RQ = 4 2 unit
36 + 4 A + A 0 A R B
Here, 20 = =
2 2
i.e. only option (2) satisfy the given condition. 4
r
S Q
21. 40 By careful observation we can say that the shaded O 4
regions of first and last squares add up to form a
single square, same is for second square from left D P C
and second square from right.
4×4×4×5 Again, 4 × 4 = 4 2 × r
⇒ Area of shaded region = = 40cm2 .
2 ⇒ r = 2 2 unit

22. 3 20 π(2 2)2 16π π


Now, required ratio = = = .
10 r – 20 1 16 × 8 8
A C × (14 + 2) × 8
r 2
r – 10

B 25. 4 E
A B

Let the radius be r. Thus by Pythagoras’ theorem for Q


H P F L
∆ABC we have (r – 10)2 + (r – 20)2 = r2
i.e. r2 – 60r + 500 = 0. Thus r = 10 or 50.
It would be 10, if the corner of the rectangle had been
lying on the inner circumference. But as per the given
diagram, the radius of the circle should be 50 cm.

23. 2 A D G C
F Let BF = 3 unit. So, area of square ABCD
G
( )
2
4 5° = 2 3 = 12 square unit.
C 4 5° So, QF = 1 unit
E
1 3
Thus, area of triangle BQF = × 3 ×1= sq. unit.
X 2 2
4 5° X
2
3
4 5° 12 − 4 ×
O So, required ratio = 2 = 2 3 − 1.
X D B 3

2 2
Let O be center CD = x

Page 4 QA - 27
QA - 28 : Geometry - 4 CEX-Q-0229/20
Answers and Explanations
1 1 2 4 3 1 4 1 5 95° 6 2 7 4 8 22 9 2 10 2
11 3 12 2 13 30° 14 4 15 2 16 4 17 2 18 2 19 3 20 2
21 4 22 2 23 1 24 1 25 1

1. 1 b – a = 80° ...(1) ∠OAB = 80° – 40° = 40° = ∠OBA (OA = OB being


b + a = 180° ... (2) (Sum of angles on a straight line) the radius)
Adding (1) & (2), 2b = 260° ∠BOA = 180° – (∠OBA + ∠OAB) = 100°
b = 130°, a = 50°
c = a = 50° (Vertically opposite angles)
5. 95° E
Portion of total circumference made up by the arc of
50 5
angle c = = A
360 36 x 25°130 ° C
O 25° 30° P
2. 4 B 100 °40°
x 40° D
6 5° B
A C
O Let the centre of circle be O.
Reflex ∠AOC = 2 × 115° = 230°
E D
∴ ∠AOC = 130° similarly ∠BOD = 120°
In ∆ABC,
∠B = 90° (Angles in semicircle) ⇒ ∠OAE = ∠OEA = 25° (Isosceles triangle)
Therefore, ∠ABE = 90 – 65 = 25° and ∠OBD = ∠ODB = 40°
Also, ∠ABE = ∠ACE (Angle subtended by same arc AE)
In ∆APB
Also, ∠ACE = ∠CED [AC ED] x + 40° + x + 25° + 30° = 180°
Therefore, ∠CED = 25° ⇒ 2x = 85°
∴ ∠AOB = 180° – 85° = 95°
3. 1 If y = 10°,
∠BOC = 10° (opposite equal sides)
∠OBA = 20° (external angle of ∆BOC ) 6. 2 ∆ ACB is a right triangle. ∠BAC = ∠CBA = 45°
∠OAB = 20 (opposite equal sides) ⇒ BC = AC = r. So that, BD = 2r.
∠AOD = 30° (external angle of ∆AOC ) Required area = (Area of triangle) – (Area of sector)
Thus k = 3
1 90  π
= r × 2r − × πr 2 = r 2  1 −  = r 2 (0.214)
4. 1 ∠BAC = ∠ACT + ∠ATC = 50° + 30° = 80 o 2 360  4
And ∠ACT = ∠ABC (Angle in alternate segment)
o 7. 4 Let SR = x cm.
So ∠ABC = 50
Q
∠BCA = 180° − ( ∠ABC + ∠BAC) D R

= 180° − (50° + 80°) = 50


o
5 x
Since ∠BOA = 2∠BCA = 2 × 50° = 100o
M P S
Alternative Method:
Join OC
3 4
∠OCT = 90° (TC is tangent to OC)
∠OCA = 90° – 50° = 40°
∠OAC = 40° (OA = OC being the radius) A
AM × AD = AP × AQ = AS × AR
∠BAC = 50° + 30° = 80°
or 3 × 8 = 4 × (4 + x ) ⇒ x = 2 cm.

QA - 28 Page 1
8. 22 r
A Solving this, we get x = .
C 3
πr 2
Thus the area of the circle with centre S = .
O 9
P 2 1 1
The total area that can be grazed = πr  + 
4 9
B D
Tangent from a common point to a circle are of equal 13πr 2
length, so PA = PB =
36
Let O be the point at which tangent CD touches the
Thus the area of the field that cannot be
circle, so AC = OC and OD = BD. Therefore perimeter
of triangle PCD is PC + CO + OD + DP = 22 units. πr 2 13πr 2 5πr 2
grazed = − =
2 36 36
OP PR 4 5 2
9. 2 = = πr
OQ QS 3 36
The percentage = × 100 ; 28.
OP = 28 1 2
OQ = 21 πr
2
PQ = OP – OQ = 7
PQ 7 1
= = 13. 30°
OQ 21 3
S
Alternative method: P
OP PR 4
= =
OQ QS 3
Q R
OP 4 OP − OQ 1 PQ 1
⇒ −1= −1 ⇒ = ⇒ = PQ = PR = PS
OQ 3 OQ 3 OQ 3
Draw a circle with P as a centre an PQ as radius.
10. 2 PR + QS = PQ = 7 1
∠QSR = × ∠QPR (Angle at center is twice the
PR 4 2
= = ⇒ QS = 3 cm
QS 3 angle at the circumference)
1
= × 60 = 30°.
11. 3 SO = OQ2 – QS2 2

= 212 – 32 = 24 × 18 = 12 3 cm 14. 4

12. 2 If the radius of the field is r, then the total area of the

πr 2
field = . O C
2
The radius of the semi-circles with centre's P and 2 D
O
r
R= .
2 A B
2
πr
Hence, their total area = Let the radius of smaller circle be r.
4
Let the radius if the circle with centre S be x. ∴ O′B = r 2
r r  ∴ OB = O′B + O′D + OD
Thus, OS = (r – x), OR = and RS =  + x  .
2 2  =r 2 +r +2
Applying Pythagoras Theorem, we get Also OB = 2 2
2 2 ⇒r 2 +r +2=2 2
r r 
(r – x)2 +   =  + x
2 2  ⇒r =6–4 2

Page 2 QA - 28
15. 2 A B 1 cm C D 18. 2 C
B
9 0° 6 0° 6 0° 9 0°
6 0°

P Q 2 2
Drawn figure since it have not to be within distance of
1 m so it will go along APQD, which is the path of
minimum distance. A E F D
8
90 π 1 1
AP = × 2π × 1 = × AB × BD = × AD × BE
360 2 2 2
π 60 15
Also, AP = QD = ⇒ 2 82 − 22 = 8 × BE ⇒ BE = =
2 4 2
So, the minimum distance = AP + PQ + QD 2
 15  15 1
AE = 22 −   = 4 − =
π π  2  4 2
= + 1+ = 1+ π
2 2
 1 1
BC = EF = 8 −  +  = 7
2 2

16. 4 A
19. 3 B

F C
B C E
A
G
O
D
It is given that AB = BC = AC = BD = DC = 1 cm.
D
Therefore, ∆ABC is an equilateral triangle.
Let AF = x, DG = y and O is the centre of circle.
Hence, ∠ACB = 60° AF = FC = x, DG = BG = y (perpendicular from centre
of a circle to a chord divides it in two equal parts)
» = 60 × π(1)2 = π
Now area of sector AB FE = OG = x – EC
360 6 FO = EG = y – BE
Now, In ∆AFO
3 2 = 3 x2 + (y – BE)2 = r2
Area of equilateral triangle ∆ABC = (1) ....... (1)
4 4 In ∆OGD,
y2 + (x – EC)2 = r2 ....... (2)
¼ Adding (1) & (2),
Area of remaining portion in the common region ABC
2x2 + 2y2 + EC2 + BE2 – 2xEC – 2yBE = 2r2 ......(3)
π EA2 + EB2 + EC2 + ED2 = (2x – EC)2 + EB2 + EC2 + (2y
3
excluding ∆ABC = 2 ×  −  – BE)2
6 4  = 4x2 + 4y2 + EC2 + EB2 – 4x EC – 4yBE .......(4)
Hence, the total area of the intersecting region Using (3) & (4),
EA2 + EB2 + EC2 + ED2 = 4r2 = 4 × 22 = 16 km
3 π 3  = 2π − 3 sq. cm. Alternative approach,
= 2× + 4 ×  − 
4 6 4  3 2 B

17. 2 Area of shaded portion r


= 2 × (Area of sector POQ – area of ∆POQ ) C
E r
π 1  A r
= 2  r 2 − × r 2 × r 2 × sin120°
3 2 
r
 2π 3 2
= − r D
 3 2  Assume AC & BD as diameter of circle, then E is
centre of circle
EA2 + EB2 + EC2 + ED2 = r2 + r2 + r2 +r2 = 4r2 = 4 × 22
= 16 km.

QA - 28 Page 3
20. 2
23. 1

C1 C2
1 0 .5 0 .5 1 1
A B

C3

πr2 = 16π
1 1 ∴r = 4
(2π × + 2π × 1 + 2π × 1.5 + .... + 2π × 6.5) ∆C1C2C3 is an equilateral ∆ of side 2r = 8
2 2 Area of shaded region = Area of equilateral ∆ – 3 ×
1
= π( + 1 + 1.5 + ...... + 6.5) 3 60
2 Area of a sector = × 82 − 3 × × π × 42
7 22 7 4 360
= π × × 13 = × × 13 = 143 cm
2 7 2 = 16 3 − 8 π

21. 4 P Q R

24. 1 A

C2 28 45
C1 O O’
53

Let C1, C2, & C3 be three centres of three circles of B


radius a, b & c respectively PQ = 2 ac (Length of OAO’B is a kite and triangle OAO’ is a right angle
direct common tangent when two circles touch each triangle.
other externally) Hence, twice the area of triangle OAO’ is equal to the
Similarly, area of kite.
QR = 2 bc,PR = 2 ab 1  1
2  × 28 × 45 = × 53 × AB
PR = QR + PQ 2  2
2 ab = 2 bc + 2 ac
2 × 28 × 45
AB = = 47.5 cm.
Divide by abc on both sides, 53
1 1 1
= +
c a b 25. 1 Consider a square of side x.
Therefore, its area = x2
Therefore, area of the largest circle which can be cut
22. 2 C
πx2
from square = .
4
11 13
2 π 2  π
Therefore, area scrapped = x − x = x2 1 − 
4  4
r
r
 π
x2 1 − 
A D B Area scrapped  4 π
∴ = = 1 − = Cons tan t
20 Area of square x2 4
Let D be centre of semi-circle As this ratio is constant whether we cut a circle from
1 1 small square or larger square, scrapped area will be a
× 11 × r + × 13 × r = S(S − a)(S − b)(S − c) fixed percentage of square. Therefore, in our problem
2 2
as two squares are of the same size, the ratio will
⇒ 12r = 22(22 − 11)(22 − 13)(22 − 20) be 1 : 1.
⇒ 12r = 66 ⇒ r = 5.5
∴ 2r = 11.

Page 4 QA - 28
QA - 29 : Geometry - 5 CEX-Q-0230/20
Answers and Explanations
1 2 2 3 3 2 4 4 5 2 6 3 7 2 8 3 9 2 10 3
11 1 12 2 13 6 14 4 15 4 16 4 17 3 18 1 19 2 20 3
21 3 22 2 23 1 24 3 25 2

Ratio of curved surface areas


1. 2
= 2πr × r : 2π r × r : π × r × 2r
= 2: 2:1
cm

25
25

5. 2 P A F Q

D N G
4 4 cm 7
If you fold the arc you will get a cone with length = 25 M O
2πr = 44
22 E P B
i.e. 2 × × r = 44
7
⇒r=7 S H C R
Now, h2 = l2 – r2 = 252 – 72 = 242 Area of the path = Area of £ ABCD + Area of £ EFGH
1 + 4 × Area of (∆APD) – Area of ( £ MNOP)
V= × π r2 h
3 [As all triangles are congruent and 4 × Area of ∆APD
= Area of ( £ MNOP)]
1 22
= × × 7 × 7 × 24 = 1232 = AB × BC + EF × FG = 2(AB × BC)
3 7
= 2 × 40 2 × 4 2
2. 3 Sum of all the edges = 4(l + b + h) = 96 = 2 × 320 = 640.

diagonal = 15 = l2 + b2 + h2 6. 3 Total volume of water drops = Volume of glass


l2 + b2 + h2 = 225 3
l + b + h = 24 4  1  π
32000 × ×π×  = × r2 × 2r
(l + b + h)2 = 242 3  20  3
l2 + b2 + h2 + 2 (lb + bh + hl) = 576
225 + 2 (lb + bh + hl) = 576 32000 × 4
r3 =
2 (lb + bh + hl) = 351 = Total surface area 8000 × 2
r = 2 cm
3. 2 The area of well = 12 × 5 = 60m2 h = 2r = 4 cm
depth = 24.9 m
(250 × 60 – 60) × h = 60 × 24.9 7. 2
remaining increase in volume of earth
area height from well
12
60(250 − 1)h = 60 × 24.9
⇒ h = 0.10 m = 10 cm
9
20
4. 4 If you fold the flaps you get a cuboid

r r 2 Longest diagonal = 122 + 92 + 202 = 25 cm


r
Smallest diagonal = 122 + 92 = 15 cm
r
r r So, sum of diagonals = 25 + 15 = 40 cm

QA - 29 Page 1
8. 3 A
22 196 × 10
1 =2× × 14 × = 24640 cm.
2 7 7
B Since, there will be some shortage of length so length
2 of wire = 24639 cm (approx.)
1
C 12. 2
3
2
D h h h
4
3
E 2r 3r 4r

AB : AC : AD : AE = 2 : 3 : 5 : 8
Ratio of the volumes
⇒ V1 : V1 + 2 : V1 + 2 + 3 : V1 + 2 + 3 + 4 4x
= AB3 : AC3 : AD3 : AE3 8x
= 8 : 27 : 125 : 512 1 6x
V1 : V2 : V3 : V4 = 8 : 19 : 98 : 387 Ratio of volumes of cylindrical boxes = 4 : 9 : 16
Ratio of volumes of cuboidal boxes = 4 : 8 : 16
9. 2 If perimeter is constant, as the number of sides To equate the volume of the middle box,
increases areas also increases. Cylinder = 32 : 72 : 128
So, C > H > S > T Cuboid = 36 : 72 : 144
Total volume of cylinder = 232x
Total volume of cuboid = 252x
10. 3 24 18
 Volume  232x
  = = 0.3314 x
Pr ice  A 700
18
24  Volume  252x
  = = 0.315x
6 6 6 Pr ice  B 800
6 ⇒ Set A has a better deal.
42
We get 5 squares. 13. 6
2

24 3
3

42
HCF of (24, 42) = 6
So, the identical squares will be off 6 × 6. 3
Number of squares will be 4 × 7 = 28.
So, the ratio will be 5 : 28. πr2h = 9π
⇒r= 3 [h = 3 cm, given]
11. 1 2 2 2
h = 2 − ( 3)
h=1
So, the top most point will be 6 cm above the ground.

14. 4
0 .5
Since separation between two consecutive turns is
7 mm, 7
height 196 × 10
number of turns = =
diameter 7
Total length of wire = circumference × number of
turns
C h ee se spre a d d ia m ete r = 6
height
=2×π×r× A re a = π × 3 2
diameter

Page 2 QA - 29
17. 3 A B
0 .5 3

10

G F
6
D
C hee se spre ad d iam ete r = 9
2 E
A rea = π × 4.5

0 .5
C
Here, ABC is a 30°, 60° and 90° traingle and BC is
13
laying tangentially to the ball.
Hence, ∆ABC ~ ∆EDC ~ ∆CFG
3 1
= ⇒ DC = 2 ⇒ CE = 3
6 DC
C he ese sp read diam eter = 12
2
A rea = π × 6 3 AC
Ratio of areas = 3 : 4.52 : 62
2 Similarly, =
GF CG
= 4 : 9 : 16
Ratio of price = 240 : 540 : 960 3 3 3
= 4 : 9 : 16 = ⇒ GF = 3
GF 3
⇒ So, all deals are same.
π 4π 3
Now, required volume = ( 3)2 ⋅ 3 − ⋅1
F 48 G 3 3
15. 4
4π 5π
1 = 3π − = .
3 3
E D

18. 1
B
C h
1
2.6 A
4
The cross sectional view of pool. For every 2.6 m
inclination there is height of 1 m. 2
BC2 = 2.62 – 12 = 5.76 m2
BC = 2.4 m If the flaps are closed
∆ABC ~ ∆ADE
42 = 2 2 + h 2 ⇒ h = 2 3
BC AB 2.4 1
= ; = ⇒ AD = 20 m ⇒ AG = 21 m
DE AD 48 AD 3 3
1 Area of regular hexagon = (side)2
Area of AEFG = × 48 × (1 + 21) = 528 m2 2
2
Volume of pool = Area of AEFG × width 3 3
= ×2×2 = 6 3
= 528 × 20 = 10560 m3 2
1
16. 4 Let radius of bigger sphere = r Volume of pyramid = × base area × height
3
1 1
volume of cone = × π × r 2 × h = πr 3 1
3 3 = × 6 3 × 2 3 = 12
volume of cone = volume of smaller sphere 3
1/ 3
1 3 4 3 1  1 19. 2 Total volume = 150 × 120 × 100 = 1800000
πr = πr1 ; r13 = r 3 ; r1 =   r
3 3 4 4 Water available = 1281600
Remaining volume = 1800000 – 1281600 = 518400
ratio of surface areas = 4πr12 : 4πr 2
Bricks absorb 10% of water, so they increase 90%
2/3
 1 of volume.
=  2 r 2 : r 2 = 1: 24 / 3. 518400 = n × 90% × 20 × 6 × 4 ⇒ n = 1200
2 

QA - 29 Page 3
20. 3 Circumference of a wheel = π × d If it crawls the smallest distance will be equal to
22 (x + y)2 + z2
=
× 56 = 176 cm
7
Now, (x + y) should be minimum
Distance travelled in a min = 176 × 400
= 70400 cm = 704 m = (3 + 4)2 + 122 = 193
Distance travelled in 120 hrs by all
5 wheels = 120 × 5 × 712 × 60 Ratio of distances = 13 : 193 .
= 25344000 m = 25344 km
Distance covered by 4 wheels
24. 3
25344
= = 6336.
4

3 5
21. 3
4

20 15
h
Radius of the circular mark = 4
Circumference = 2πr = 8π

25
It is revolved along hypotenuse so 2 cones will be 25. 2
formed.
1 1
× h × 25 = × 20 × 15 48
2 2
h = 12

20 15
h 7

25 Surface area of cylinder


= 2πr (r + h)
20 15
22
=2× × 7 (48 + 7) = 2420
7
1 If you cut horizontally you will get 2 additional
Volume × π × 122 × 25 = 1200 π.
3 surfaces and both will be circles.
So increment in the surface area
22. 2 If the cylinder is cut and opened, the path will look like = 2 × πr2
22
=2× × 7 × 7 = 308.
7
33
If you cut vertically you will 2 additional surfaces
having rectangular faces increment in the surface
44 area will be = 48 × 14 × 2 = 1344.
33

Distance travelled = 2 332 + 442 = 2 × 55 = 110.

23. 1 If we cut and open the box

z
2420 + 308
x y Ratio of areas =
2420 + 1344
If insect flies the smallest distance covered will be
2728 682
= = .
body diagonal = 32 + 42 + 122 = 13 3764 941

Page 4 QA - 29
CEX-Q-0231/20
QA - 30 : P and C - 1
Answers and Explanations

1 – 2 1 3 1 4 2 5 1 6 3 7 4 8 1 9 4 10 4
11 2 12 4 13 4 14 – 15 3 16 3 17 3 18 – 19 3 20 –
21 4 22 1 23 – 24 – 25 3 26 1 27 – 28 2 29 2 30 3

1. A. Since, the thief is standing outside the room, he Hence, a favourable case can be as
can enter into the room from a window in 3 ways
_ m1 _ m2 5 women m3 _ m4 _ m5 _
and come out from a door in 2 ways.
i.e. total (3 × 2) = 6 ways. And the sixth women can be seated in 5 ways.
B. The thief can enter the room in 5 ways, and come So, m = 6 × 5! × 6! × 5 = 6! × 6! × 5
out in 5 ways (from a widow or from a door)
m
Thus, 5 × 5 = 25 ways. Hence, = 5.
n
2. 1 The first strip can be of any of the four colours, The
2nd can be of any colour except that of the first 6. 3 Case I: When boys and girls are alternate
(i.e. 3). Similarly, each subsequent strip can be of any One of the possible arrangements is shown below
colour except that of the preceding strip i.e. 3. B1G1B2G2B3G3B4G4B5G5
Hence, the number of ways = 4 × 35 = 12 × 81. The boys and the girls can be arranged in 5! ways
each i.e. total 5! × 5! ways.
3. 1 Number is of the form a1 a2 a3 a4 a5, where a1 ≠ 0 But the arrangement can also start with a girl at first
Possible geometric progressions are position.
(1, 1, 1), (2, 2, 2), (3, 3, 3) ... (9, 9, 9) and Thus, total 2 × 5! × 5! ways.
(1, 2, 4), (1, 3, 9), (2, 4, 8), (4, 6, 9)
Case II: When no two girls are together
∴ Total number of ways = 9 × (1 × 10 × 1 × 10 × 1) +
We will fix the boys’ position first
4 × (3! × 10 × 10) = 3300.
B1 B2 B3 B4 B5
The girls can be seated in 6C5 × 5! ways.
4. 2 Here, we need to find the number of times we write
2 while writing all the three digit numbers. Whereas, the boys can be arranged in 5! ways.
Thus, total = 6C5 × 5! × 5! = 6 × 5! × 5! ways.
1st 2nd 3rd Hence, required difference = 6 × 5! × 5! – 2 × 5! × 5!
= 4 × 5! × 5!
If we fix 2 in the first place, then 2nd and 3rd places
can be filled in 10 ways each.
i.e. 10 × 10 = 100 times 7. 4 We need to select digits only for unit's, ten's and
If we fix 2 in the second place, then 1st and 3rd places hundred's places. At remaining three places these
can be filled in 9 and 10 ways respectively. digits will be repeated. Unit's place can be filled in
i.e. 9 × 10 = 90 times, and if we fix 2 in the third place, 4 ways (because 0 cannot come at unit’s place) and
then 1st and 2nd places can be filled in 9 and 10 ways ten's and hundred's places can be filled in 10 ways
respectively. each.
i.e. 9 × 10 = 90 times. ∴ Total number of ways = 10 × 10 × 4 = 400.
Thus, total 100 + 90 + 90 = 280 times.
8. 1 If the person uses 1 step at a time, then 1 way.
5. 1 We will make a graph of 6 women in which all of them If the person uses 2 steps at a time, then 1 way.
are arranged in consecutive.
Hence, n = 6! × 6! (since the women can be arranged 3!
If the person uses 2, 1, 1 step, then = 3 ways.
in 6! ways inside their group). 2!
In second case, we have to keep exactly 5 women If the person uses 3, 1 step, then 2 ways.
consecutively. So we will first select 5 women out of If the person uses 4 steps, then 1 way.
6 women. In 6C5 = 6 ways and women in group can i.e. total = 1 + 1 + 3 + 2 + 1 = 8 ways.
be arranged in 5! ways.

QA - 30 Page 1
9. 4 The number of ways in which the child can choose or Case III:
not choose an Eclairs = 5 + 1 = 6.
Similarly, for KitKat and Melodies, there are 5 and 4
4 ways available. 2 3 6 → 3! ways
Now, for 7 distinct chocolates, there would be 2 ways or 2 6 0 → 2 × 2 × 1 = 4 ways
for each.
Thus, total no. of ways in which the person can or 2 3 0 → 2 × 2 × 1 = 4 ways
choose one or more chocolates = 6 × 5 × 4 × 27 – 1 i.e. 14 ways.
= 15359
Case IV:
10. 4 Total four-digit numbers that can be formed with
0, 1, 2, 7 = 3 × 4 × 4 × 4 = 192 6
Out of these, in 3 × 3 × 2 × 1 = 18 numbers, no digit is 2 3 4 → 3! ways = 6 ways
repeated.
2 4 0 → 4 ways
⇒ In remaining 192 – 18 = 174 numbers, we will have
at least one digit repeated. i.e. 10 ways
Total = 12 + 14 + 14 + 10 = 50 ways
11. 2 Since there should not be any empty basket, this is the
case of partitions.
i.e. 6 can be divided in three parts as 15. 3
(1 + 2 + 3), (2 + 2 + 2) and (1 + 1 + 4) 0 1 1 1 1
Thus, there are 3 ways to fill the baskets with balls. 1 0 1 1 1
12. 4 Selecting 7 elements is as good as rejecting 2 elements. 1 1 0 1 1
So, we can reject 2 elements as AA or BB or CC or AB
or BC or AC, i.e. 6 ways. 1 1 1 0 1

13. 4 Since, there can be 3 members in a committee, there 1 1 1 1 0


can be at most one couple.
The number of ways in which a couple is definitely This is a possible 5 × 5 table with each entry either 0
selected = 4C1 × 6C1 = 24 ways. or 1 and sum of each row or column is 4.
Total number of selecting 3 people from 8 people = 8C3 We can arrange the rows or column in 5! = 120 ways
∴ Total cases where no couple is there to get every possible combination.
= 8C3 – 24 = 32.
16. 3 Let a and b be two numbers.
14. 50 For a number to be divisible by 6, it must be even and Then, a + b = 120
it’s digit sum must be a multiple of 3. Since a and b both cannot be equal to greater than 60.
Case I: Let 0 ≤ a ≤ 59 and 61 ≤ b ≤ 120
The total number of ways in which ‘a’ can be chosen
0 = 60C1 = 60
6 4 2 → 3! ways The value of b depends on the value of a and there is
one value of b corresponding to one value of a.
or 2 3 4 → 3! ways
So, total number of sets having two numbers = 60
i.e. 12 ways
17. 3 The arrangement of 8 distinct fruits, in a row, is given
Case II: by 8! ways.
But the condition is Apple(A) must be before
Banana(B), and B must be before Mango(M).
2
The possible cases of the arrangement of these three
3 6 4 → 3! ways fruits can be given as,
or 4 6 0 → 2 × 2 × 1 = 4 ways[Since 0 ABM, AMB, BMA, BAM, MAB, MBA (Not necessarily
cannot appear at first conjecutive)
position] Here, out of 3! Arrangements, only 1 arrangement is
valid.
or 3 4 0 → 2 × 2 × 1 = 4 ways[Since 0
cannot appear at first 8!
So, out of 8! Arrangements, there would be
position] 3!
arrangements valid.
i.e. 14 ways.

Page 2 QA - 30
18. A. When two particular players are always selected, B. First let us think of the distributions. The boxes
then we will select those 2 players first, can have
i.e. 1 way, and then we need to select 4 more 1, 2, 3: This can be done in 6C3 × 3C2 ways. First
players out of 9 available players. select 3 out of 6, and then 2 out of the remaining
i.e. 9C4 ways. 3. This is nothing but distributing 6 as 3, 2, 1 which
B. When two particular players are never selected,
then we will reject those 2 players first, i.e. 6!
can be done in ways
1 way, and then we need to select all the 6 players 2! × 3! × 1!
out of 9 available players. 1, 1, 4: This can be done in 6C4 ways. Once we
i.e. 9C6 ways. select 4 out of 6, the other two go into one box
each. Since the boxes are identical, we do not
19. 3 Taking I for India and S for South Africa. We can arrange
have to worry about selecting anything beyond
I and S to show the wins for India and South Africa
the first set of 4 toys.
respectively.
2, 2, 2: This looks like it could be 6C2 × 4C2 ways.
Let, South Africa won the series, then last match is
always won by South Africa, and it will be fixed. But this will carry some multiple counts. The idea
Thus, following are the possible ways in which this we are using here is simple – select 2 out of 6 and
can be done then select 2 out of 4.
When we do this, a selection of AB, and then CD
Wins for India Wins for S.A. No. of ways will get counted. This will get accounted as AB,
1 0 4 1 CD, EF. However, we will also be counting a
selection of CD, AB, EF, and EF, AB, CD. Since the
2 1 4 4!/3! = 4
boxes are identical, all these selections are
3 2 4 5!/2!3! = 10 effectively the same. So, number of ways would
4 3 4 6!/3!3! = 20 6
C2 × 4C2
be
So, total number of ways = 35 3!
In the same number of ways India can win the series. So, total number of ways of doing this would be
Thus, total number of ways = 35 × 2 = 70. 60 + 15 + 15 = 90 ways.

20. A-540, B-90 21. 4 For a 3-digit number:


A. Again, let us start with the distributions. Case (i): All digits different = 4P3
Scenario I: (1, 2, 3): This can be done in 6C3 ×
3 3!
C2 × 3! ways. First select 3 out of 6, and then 2 Case (ii): Two 1’s and one other number = 3C1 ×
out of the remaining 3. After we have done this, 2!
the toys can go into the three distinct boxes in 3!
ways. 360 ways 3!
Hence, total number of ways = 4P3 + 3C1 × .
Scenario II: 1, 1, 4: This can be done in 6C4 × 3! 2!
ways. Once we select 4 out of 6, the other two 22. 1 This is the problem of derangement.
go get broken up as 1 and 1. Now, we have In this case, we will first select the 3 correct letters
something akin to ABCD, E and F to be allotted into which are going into right envelopes.
3 distinct boxed. This can be done in 3! ways. The number of ways in which this can be done = 8C3
90 ways The rest 5 letters are going into wrong envelopes,
Scenario III: 2, 2, 2: This should be 6C2 × 4C2 ways. which is given by derangement of 5, i.e. D(5)
The idea we are using here is simple – select 2
out of 6 for the first box and then select 2 out of 4 1 1 1 1 1
Since, D(n) = n!  – + – + .... 
for the second box. 90 ways.  2! 3! 4! 5! n! 
Total number of ways = 360 + 90 + 90 = 540
ways. ⇒ D(5) = 44
Now, this question can be rephrased wonderfully Thus, total number of ways in which this can happen
like this: = 8C3 × 44 ways.
How many onto functions can be defined from
{a, b, c, d, e, f} to {1, 2, 3}? 23. There are 10 digits available, i.e. from 0 to 9
You can solve the above question by thinking of If we select any three digits, the condition will
all functions from the first set to the second and automatically be satisfied.
subtracting the non–onto functions from that. Thus, 10C3 = 120 ways.
Needless to say, we would get the same answer.

QA - 30 Page 3
24. 16 The number of heads are greater than the number of The total number of arrangements would be 10!
tails. The following cases can be made But A4 must be before A3, A3 must be before A2 and
A2 must be before A1. i.e. out of 4! Arrangements,
No. of Possible only 1 arrangement would be valid. Similarly, for
No. of tails No. of ways
heads arrangement
columns B, C and D, out of 3!, 2! and 1! arrangements
0 5 HHHHH 1 respectively, only 1 arrangement would be valid in
1 4 THHHH 5!/4! = 5 each case.
2 3 TTHHH 5!/2!3! = 10
10!
Thus, total number of ways =
Hence, total number of ways for the possibility = 16 4!3!2!1!

25. 3 The student can select 1 book or 2 books or 3 books


and so on up to n books. 28. 2 A B C D E F G H I J K L
x x x x x
The number of ways can be given by Let the 12 persons be A, B, C, D, E, F, G, H, I, J, K and L.
2n+1C + 2n+1C + 2n+1C + 2n+1C +…+ 2n+1C
0 1 2 3 n We will select 5 persons which are not consecutive
= 255 + 2n+1C0
first. Suppose we have selected D, F, H, J and L.
⇒ 2 =2 .
2n 8
Now, we are left with A, B, C, E, I, G and K, and we
⇒ n=4
have to place D, F, H, J and L in such a way that they
are not consecutive.
26. 1 Since, 2 + 3 + 4 +…+ 13 = 90
Now, when 7 people A, B, C, E, G, I and K are seated
Thus, first boy will be getting 2 chocolates, second
boy will be getting 3 chocolates and so on. then there would be 8 places in which D, F, H, J and L
This can be done in 12! ways. can be seated.
Also, we have to keep the number of chocolates This can be done in 8C5 = 56 ways.
distinct and we are still left with 2 more chocolates.
Either these 2 chocolates can be given to the boy who 29. 2 In the set of 43 consecutive numbers, there would be
is getting 13 chocolates or we can give 1 chocolate 14 numbers in the form of 3n + 1, 14 numbers in the
each to the boys with 12 and 13 chocolates(to keep form of 3n + 2 and 15 numbers in the form of 3n
number of chocolates distinct) (Note: the number of numbers of the given forms
Thus, 2 × 12! ways. depends upon the starting point of these 43 numbers,
but there won’t be any effect in the number of ways in
10! which we can make a + b + c divisible by 3).
27. Here, we can select three numbers which are in the
4!3!2!1!
We number the targets as 1, 2, 3, 4 as shown below. form of 3n, or three numbers which are in the form of
3n + 1, or three numbers which are in the form of 3n +
A B C D 2 and one number each from each category because
there sum would be a multiple of 3.
So, total number of ways in which this can be done is
14C + 14C + 15C + 14C 14C 15C = 4123 ways.
1 1 1 1 3 3 3 1 1 1

30. 3 Let the selected numbers be a, b and c. Since, these


2 2 2 are in AP, a + c = 2b.
Here, RHS is always even, so LHS must be even.
This is possible if both a and c are even or both are
3 3 odd.
Since, from 1 to 15, there are 8 odd numbers and
7 even numbers, so number of ways of selection
= 8C2 + 7C2 = 28 + 21 = 72.
4

Page 4 QA - 30
CEX-Q-0232/20
QA - 31 : P and C - 2
Answers and Explanations

1 2 2 1 3 1 4 4 5 4 6 3 7 3 8 1 9 2 10 4
11 1 12 4 13 1 14 1 15 3 16 3 17 2 18 4 19 3 20 3
21 6 22 1 23 3 24 1 25 3

1. 2 Since we have to keep the vowels together, we will 6. 3 Since, we don’t have to consider the adjacent
make all A’s and I together in the form of a group. students, each student will sing a song with 12 more
students so it is similar to finding the number of
A A I , W, R, N, L, S diagonals in a polygon with 15 sides.
So, 15C2 – 15 = 90 distinct pairs.
These can be arranged in 6! ways. Thus, the chorus will last for 90 × 2 = 180 minutes.
Further, AAI can be arranged in 3 ways
So, total number of possible arrangements = 3 × 6! 7. 3 To form a quadrilateral, we need to select 2 vertical
lines and 2 horizontal lines.
2. 1 Since we have to keep no two S together, we will first This can be done in nC2 × mC2
fix other letters as
_L _ E_ E_ P_L_E mn  m - 1 n - 1
= ways.
There are 7 places in which S can be placed in 4
7C ways. Also, the letters LEEPLE can be arranged in
3
8. 1 There would be 5 different positions for the first person
6! who is going to sit.
ways.
3!2! Also, the rest 9 people can be seated in 9! ways.
Thus, total number of ways = 5 × 9! ways.
6!
Thus total number of ways = 7C3 × = 2100.
3!2!
9. 2 1
3. 1 There are three cases:
(1) No fruit is selected: Only one way.
(2) One fruit is selected: Four ways. 2
(3) Two fruits are selected:
(a) Both fruits are identical: Three ways. We will try to break the symmetry. So, the face 1 can
(b) Both fruits are distinct: 4C2 = 6 ways. be painted in 1 way, and then the facing face (face 2)
Hence, required number of ways = 14. can be painted in 5 ways.
Now, we will get a circular arrangement with 4 places,
4. 4 Number of diagonals in any polygon = nC2 – n i.e. the remaining faces can be painted in 3! ways.
So, total number of ways = 5 × 3! = 30 ways.
∴ n C2 − n = 2n ⇒ n C2 = 3n

10. 4 A C Y
n(n − 1)
⇒ = 3n
2 X

⇒n=7 E D
Hence, the polygon is a heptagon. P
F
5. 4 We will first let all the boys occupy 4 seats around the
circular table. This can be done in 3! ways.
Now, 4 girls can take the 4 seats between the boys.
This can be done in 4! ways. B
Thus, total number of ways in which they can be For the shortest route, Neelam follows the following
seated = 3! × 4! = 6 × 24 = 144. path:
A →E →F→B

QA - 31 Page 1
As we have to form a quadrilateral, we will select
Number of ways to reach from A to E:
(2 + 2 )! = 6 4 points out of 28 points, which can be done in 28C4
2! × 2! ways.
Number of ways to reach from E to F: 1
( 4 + 2 )! = 15 15. 3
 n − 3
In any hexagon, there are n  = 9 diagonals.
Number of ways to reach from F to B:
4! × 2!  2 
⇒ Total number of possible shortest paths First let us draw the diagonals and try to visualise this
= 6 × 1 × 15 = 90 diagram

11. 1 Neelam has to reach C via B. A B


From A to B, the number of paths are 90, as found in
the previous question.
From B to C, Neelam follows the route: F C
Case I: B → X → C
OR Case II: B → Y → C .
Case I: B → X → C E D

(5 + 1)! There are six ‘short’ diagonals AC, AE, CE, BD, BF, and
Number of ways to reach from B to X: =6
5! × 1! DF. These intersect with other diagonals at 3 points
each.
Number of ways to reach from X to C : 2
There are 3 long diagonals – AD, BE and CF. These
So, total number of paths are 6 × 2 = 12 ways.
intersect with other diagonals at 4 points each.
Case II: B → Y → C : Note that the ‘short’ diagonals need not be shorter
There is just one way. than the ‘long’ diagonal.
Therefore, from B to C, there are 6 × 2 + 1 = 13 So, the total number of points of intersection should
ways be 6 × 3 + 3 × 4 = 30. But in this case, we would count
∴ Total number of ways of reaching from A to C, every point of intersection twice. So, number of points
via B = 90 × 13 = 1170.
30
of intersection would be exactly half of this =
12. 4 The minimum height that can be achieved is 20 cm. 2
The, maximum height that can be achieved is 50 cm. = 15 points.
But, height equals to 49 cm cannot be achieved.
Thus, total 30 distinct heights can be achieved. 16. 3 The number of ways in which a pack of 52 cards can
52!
13. 1 The first word with 2 distinct consonants and 3 distinct be divided into four persons =
vowels would be ABCDE. (13!)4
Now, if we fix AB in the first two positions, there must 4 person sitting around a circular table will have no
be 1 vowel and 2 consonants more at the next bearing on the total number of distribution of cards.
3 places, which can be done in 20C2 × 4C1× 3! ways.
i.e. the number of words starting with AB = 20C2× 17. 2 First we divide 52 cards into 2 groups — 4 cards and
4C × 3! = 190 × 4 × 6 = 4560.
1 48, i.e. (16 × 3) cards.
Next, we move on to words starting with ACB, in the
form of ACB_ _ would be 19C1 × 4C1 × 2! = 152 ways. 52!
This can be done in ways.
Now, the words starting with ACDB, there would be 48! 4!
4 distinct words on this list- ACDBE, ACDBI, ACDBO, Now, a group of 48 cards can be divided into 3 groups
ACDBU
Further, the next words would be ACDEB and ACDEF 48!
of 16 cards each in ways.
(16!)
3
Thus, rank of ACDEF = 4560 + 152 + 4 + 2 = 4718. 3!

14. 1 Since we have to maximize the number of Hence, the required number of ways
quadrilaterals, let 1 line is getting intersected by rest 52! 48! 52! 52!
7 lines. = × = =
48! 4! (16!) 3! (16!) ( 4!)(3!) (16!) (3!) 4
3 3 3 2

There would be 7 point of intersections on a line.


Thus, total points of intersection on 8 lines = 8 × 7
= 56. 1 52! 
=  
But in this case, all the points are getting counted 4  (16!) (3!) 
3 2
 
56
twice. Thus, total points of intersection = = 28
2

Page 2 QA - 31
18. 4 Here we have three different cases.
22. 1 Delhi (1) (2) (3) (4) .......... (11) Dhanbad
1. All rings are worn in just one finger, this is possible """" """" !
Intermediate Stations
in 5 × 3! = 30 ways.
2. All rings are worn in 2 of the 5 fingers. This is Firstly, we will select 4 intermediate stations which
are not consecutive.
5
possible in C2 × 2C1 ×3 C2 × 2 = 120 ways. Let us select stations (3), (5), (8) and (10)
3. All rings are worn in 3 of 5 fingers. This is possible D (1) (2) (3) (4)
" " ! { (5) (6)
" "
(7) (8) (9)
! { (10) (11)
{ G
b d e
in 5 P3 = 60 ways.
a c

The selection of 4 intermediate station divides the


∴ Total number of ways = 30 + 120 + 60 = 210. remaining station into five groups a, b, c, d and e such
that a + b + c + d + e = 11 – 4 = 7
Alternate method: a+b+c+d+e=7
If we consider the three rings to be identical and five Since there must be at least 1 station between any
fingers are F1, F2, F3, F4 and F5 then the question gets two stoppages,
converted to F1 + F2 + F3 + F4 + F5 = 3 Number of positive integral solutions of above equation
∴Total number of non-negative solutions = 7C3 = 35 = 6C4 = 15 ways.
As rings are distinct, required answer = 35 × 3! = 210.
Or, 23. 3 Let a, b and c represents VARC, LRDI and QA
The first ring can be worn in 5 ways, then there will respectively.
be 6 more places for next ring. After placing 2nd ring Here, a + b + c ≤ 60
there would be 7 more distinct places for the 3rd ring. where, a, b and c are whole numbers.
Hence, total number of ways = 5 × 6 × 7 = 210 We introduce z such that
⇒ a + b + c + z = 60, where z is a whole number.
19. 3 Let the number of Blue, Green and Yellow balls be x, The number of solution of this is given by 63C3.
y and z respectively.
Then x + y + z = 20, where x, y and z are whole 24. 1 In this case, scoring 240 marks is as good as not
numbers. scoring 60 marks.
The solution of this equation is given by 20+3–1C2 Thus, a + b + c = 60
= 22C2 = 231 ways. The number of solution for this equation = 62C2.

20. 3 Any term of the expansion is given by apbqcrdse t 25. 3 Here, a + b + c = 140 ...(i)
such that p + q + r + s + t = 20, where p, q, r, s and t are But 0 ≤ a, b, c ≤ 100
whole numbers. So, we will consider all the cases first and then
The solution of this equation is given by 20+5-1C5-1 subtract those cases which violates the conditions.
= 24C4 Total number of solutions of equation (i) = 142C2
Thus, there would be 24C4 terms. But this also includes those cases in which a, b or c is
more than 100.
21. 6 Let Amal, Bimal and Kamal are getting A, B and K So, we will assign 101 marks to a such that it could
number of pens. violate the condition.
So, A + B + K = 8 ⇒ 101 + a + b + c = 140
But A ≥ 1, B ≥ 2 and k ≥ 3 ⇒ a + b + c = 39
⇒A+B+K=2 ⇒ 41C2 solutions.
Where A, B and K are whole numbers. To violete the conditions, same can be done with b
The number of solution of this equation is 4C2 = 6 and c as well.
So, total number of ways in which condition can be
violated = 3 × 41C2
Thus, the number of ways in which he can score
140 marks = 142C2 – 3 × 41C2 = 7551 ways.

QA - 31 Page 3
CEX-Q-0233/20
QA - 32 : Probability
Answers and Explanations

1 3 2 3 3 3 4 1 5 1 6 2 7 1 8 1 9 2 10 3
11 4 12 2 13 5 14 3 15 5 16 1 17 1 18 2 19 4 20 3

1. 3 Let R denotes the event where a red ball is selected. Thus, the total ways in which the two people are
Let S be the event wherein the smallest red ball is always together = 5! × 2 = 240 ways.
selected. And the number of ways in which all the 7 people can
Thus, probability of selecting the smallest ball given be arranged around the circular table is 6! = 720.
that it is red is given by, Thus, the probability of having the two people always

 1 240
  sitting together is = 0.33.
 S  P(S ∩ R)  18  1 720
P  = = = .
 R P(R)  6 6
  5. 1 Out of 2n numbers, if we select (n + 1) numbers, there
18
must be a pair of consecutive numbers. So, in all the
cases the probability of getting 1 as their HCF is 100%
2. 3 In this question, the number of students i.e. 30 is a i.e. 1.
redundant data and thus needs to be ignored.
The probability of a project getting rejected by Don is 6. 2 Let us first find the probability of having no pairs in the
0.4 and that of by Ron is 0.6. four drawn cards.
Hence, the probability of a project being rejected by The first card is picked at random.
both Don and Ron is 0.4 × 0.6 = 0.24. The next card can be picked from the remaining 11
cards with the exception of 1 card which is of the
1 same value as the first card.
3. 3 Probability of selection of bag I and bag II each is . The third card can be picked from the remaining 10
2
cards with the exception of the cards having the same
3 value as the first 2 picked cards.
Probability of picking up a red ball from bag I = The fourth card can be picked from the remaining 9
7
cards with the exception of the cards having the same
3 1 value as the first 3 picked cards.
⇒ Probability of a red ball from bag I = 7 × 2 So, the probability of the above event
10 8 6 16
5 = × × =
Probability of picking up a red ball from bag II = 11 10 9 33
11
So, the probability of picking at least one pair
5 1
⇒ Probability of a red ball from bag II = 11 × 2 16 17
= 1− = .
33 33
5 1
×
11 2 35
Now, the required probability = 3 1 5 1 = . 7. 1 For the preparation to be uneatable it should have at
68
× + × least 1 rotten egg.
7 2 11 2 The probability of having all good eggs in the selected
18
C4 3060
4. 1 Let us consider the 2 people are sitting next to each 4 eggs is 20
= = 0.632
other. C4 4845
Thus, considering them to be as one individual we Thus, the probability of having a spoiled preparation
can say that there are 6 people to be arranged around = the probability of having at least one rotten egg in
a circular table and this can be done in 5! ways. the 4 eggs picked for making the preparation
And the 2 people can be arranged between = 1 – P(Having all good eggs for the preparation)
themselves in 2 ways. = 1 – 0.632 = 0.368.

QA - 32 Page 1
8. 1 For the number to be divisible by 25, the last 2 digits of The number of ways in which 4 tails can occur out of
the number should be divisible by 25, i.e. the last 2 7 tosses is the number of combinations of 7 objects
digits should be 00, 25, 50 or 75. taken 4 at a time.
Thus, in the given case, we are supposed to consider The number of combinations of n objects taken r at a
numbers ending with 25.
n!
The total number of ways in which a four digit number time is given by C (n, r) = .
(r!(n − r)!)
can be formed using the given four digits
= 4 × 3 × 2 × 1 = 24 ways.
7! 5040
And the number of ways in which the four digit number C (7, 4) = = = 35
(4!(3!)) 144
can be formed with ending 25 = 2
[i.e. the thousands and hundreds place can be filled in
35
2 ways, 13 or 31.] Hence, the probability of occuring of 4 tails is .
128
2 1
Hence, the required probability = = .
24 12 12. 2 Let probability of getting an even number be x.
⇒ Probability of getting an odd number = 2x
In a particular throw either an odd number, out of three
9. 2 A B odd numbers, or an even number, out of three even
numbers, will appear.
O So, 3(x) + 3(2x) = 1
6 0° 1
⇒ x=
A B 3
Now a two-digit prime number possible in two
Centre will lie inside ∆ABC only when C lies on minor
Arc A’B’. successive throws is 11 only i.e. sum of 5 and 6.
60 1 1 2 2 1 4
∴ Required probability = = . So, required probability = × + × =
360 6 9 9 9 9 81

10. 3 In this case, it is easy to calculate the probability of not


drawing a blue marble. The probability of an event 13. 5 The probabilities of not receiving the gift from retail A,
occurring plus the probability of the event not occurring B, C and D are 0.4, 0.2, 0.1 and 0.5 respectively.
is equal to 1. If the probability of drawing a blue marble Then probability of receiving the gift from at least one
is at least 70%, the probability of not drawing a blue retailer = {1 – (0.4 × 0.2 × 0.1 × 0.5)} = 0.996.
marble can be at most 30%.
There is a probability of 6/10 on each draw such that 14. 3 Let the total number of balls in the bag I be ‘x’; the total
the marble drawn is not blue. The probability of event number of balls in bag II be (18 – x); the number of
A and event B occurring is the probability of event A red balls in bag I and bag II be ‘a’ and ‘c’ respectively;
times the probability of event B, given that the event A and the number of blue balls in bag I and bag II be ‘b’
has already occurred. Since the probability of not and ‘d’ respectively.
drawing a blue marble on each draw is 6/10, we must
∴ a + b = x and c + d = 18 – x
determine how many times one has to multiply 6/10 by
itself in order to reduce the probability to below 30%. a c 5
∴ × =
(0.6)n < 0.30 x 18 – x 16
(0.6)1 = 0.6; Let a × c = 5k and x × (18 – x) = 16k, where k is a
(0.6)2 = 0.36; natural number.
(0.6)3 = 0.216 Each bag has marbles of both colors.
Thus, 3 draws are required.
∴ 2 ≤ x ≤ 16 and 2 ≤ (18 – x) ≤ 16
11. 4 The number of possible outcomes can be determined As, x + (18 – x) = 18 ⇒ Max {x × (18 – x)} = 81
by the multiplication principle. The multiplication principle ∴k ≤ 5
tells us that the number of ways in which an
independent events can occur together can be By hit and trial, the only possible value of k comes out
determined by multiplying together the number of to be 5.
possible outcomes for each event. There are two ∴ x = 10, 18 – x = 8, a = 5, b = 5, c = 5 and d = 3
outcomes possible while tossing a coin i.e., heads or
5 3 3
tails. Hence, the required probability = × = .
Thus, the number of possible outcomes are 10 8 16
= 2 × 2 × 2 × 2 × 2 × 2 × 2 = 128.

Page 2 QA - 32
17. 1 Required probability = (1 – 0.418) (1 – 0.612)
15. 5 A B (1 – 0.355) (1 – 0.520) ≈ 0.069

P Q 18. 2 Probability of damaged packets in all packaging runs


4 = 0.6 × 2x + 0.4 × x, where ‘x’ is the probability of
packaging at normal speed.
4 1 0 cm ⇒ 0.112 = 0.6 × 2x + 0.4 × x
⇒ 0.112 = 1.2x + 0.4x
R S ⇒ x = 0.07
∴ Probability of non damaged packets at normal speed
D C = 1 – 0.07 = 0.93.

Here the centre of the circle must be within the smaller 19. 4 In order to catch the bus, the mechanic has to leave
square drawn as shown in the figure above. the factory in 12 minutes. As inspecting one machine
takes 6 minutes, he will identify the two faulty machines
4×4
⇒ Probability = = 0.16 in 12 minutes if the first two machines he inspects are
10 × 10 either both faulty or both working properly.
1 1 1 1 1
16. 1 There are two multiples of 3 i.e. 3 and 6. Required probability = × + × =
2 3 2 3 3
2 1
So, probability of getting 3 or 6 = = 20. 3 We will draw a square of side ‘a’ and an arc of radius
6 3
‘a’. All rectangles with diagonal less than or equal to a
2 will lie within or on the quadrant of the circle as shown
⇒ Probability of not getting 3 or 6 = below.
3
The probability of getting 3 or 6 in ‘n’ throws is given a
n 10 − n
10  1  2
by Cn ×   ×   .
 3  3 a
For at least 8 throws, n will be 8, 9 or 10
⇒ Required probability
8
 1  2 
2
 1  2   1
9 1 10
 2
0  π
= 10C8     + 10C9     + 10C10   a2  1 − 
 3  3  3  3  3    4 π
3 Hence, required probability = = 1− .
a2 4
1 201
= [180 + 20 + 1] = 10 .
310 3

QA - 32 Page 3
Solutions CEX-Q-0234/20
QA - 33 : Trigonometry

1 2 2 3 3 1 4 27° 5 2 6 2 7 3 8 3 9 2 10 2
11 4 12 3 13 1 14 4 15 3 16 3 17 1 18 2 19 1 20 4
21 4 22 4 23 2 24 2 25 2 26 4 27 4 28 4 29 3 30 1

c 0 6. 2 tan5° tan25° tan30° tan 45° tan65° tan85°


π  π 180 
1. 2  9  =  9 × π  = 20° = ( tan5° tan85° )( tan25° tan65° ) tan30°
   
=  tan5° tan (90° − 5° )  tan 25° tan (90° − 25° ) tan30°
2. 3 Let l be the length of the arc subtending an angle
θ radian at the centre of a circle of radius r. Then = ( tan5° cot 5° )( tan25° cot 25° ) tan30°
c
l  π  = 1× 1×
1
=
1
θ= . We have, r = 6 cm and θ = 18° = 18 ×
r  180  3 3

c
 π 
θ=  7. 3 2(cos4 60° + sin4 30°) − (tan2 60° + cot 2 45°) + 3 sec 2 30°
 10 
 1  4  1  4   2  2 
2
π 6π 3π
( 3)
l l 2
∴θ = ⇒ = ⇒l= = = 2   +    −  + (1)  + 3  
10 6    2      3
r 10 5 2

tan1° tan2° tan3°...tan89°  1 1 4 1 1


 − (3 + 1) + 3 × 3 = 2 × − 4 + 4 =
3. = 2 +
 16 16  8 4
= tan (90° − 89° ) tan (90° − 88° ) tan (90° − 87° )
...tan87° tan88° tan89° 8. 3 A
= cot 89° cot 88° cot 87°...tan87° tan88° tan89°
√5 cm
1 cm
= (cot 89° tan89° )(cot 88° tan88° )(cot 87° tan87° )
θ
...(cot 44° tan 44°)tan 45° C 2 cm B
= 1 × 1 × 1 ... × 1 × 1 = 1 2 2
 2  2 4 4 24
cos2 θ + cot 2 θ =   +  = + =
4. sin (θ + 36° ) = cos θ  5  1 5 1 5

or cos 90° − (θ + 36° ) = cos θ 9. 2


13
∴ 90° − (θ + 36° ) = θ ⇒ 2θ = 54° ⇒ θ = 27°
5

3 sin62° sec 42°


5. − 12 B
cos 28° cos ec48°
2 2
 5   5 
3cos (90° − 62° ) cos ec (90° − 42° ) tan2 B − sin2 B =   −  
= −  12   13 
cos 28° cos ec48°
25 × 169 − 25 × 144 25 × 25
3cos 28° cos ec 48° = =
= − = 3(1) – 1 = 2 144 × 169 144 × 169
cos 28° cos ec 48°
[ Q sin (90° – θ) = cos θ, cosec (90° – θ) = sec θ] 2
 5   5  25 × 25
2
sin2 B tan2 B =     =
   
13 12 169 × 144

QA - 33 Page 1
16. 3 We have, sin(60° + θ) − sin(60° − θ) = 2 cos 60°.sin θ
m2 − n2 m2 + n2
10. 2 sinθ = ⇒ cosec θ = [Q 2 cos A.sinB = sin(A + B) − sin(A − B)]
m2 + n2 m2 − n2
Squaring both side 1
= 2. .sin θ = sin θ
2
(m2 + n2 )
2

17. 1 2sin75°.cos15° = sin(75° + 15°) + sin(75° – 15°)


cosec2 θ=
(m2 − n2 )
2
By using [2sinA.cosB = sin(A + B) + sin(A – B)]

3 2+ 3
= sin90° + sin60° = 1 + =
m4 + n4 + 2m2n2 2 2
cot2 θ = cosec2 θ – 1 = –1
m4 + n4 − 2m2n2
tan A + tanB
4m2n2 18. 2 tan(A + B) =
cot 2 θ = 1 − (tan A × tanB)
2 2 2
(m − n )
7 1
m2 − n2 We have tan A = and tanB =
tan θ = 8 15
2mn
7 1
+
11. 4 cos240° cos120° – sin240° sin120° tan(A + B) = 8 15 = 105 + 8
7 1 120 − 7
we can also write this as 1− .
8 15
⇒ cos(270° – 30°).cos(90° + 30°)
– sin(270° – 30°) sin( 90° + 30°) π
113 π  
⇒ ( − sin30°)( − sin30°) − [− cos 30°.cos 30°]
tan(A + B) = = 1 = tan Q tan 4 = 1
113 4  

1 1 3 3 1 3 π
⇒ . + . = + =1 ∴A + B =
2 2 2 2 4 4 4

1 + sin θ 1 + sin θ 1 + sin θ (1 + cos 2A) + sin 2A 2 cos2 A + 2 sin A cos A


× = 19. 1 =
12. 3
1 − sin θ 1 + sin θ (1 − cos 2A) + sin 2A 2 sin2 A + 2 sin A cos A
1 − sin2 θ
Q (1 + cos 2θ = 2cos2 θ) and (1 − cos 2θ = 2sin2 θ)
1 + sin θ 1 + sin θ 1 sin θ  
= = = +
cos2 θ cos θ cos θ cos θ = sec θ + tan θ
2 cos A[cos A + sin A] cos A
= = = cot A
2 sin A[sin A + cos A] sin A
13. 1 cot 4 A − 1 = (cot 2 A)2 − 1 = (cot 2 A − 1)(cot 2 A + 1)
20. 4 A
= (cot 2 A − 1).cos ec 2 A [Q cos ec 2θ = 1 + cot 2 θ]

= (cos ec 2 A − 1 − 1) cos ec 2 A = cos ec 4 A − 2 cos ec 2 A 8m R o pe


P o le
14. 4 sin(θ + 45°) = sin θ.cos 45° + cos θ.sin 45° 3 0°
[using sin(A + B) = sinA cosB + cosA sinB] B C
In right-angled triangle ABC,
1 1 1
= sin θ. + cos θ. = [sin θ + cos θ] AB 1 8
2 2 2 sin30° = ⇒ = ⇒ AC = 16 m
AC 2 AC

cos2 θ 21. 4 A
2 −1
15. 3 We have cot θ − 1 sin2 θ
=
cot 2 θ + 1 cos ec 2θ

cos2 θ − sin2 θ sin2 θ θ


= × = cos2 θ − sin2 θ = cos 2θ
sin2 θ 1 B C
AC = height and BC = length of the shadow
(Q cos 2A = cos2 A − sin2 A) Let θ be the angle of elevation

Page 2 QA - 33
AC ⇒ x = (60 − h ) 3 ... (i)
∴ tan θ = But AC = BC
BC In ∆APQ,
∴ tan θ = 1 ⇒ tan θ = tan 45° 60
tan60° =
∴ θ = 45° x

22. 4 A ⇒ 3x = 60 ⇒ (60 − h ) 3 × 3 = 60
⇒ 60 − h = 20 ⇒ h = 40 m

h 25. 2 Let height of the hill be AD mts.


In ∆ ABD
6 0°
B C AD
3 6m
tan 45° =
30 + CD
Let the height of the wall (AC) = h mts.
A
h
Q tan 60° =
36
∴ h = 36 × tan 60°
h
h = 36 3
h = 36 × 1.732 4 5° 6 0°
h = 62. 353 ; 62.35 mts. B C D
3 0m
A ⇒ AD = 30 + CD
23. 2
In ∆ ACD
3 0° AD AD AD
C E tan 60° = ⇒ CD = =
CD tan 60° 3
5 6m
Combining the two equations we have
AD
AD = CD + 30 ⇒ AD = + 30
6 0° 3
B D
In ∆ABD
( )
30 3
AD 3 − 1 = 30 3 ⇒ AD =
AB AB 56 3 −1
tan 60° = ⇒ BD = =
BD tan 60° 3 ⇒ AD = 15 (3 + 3) = 15 × 4.732 = 70.980 = 70.98 mts
56 56
= mts = 3 mtr. 26. 4
3 3 B

24. 2 Q
30°
60° h
6 0– h
6 0m
θ 60°
B uild ing

30° D C A
B C
x 2x x

h h
Let AB be the flag staff and let θ be the angle
Tow e r

In ∆ABC,
h h
60° tan 60° = ⇒ 3 = ⇒h= 3x
x x
A x P In ∆ABD,
In ∆BCQ,
h 3x 1
60 − h 1 60 − h tan θ = ⇒ tan θ = ⇒ tan θ = ⇒ θ = 30°
tan30° = ⇒ = 3x 3x 3
x 3 x

QA - 33 Page 3
27. 4 A 29. 3 B

L ad de r

2 50 m 4 0m
W a ll
6 0° 3 0°
B C D
Let A be the point where rescue helicopter was and θ
AC be the height of helicopter from surface. C 2 0m A
In ∆ABC Let θ be the angle of elevation. Then
AC AC 20 1
tan 60° = ⇒ AC = tan 60° × BC = 3 × BC cos θ = ⇒ cos θ = = ⇒ θ = 60°
BC CB 40 2
AC 250
BC = = m. 30. 1 A
3 3
In ∆ACD
h
AC AC 250
tan 30° = ⇒ CD = = = 250 3 m.
CD tan 30° 1 Tow e r
3
Q Distance between the two person = BC + CD 3 0° 6 0°
O 2 00 m C x B
250  1 
= + 250 3 = 250  + 3
3  3  In ∆ABC,
h h
 1 + 3  1000 tan60° = ⇒ 3= ... (i)
= 250  = mts. x x
 3  3
In ∆ABO,
h 1 h
28. 4 A tan30° = ⇒ = ... (ii)
200 + x 3 200 + x
From (i) and (ii),
1 25 m
h = 100 3 m

3 0°
B C
Let AC be the height of the kite above the ground.
AC
Q sin 30° =
AB

1
⇒ AC = AB × Sin 30° = 125 × = 62.5 m
2

Page 4 QA - 33

You might also like